Sie sind auf Seite 1von 52

Student Name : <b>ashish rajput</b><br>Exam Name : <b>NEET-

4_2012_13</b><br><table width="700" cellpadding=3 align=center style="border:solid


1px black; border-collapse:collapse; font-family:Verdana; font-size:small "><tr
valign=top height="15" style="background-color:#050d1a;color:white;"><td width="30"
style="border:solid 1px black; border-collapse:collapse "><b>S.No.</b></td><td
width="250" style="border:solid 1px black; border-collapse:collapse
"><b>Question</b></td><td width="200" style="border:solid 1px black; border-
collapse:collapse "><b>Options</b></td><td width="50" style="border:solid 1px
black; border-collapse:collapse "><b>Correct</b></td><td width="250"
style="border:solid 1px black; border-collapse:collapse
"><b>Explain</b></td></tr><tr valign=top height="15" style="background-
color:#EFF3FB;"><td width="30" style="border:solid 1px black; border-
collapse:collapse ">1</td><td width="250" style="border:solid 1px black; border-
collapse:collapse ">All are the arteries of
sternomastoid except?</td><td width="200" style="border:solid 1px black; border-
collapse:collapse ">A: Occipital artery<br><br>B: Superior thyroid<br><br>C:
Inferior thyroid<br><br>D: Suprascapular<br><br></td><td width="50"
style="border:solid 1px black; border-collapse:collapse ">C</td><td width="250"
style="border:solid 1px black; border-collapse:collapse ">BD Chaurasia 4rd edtn pg
no. 73
Explanation:
Blood Supply
Arterial supply—one branch each from superior thyroid artery and suprascapular
artery and, two branches from the occipital artery veins follow the arteries. Veins
follow the arteries.</td></tr><tr valign=top height="15" style="background-
color:White;"><td width="30" style="border:solid 1px black; border-
collapse:collapse ">2</td><td width="250" style="border:solid 1px black; border-
collapse:collapse ">Taenia coli is present in?</td><td width="200"
style="border:solid 1px black; border-collapse:collapse ">A: Stomach<br><br>B:
Appendix<br><br>C: Small intestine<br><br>D: Pelvis<br><br></td><td width="50"
style="border:solid 1px black; border-collapse:collapse ">A</td><td width="250"
style="border:solid 1px black; border-collapse:collapse ">Clinical anatomy by
Harold Ellis 11th edtn pg no. 89
Explanation:
The stomach wall is reinforced by an innermost oblique coat of
muscle and the colon is characterized by the condensation of its longitudinal
layer into three taeniae coli.</td></tr><tr valign=top height="15"
style="background-color:#EFF3FB;"><td width="30" style="border:solid 1px black;
border-collapse:collapse ">3</td><td width="250" style="border:solid 1px black;
border-collapse:collapse ">Cells located in between the endothelium and the basal
lamina and involved in contractile activity in the nephron is called</td><td
width="200" style="border:solid 1px black; border-collapse:collapse ">A: mesangial
cells<br><br>B: interstitial cells<br><br>C: leydig cells<br><br>D: juxtaglomerular
cells<br><br></td><td width="50" style="border:solid 1px black; border-
collapse:collapse ">A</td><td width="250" style="border:solid 1px black; border-
collapse:collapse ">ref: Ganong 21st edition pg no ;
exp:The glomerulus, which is about 200 um in diameter, is formed by the
invagination of a tuft of capillaries into the dilated, blind end of the nephron
(Bowmans capsule). The capillaries are supplied by an afferent arteriole and
drained by a slightly smaller efferent arteriole .There are two cellular layers
separating the blood from the glomerular filtrate in Bowmans capsule: the capillary
endothelium and the specialized epithelium of the capsule that is made up of
podocytes overlying the glomerular capillaries . These layers are separated by a
basal lamina. Stellate cells called mesangial cells are located between the basal
lamina and the endothelium. They are similar to cells called pericytes, which are
found in the walls of capillaries elsewhere in the body. Mesangial cells are
especially common between two neighboring capillaries, and in these locations the
basal membrane forms a sheath shared by both capillaries . The mesangial cells are
contractile and play a role in the regulation of glomerular filtration .They also
secrete various substances, take up immune complexes, and are involved in the
production of glomerular disease</td></tr><tr valign=top height="15"
style="background-color:White;"><td width="30" style="border:solid 1px black;
border-collapse:collapse ">4</td><td width="250" style="border:solid 1px black;
border-collapse:collapse ">False is?</td><td width="200" style="border:solid 1px
black; border-collapse:collapse ">A: Cardiolipin Is a Major Lipid of
cytosol<br><br>B: Lysophospholipids Are Intermediates in the Metabolism of
Phosphoglycerols<br><br>C: Plasmalogens Occur in Brain & Muscle<br><br>D:
Sphingomyelins Are Found in the Nervous System<br><br></td><td width="50"
style="border:solid 1px black; border-collapse:collapse ">A</td><td width="250"
style="border:solid 1px black; border-collapse:collapse ">Reference: Biochemistry
harper 26th edition page no : 115
Explanation:
Cardiolipin Is a Major Lipid of Mitochondrial Membranes
Lysophospholipids Are Intermediates in the Metabolism of Phosphoglycerols
Plasmalogens Occur in Brain & Muscle
Sphingomyelins Are Found in the Nervous System
</td></tr><tr valign=top height="15" style="background-color:#EFF3FB;"><td
width="30" style="border:solid 1px black; border-collapse:collapse ">5</td><td
width="250" style="border:solid 1px black; border-collapse:collapse ">arrange the
following materials in the order of their linear coefficient of thermal expansion
from highest to the lowest:
1) tooth structure
2) Dental amalgam
3) Composites
4) Dental wax</td><td width="200" style="border:solid 1px black; border-
collapse:collapse ">A: 4-3-2-1<br><br>B: 4-2-3-1<br><br>C: 4-2-1-3<br><br>D: 2-4-3-
1<br><br></td><td width="50" style="border:solid 1px black; border-
collapse:collapse ">A</td><td width="250" style="border:solid 1px black; border-
collapse:collapse ">reference: Sturdevant 4th edition page no: 137
Explanation: dental material with their linear coefficient of thermal expansion:
1) aluminous dental porcelain= 4
2) In ceram= 8-10
3) traditional dental cements= 8-10
4) tooth structure = 9-11
5) stainless steel= 11
6) PFM ceramics or alloys= 14
7) gold foil = 14-15
8) gold casting alloy = 16-18
9) Co -Cr alloy = 18-20
10) hybrid glass ionomer = 20-25
11) dental amalgam = 25
12) composites= 28-50
13) direct filling resins= 72- 83
14) Waxes = 260-600</td></tr><tr valign=top height="15" style="background-
color:White;"><td width="30" style="border:solid 1px black; border-
collapse:collapse ">6</td><td width="250" style="border:solid 1px black; border-
collapse:collapse ">61. The most conspicuous clinical sign of right-sided heart
failure is</td><td width="200" style="border:solid 1px black; border-
collapse:collapse ">A: 1. hypertension.<br><br>B: 2. mitral stenosis.<br><br>C: 3.
pulmonary edema.<br><br>D: 4. systemic venous congestion.<br><br></td><td
width="50" style="border:solid 1px black; border-collapse:collapse ">D</td><td
width="250" style="border:solid 1px black; border-collapse:collapse ">Reference:
McMurray JJ, Pfeffer MA (2005). "Heart failure". Lancet 365 (9474): 1877–89
Explanation:Right-sided failure
Backward failure of the right ventricle leads to congestion of systemic
capillaries. This generates excess fluid accumulation in the body. This causes
swelling under the skin (termed peripheral edema or anasarca) and usually affects
the dependent parts of the body first (causing foot and ankle swelling in people
who are standing up, and sacral edema in people who are predominantly lying down).
Nocturia (frequent night time urination) may occur when fluid from the legs is
returned to the bloodstream while lying down at night. In progressively severe
cases, ascites (fluid accumulation in the abdominal cavity causing swelling) and
hepatomegaly (enlargement of the liver) may develop. Significant liver congestion
may result in impaired liver function, and jaundice and even coagulopathy (problems
of decreased blood clotting) may occur.</td></tr><tr valign=top height="15"
style="background-color:#EFF3FB;"><td width="30" style="border:solid 1px black;
border-collapse:collapse ">7</td><td width="250" style="border:solid 1px black;
border-collapse:collapse ">CAD/CAM restorations are routinely cemented with</td><td
width="200" style="border:solid 1px black; border-collapse:collapse ">A: Zinc
phosphate<br><br>B: glass-ionomer cements<br><br>C: composites<br><br>D: any of the
above<br><br></td><td width="50" style="border:solid 1px black; border-
collapse:collapse ">C</td><td width="250" style="border:solid 1px black; border-
collapse:collapse ">reference: Sturdevant 4th edition page no:
Explanation:
CAD/CAM restorations are routinely cemented with moderately filled composites
(homogeneous microfills or hybrids). The composite cements are not as mechanically
strong as composite restorative materials but do provide the best abrasion
resistance because of the microprotection effect of closely spaced filler
particles.

Zinc phosphate and glass-ionomer cements are not recommended for use with milled
ceramic restorations</td></tr><tr valign=top height="15" style="background-
color:White;"><td width="30" style="border:solid 1px black; border-
collapse:collapse ">8</td><td width="250" style="border:solid 1px black; border-
collapse:collapse ">thermal shock : cracking and crazing of the fired dental
porcelain results due to</td><td width="200" style="border:solid 1px black;
border-collapse:collapse ">A: too rapid cooling after glazing<br><br>B: Very slow
cooling<br><br>C: both of the above<br><br>D: none of the above<br><br></td><td
width="50" style="border:solid 1px black; border-collapse:collapse ">C</td><td
width="250" style="border:solid 1px black; border-collapse:collapse ">reference:
Craig dental materials page no: 556
Explanation:
Too-rapid cooling of the outer layers may result in surface crazing or cracking;
this is also called thermal shock.
Very slow cooling (e.g., in a furnace) as well as multiple firings, might induce
the formation of additional leucite and increase the overall coefficient of thermal
expansion of the ceramic, and may also result in surface cracking and crazing.

Slow cooling is preferred, and is accomplished by removing the fired restoration


from the furnace as soon as the firing is finished and placing it under a glass
cover to protect it from air currents and possible contamination by
dirt.</td></tr><tr valign=top height="15" style="background-color:#EFF3FB;"><td
width="30" style="border:solid 1px black; border-collapse:collapse ">9</td><td
width="250" style="border:solid 1px black; border-collapse:collapse ">Growth on a
cell-free artificial solid medium is possible for following except</td><td
width="200" style="border:solid 1px black; border-collapse:collapse ">A: Ureaplasma
urealyticum<br><br>B: Mycoplasma Pneumoniae<br><br>C: C and L form of proteus
vulgaris<br><br>D: Chlamydia<br><br></td><td width="50" style="border:solid 1px
black; border-collapse:collapse ">D</td><td width="250" style="border:solid 1px
black; border-collapse:collapse ">Reference Ananthanarayan 6th edtn pg no. 389
Explanation:
Chlamydiae are obligate intracellular bacterial parasites of humans, animals and
birds with tropism for squamous epithelial cells and macrophages of the respiratory
and gastrointestinal tracts. Due to their filter-ability and failure to grow in
cell-free media, they were considered to be viruses. Based on the human diseases
they were then known to cause, they were called psittacosis-lymphogranuloma-
trachoma (PLT)</td></tr><tr valign=top height="15" style="background-
color:White;"><td width="30" style="border:solid 1px black; border-
collapse:collapse ">10</td><td width="250" style="border:solid 1px black; border-
collapse:collapse ">All are short acting bronchodilators except</td><td
width="200" style="border:solid 1px black; border-collapse:collapse ">A:
Salbutamol<br><br>B: Salmetrol<br><br>C: Terbutaline<br><br>D:
bitolterol<br><br></td><td width="50" style="border:solid 1px black; border-
collapse:collapse ">B</td><td width="250" style="border:solid 1px black; border-
collapse:collapse ">K D Tripathi 5th edtn pg no. 200
Explanation:
Salmetrol is a long acting selective beta 1 agonist with slow onset of action :
used daily twice by inhalation for maintenance therapy or nocturnal asthma but not
for acute asthma</td></tr><tr valign=top height="15" style="background-
color:#EFF3FB;"><td width="30" style="border:solid 1px black; border-
collapse:collapse ">11</td><td width="250" style="border:solid 1px black; border-
collapse:collapse ">All are true of omeprazole except?</td><td width="200"
style="border:solid 1px black; border-collapse:collapse ">A: Used in zollinger-
ellison syndrome<br><br>B: Inhibits oxidation of the
warfarin<br><br>C: Inactivates H+K+ATPase reversibly<br><br>D: It is a proton pump
inhibitor<br><br></td><td width="50" style="border:solid 1px black; border-
collapse:collapse ">C</td><td width="250" style="border:solid 1px black; border-
collapse:collapse ">K D Tripathi 5th edtn pg no. 591
Explanation:
omeprazole at pH less than 5 rearranges to two charged cationic forms that react
covalently with SH groups of H+K+ATPase and inactivate it
irreversibily.</td></tr><tr valign=top height="15" style="background-
color:White;"><td width="30" style="border:solid 1px black; border-
collapse:collapse ">12</td><td width="250" style="border:solid 1px black; border-
collapse:collapse ">Benign tumor of striated muscles</td><td width="200"
style="border:solid 1px black; border-collapse:collapse ">A: Leiomyoma<br><br>B:
rhabomyoma<br><br>C: leiomyosarcoma<br><br>D: rhabdomyosarcoma<br><br></td><td
width="50" style="border:solid 1px black; border-collapse:collapse ">B</td><td
width="250" style="border:solid 1px black; border-collapse:collapse ">Harshmohan
3th edition pg no 293
Leiomyoma-benign tumor of smooth muscle
Rhabdomyoma-benign tumor of striated muscle
Leiomyosarcoma-malignant tumor of smooth muscle
Rhabdomyosarcoma-malignant tumor of striated muscle</td></tr><tr valign=top
height="15" style="background-color:#EFF3FB;"><td width="30" style="border:solid
1px black; border-collapse:collapse ">13</td><td width="250" style="border:solid
1px black; border-collapse:collapse ">Difference between primary and permanent
enamel is</td><td width="200" style="border:solid 1px black; border-
collapse:collapse ">A: in prism arrange ment<br><br>B: in mineral
conntent<br><br>C: no difference in enamel,difference in dentin<br><br>D:
none<br><br></td><td width="50" style="border:solid 1px black; border-
collapse:collapse ">A</td><td width="250" style="border:solid 1px black; border-
collapse:collapse ">Reference and Explaination,
WHEELERS DENTAL ANATOMY PG no. 53
this question was asked in AIIMS.
In decidous dentition the rods are horizontal at the cervical and central parts of
crown in decidous teeth while in permanent teeth the rods bent cervically in the
cerviacl region.</td></tr><tr valign=top height="15" style="background-
color:White;"><td width="30" style="border:solid 1px black; border-
collapse:collapse ">14</td><td width="250" style="border:solid 1px black; border-
collapse:collapse ">the smallest root of maxillary first molar is</td><td
width="200" style="border:solid 1px black; border-collapse:collapse ">A: palatal
root<br><br>B: mesio buccal root<br><br>C: disto buccal root<br><br>D:
none<br><br></td><td width="50" style="border:solid 1px black; border-
collapse:collapse ">C</td><td width="250" style="border:solid 1px black; border-
collapse:collapse ">Reference and Explaination,
WHEELERS DENTAL ANATOMY PG no.273
palatal root is the largest and disto buccal root is the smallest of the three
roots of maxilllary first molar.</td></tr><tr valign=top height="15"
style="background-color:#EFF3FB;"><td width="30" style="border:solid 1px black;
border-collapse:collapse ">15</td><td width="250" style="border:solid 1px black;
border-collapse:collapse ">During the bell stage the cells covering the convex
surface & the concave surface of the bell are</td><td width="200"
style="border:solid 1px black; border-collapse:collapse ">A: inner enamel
epithelium,outer enamel epithelium<br><br>B: outer enamel epithelium,inner enamel
epithelium<br><br>C: stratum intermedium ,inner enamel epithelium<br><br>D:
stellate reticulum,outer enamel epithelium<br><br></td><td width="50"
style="border:solid 1px black; border-collapse:collapse ">B</td><td width="250"
style="border:solid 1px black; border-collapse:collapse ">REF :JAMES.K.AVERY pg no
76; During the bell stage there are four layers 1) the convex layer of the bell
covered by outer enamel epithelium 2)the concave surface by the inner enamel
epithelium 3)stratum intermedium adjacent to the inner enamel epithelium 4)
stellate reticulum forming the centre also called as the enamel pulp</td></tr><tr
valign=top height="15" style="background-color:White;"><td width="30"
style="border:solid 1px black; border-collapse:collapse ">16</td><td width="250"
style="border:solid 1px black; border-collapse:collapse ">Basic unit of enamel
is</td><td width="200" style="border:solid 1px black; border-collapse:collapse
">A: enamel prism<br><br>B: enamel rod<br><br>C: enamel knot<br><br>D: enamel
cord<br><br></td><td width="50" style="border:solid 1px black; border-
collapse:collapse ">B</td><td width="250" style="border:solid 1px black; border-
collapse:collapse ">REF :TEN CATE pg no240 : Basic unit of enamel was earlier
called enamel prism but enamel rod is a more appropriate term as it doesn’t exactly
resemble a prism.</td></tr><tr valign=top height="15" style="background-
color:#EFF3FB;"><td width="30" style="border:solid 1px black; border-
collapse:collapse ">17</td><td width="250" style="border:solid 1px black; border-
collapse:collapse ">most common cyanotic congenital heart disease</td><td
width="200" style="border:solid 1px black; border-collapse:collapse ">A:
asd<br><br>B: vsd<br><br>C: tatralogy of fallot<br><br>D: pentalogy of
falloty<br><br></td><td width="50" style="border:solid 1px black; border-
collapse:collapse ">C</td><td width="250" style="border:solid 1px black; border-
collapse:collapse ">reference and explaination;
davidsons 20th edition pg no. 639
refer page no 639 and see the difference between trilogy, tetralogy and pentalogy
of fallot.</td></tr><tr valign=top height="15" style="background-color:White;"><td
width="30" style="border:solid 1px black; border-collapse:collapse ">18</td><td
width="250" style="border:solid 1px black; border-collapse:collapse ">exudative
pleural effusion is seen in all except</td><td width="200" style="border:solid 1px
black; border-collapse:collapse ">A: tuberculosis<br><br>B: rheumatoid
arthritis<br><br>C: sle<br><br>D: cardiac failure<br><br></td><td width="50"
style="border:solid 1px black; border-collapse:collapse ">D</td><td width="250"
style="border:solid 1px black; border-collapse:collapse ">reference and
explaination;
davidsons 20th edition pg no

tuberculosis is an important cause of exudativeascites and not transudative


ascites.other conditions producing similar lesions are malignant
diseases,immunologic diseases like sle,rheumatic heart disease etc..</td></tr><tr
valign=top height="15" style="background-color:#EFF3FB;"><td width="30"
style="border:solid 1px black; border-collapse:collapse ">19</td><td width="250"
style="border:solid 1px black; border-collapse:collapse ">a 30 year old patient of
severe burns , needs transfusion to restore blood volume, the blood group cross
matching reports are not available, the treatment of choice</td><td width="200"
style="border:solid 1px black; border-collapse:collapse ">A: start the slow blood
transfusion without crossmatching , as it is emergency situation<br><br>B: Use
Human albumin 4.5 per cent<br><br>C: Use Dextrans<br><br>D: any of the
above<br><br></td><td width="50" style="border:solid 1px black; border-
collapse:collapse ">B</td><td width="250" style="border:solid 1px black; border-
collapse:collapse ">reference: Love and Bailey page no: 54
explanation:
One of the most urgent requirements in a patient suffering from acute blood loss is
the re-establishment of a normal blood volume. This may be achieved satisfactorily
with a number of plasma substitutes.
Human albumin 4.5 per cent has superseded the use of dried plasma and can be used
whilst cross-matching is being performed. Two to three units (1.2 litres) are given
intravenously over 30 minutes. It is valuable in patients with burns where there
has been severe loss of protein. There is no risk of transmitting
hepatitis.</td></tr><tr valign=top height="15" style="background-color:White;"><td
width="30" style="border:solid 1px black; border-collapse:collapse ">20</td><td
width="250" style="border:solid 1px black; border-collapse:collapse ">surgical
cricothyroidotomy is contraindicated at</td><td width="200" style="border:solid
1px black; border-collapse:collapse ">A: < 18 years of age<br><br>B: < 12 years of
age<br><br>C: > 40 years of age<br><br>D: a and c<br><br></td><td width="50"
style="border:solid 1px black; border-collapse:collapse ">B</td><td width="250"
style="border:solid 1px black; border-collapse:collapse ">reference: Love and
Bailey page no: 210
explanation:
• Airway patency is then maintained by chin lift or jaw thrust maneuvers, lifting
the mandible forwards and, if appropriate, inserting an airway device
oropharyngeal/nasopharyngeal or endotracheal according to clinical
judgement and expertise available).
• If unable to open the airway by the above, a surgical cricothyroidotomy may be
performed in patients over the age of 12 years by inserting a 6-mm paediatric
cuffed tracheostomy tube through the cricothyroid membrane . Under the age of 12
years the cricoid membrane is very narrow and the cricoid cartilage is the only
complete ring preventing airway collapse. surgical cricothyroidotomy is not done.
Under these circumstances, a needle cricothyroidotomy may buy some time (20
minutes) provided that a means of jet-insufflating oxygen through the needle is
available.</td></tr><tr valign=top height="15" style="background-
color:#EFF3FB;"><td width="30" style="border:solid 1px black; border-
collapse:collapse ">21</td><td width="250" style="border:solid 1px black; border-
collapse:collapse ">Which of th e following is true about the biochemical findings
of pagets disease are true</td><td width="200" style="border:solid 1px black;
border-collapse:collapse ">A: Serum alkaline phosphatase level may exceed 50
bodansky units in monostotic type<br><br>B: Elevated urinary hydroxy proline levels
are elevated<br><br>C: Urinary excreation of ntx and ctx(telopeptide)<br><br>D: All
of the above<br><br></td><td width="50" style="border:solid 1px black; border-
collapse:collapse ">D</td><td width="250" style="border:solid 1px black; border-
collapse:collapse ">Shafer’s 5th edtn pg 1005
Bichemical findings in paget’s disease
1)serum alp goes up to 50 bodansky units in monostotic type and 250 units in
polyostotic type
2)urinary hydroxy proline levels are elevated
3)more recently urinary excreation of pyridinium collagen cross links,urinary
Ntx(telopeptide),and alpha ctx have been found more sensitive indicator of bone
resorption
</td></tr><tr valign=top height="15" style="background-color:White;"><td width="30"
style="border:solid 1px black; border-collapse:collapse ">22</td><td width="250"
style="border:solid 1px black; border-collapse:collapse ">Most common form of
actinomycosis is</td><td width="200" style="border:solid 1px black; border-
collapse:collapse ">A: Cervicofacial<br><br>B: Abdominal<br><br>C:
Pulmonary<br><br>D: All are equal<br><br></td><td width="50" style="border:solid
1px black; border-collapse:collapse ">A</td><td width="250" style="border:solid
1px black; border-collapse:collapse ">Shafer’s 5th edition
Cervicofacial actinomycosis is the most common form of yhis disease
Greatest interest to the dentist.
Sulphur granules are present.
</td></tr><tr valign=top height="15" style="background-color:#EFF3FB;"><td
width="30" style="border:solid 1px black; border-collapse:collapse ">23</td><td
width="250" style="border:solid 1px black; border-collapse:collapse ">All are
features of hyperthyroidism except</td><td width="200" style="border:solid 1px
black; border-collapse:collapse ">A: Heat intolerance<br><br>B: Goitre<br><br>C:
Bradycardia<br><br>D: Weight loss<br><br></td><td width="50" style="border:solid
1px black; border-collapse:collapse ">C</td><td width="250" style="border:solid
1px black; border-collapse:collapse ">Reference: Burkets oral medicine 10th
edition, Page no 587
Explanation: In hyperthyoidism there is tachycardia (increased pulse
rate)</td></tr><tr valign=top height="15" style="background-color:White;"><td
width="30" style="border:solid 1px black; border-collapse:collapse ">24</td><td
width="250" style="border:solid 1px black; border-collapse:collapse ">Scrofula
is</td><td width="200" style="border:solid 1px black; border-collapse:collapse
">A: Tuberculosis of skin<br><br>B: Tuberculosis of lymph nodes<br><br>C:
Tuberculosis of jaws<br><br>D: Tuberculosis of spine<br><br></td><td width="50"
style="border:solid 1px black; border-collapse:collapse ">B</td><td width="250"
style="border:solid 1px black; border-collapse:collapse ">Reference: Burkets oral
medicine 10th edition, Page no 528
Explanation: Scrofula is TB of lymph nodes. TB of skin is lupus vulgaris and TB of
spine is Potts disease</td></tr><tr valign=top height="15" style="background-
color:#EFF3FB;"><td width="30" style="border:solid 1px black; border-
collapse:collapse ">25</td><td width="250" style="border:solid 1px black; border-
collapse:collapse ">The following is not true about alarm clock haedache</td><td
width="200" style="border:solid 1px black; border-collapse:collapse ">A: a.k.a
sphenopalatine neueralgia<br><br>B: Unilateral paroxysms of intense pain in the
region of the eyes,the maxilla.the ear and mastoid,base of the nose,and beneath the
zygoma.<br><br>C: Pain is thought to be generated at the level of the peri
carotid/cavernous sinus complex.<br><br>D: None<br><br></td><td width="50"
style="border:solid 1px black; border-collapse:collapse ">D</td><td width="250"
style="border:solid 1px black; border-collapse:collapse ">Shafer’s 5th edition pg
1166
Sphenopalatine neuralgia is apin syndrome complex.
►idiopathic syndrome consisting of recurrent brief attacks of
sudden,severe,unilateral peri orbital pain.
►attributed to hypothalamic hormonal influences.
►Pain is thought to generated at the level of the pericarotid /cavernous sinus
complex.
►unilateral intense pain in the region of the eyes,the maxilla,the ear and
mastoid,base of the nose.
►attack develops regularly,usually atleast once a day,over a prolonged period of
time.(alarm clock headache)
</td></tr><tr valign=top height="15" style="background-color:White;"><td width="30"
style="border:solid 1px black; border-collapse:collapse ">26</td><td width="250"
style="border:solid 1px black; border-collapse:collapse ">Carbon dioxide is
transported in four different chemical forms. Which of the following is the
principal mode of transport?</td><td width="200" style="border:solid 1px black;
border-collapse:collapse ">A: Dissolvedcarbon dioxide<br><br>B: Carbamino
compounds<br><br>C: Bicarbonate ions<br><br>D: Carbonic acid<br><br></td><td
width="50" style="border:solid 1px black; border-collapse:collapse ">C</td><td
width="250" style="border:solid 1px black; border-collapse:collapse ">Reference:
Guyton, Physiology, 11th ed, pg no: 511
Explanation:
1.Transport of Carbon Dioxide in Combination with Hemoglobin and Plasma Proteins—
Carbaminohemoglobin: In addition to reacting with water, carbon dioxide reacts
directly with amine radicals of the hemoglobin molecule to form the compound
carbaminohemoglobin (CO2Hgb). This combination of carbon dioxide and hemoglobin is
a reversible reaction that occurs with a loose bond, so that the carbon dioxide is
easily released into the alveoli, where the Pco2 is lower than in the pulmonary
capillaries.The quantity of carbon dioxide that can be carried from the peripheral
tissues to the lungs by carbamino combination with hemoglobin and plasma proteins
is about 30 per cent of the total quantity transported— that is, normally about 1.5
milliliters of carbon dioxide in each 100 milliliters of blood. However, because
this reaction is much slower than the reaction of carbon dioxide with water inside
the red blood cells, it is doubtful that under normal conditions this carbamino
mechanism transports more than 20 per cent of thetotal carbon dioxide
2.Dissociation of Carbonic Acid into Bicarbonate and Hydrogen Ions. In another
fraction of a second, the carbonic acid formed in the red cells (H2CO3) dissociates
into hydrogen and bicarbonate ions (H+ and HCO3 –). Most of the hydrogen ions then
combine with the hemoglobin in the red blood cells, because the hemoglobin protein
is a powerful acid-base buffer. In turn, many of the bicarbonate ions diffuse from
the red cells into the plasma, while chloride ions diffuse into
the red cells to take their place. This is made possible by the presence of a
special bicarbonate-chloride carrier protein in the red cell membrane that shuttles
these two ions in opposite directions at rapid velocities. Thus, the chloride
content of venous red blood cells is greater than that of arterial red cells, a
phenomenon called the chloride shift.
The reversible combination of carbon dioxide with water in the red blood cells
under the influence of carbonic anhydrase accounts for about 70 per cent of the
carbon dioxide transported from the tissues to the lungs.
3.Transport of Carbon Dioxide in the Dissolved State. A small portion of the carbon
dioxide is transported in the dissolved
state to the lungs. This is about 7 percent of the total CO2.</td></tr><tr
valign=top height="15" style="background-color:#EFF3FB;"><td width="30"
style="border:solid 1px black; border-collapse:collapse ">27</td><td width="250"
style="border:solid 1px black; border-collapse:collapse ">which of the following
result in the highest dose to the gonads</td><td width="200" style="border:solid
1px black; border-collapse:collapse ">A: skull radiograph<br><br>B: Radiographs
involving the extremities<br><br>C: retrograde pyelogram<br><br>D: Radiographs
involving the neck<br><br></td><td width="50" style="border:solid 1px black;
border-collapse:collapse ">C</td><td width="250" style="border:solid 1px black;
border-collapse:collapse ">Ref: White & Pharoah 5th Ed Pg 53. Radiographs of the
abdomen result in the highest dose to the gonads; those involving the head, neck,
and extremities result in the lowest. For example, a radiograph of the kidneys,
ureters, and bladder (retrograde pyelogram) delivers a gonad dose of 1.07mGy to
women and 0.08mGy to men, whereas a radiograph of the skull delivers a gonadal dose
of less than 0.005mGy in both sexes. Dental x-ray examinations result in a
genetically insignificant dose of only 0.001 mGy.</td></tr><tr valign=top
height="15" style="background-color:White;"><td width="30" style="border:solid 1px
black; border-collapse:collapse ">28</td><td width="250" style="border:solid 1px
black; border-collapse:collapse ">Indications for a preoperative radiograph include
all except:</td><td width="200" style="border:solid 1px black; border-
collapse:collapse ">A: history of difficult extractions<br><br>B: if the tooth is
very difficult to remove by forceps<br><br>C: an isolated maxillary molar<br><br>D:
all of the above.<br><br></td><td width="50" style="border:solid 1px black;
border-collapse:collapse ">D</td><td width="250" style="border:solid 1px black;
border-collapse:collapse ">Ref: Geoffrey L. Howe 2nd Ed Pg 5, 6 & 7. Indications
for a preoperative radiograph include: history of difficult / attempted
extractions; tooth abnormally resistant to forceps extraction; if, after clinical
examination, it has been decided to remove the tooth by dissection; teeth / roots
in close relationship to maxillary antrum, inferior alveolar or mental nerves;
third molars, instanding premolars, or misplaced canines; heavily restored teeth;
any tooth affected by periodontal disease accompanied by some sclerosis of the
supporting bone (as they are often hypercementosed); teeth subjected to trauma; an
isolated maxillary molar as the bone may be weakened by the presence of a large
antrum; any partially or unerupted tooth; any tooth with abnormal crown or delayed
eruption.</td></tr><tr valign=top height="15" style="background-color:#EFF3FB;"><td
width="30" style="border:solid 1px black; border-collapse:collapse ">29</td><td
width="250" style="border:solid 1px black; border-collapse:collapse ">Grooving of
the root (of 3rd molar) due to Inferior Alveolar Canal (IAC) is usually situated
on:</td><td width="200" style="border:solid 1px black; border-collapse:collapse
">A: buccal aspect of the root<br><br>B: lingual aspect of the root<br><br>C: tip
of the root<br><br>D: any aspect of the root<br><br></td><td width="50"
style="border:solid 1px black; border-collapse:collapse ">B</td><td width="250"
style="border:solid 1px black; border-collapse:collapse ">Ref: Chitra Chakravarthy
1st Ed Pg 95. On a periapical radiograph, if the band of radiolucency crosses the
root and coincides with the outline of the IAC then it indicates grooving of the
root by IAC. This is because of the decreased amount of tooth substance between X-
ray source and film at the level of the groove. Grooving due to IAC is usually
situated on the lingual surface of the roots.</td></tr><tr valign=top height="15"
style="background-color:White;"><td width="30" style="border:solid 1px black;
border-collapse:collapse ">30</td><td width="250" style="border:solid 1px black;
border-collapse:collapse ">If 0.0125 mg/ml of vasoconstrictor is present in a LA
solution, then the ratio is</td><td width="200" style="border:solid 1px black;
border-collapse:collapse ">A: 1:25,000<br><br>B: 1:50,000<br><br>C:
1:40,000<br><br>D: 1:80,000<br><br></td><td width="50" style="border:solid 1px
black; border-collapse:collapse ">D</td><td width="250" style="border:solid 1px
black; border-collapse:collapse ">Ref; Malamed 2nd Ed Pg 34. The concentration of
1:80,000 vasoconstrictor is 0.0125 mg/ml.</td></tr><tr valign=top height="15"
style="background-color:#EFF3FB;"><td width="30" style="border:solid 1px black;
border-collapse:collapse ">31</td><td width="250" style="border:solid 1px black;
border-collapse:collapse ">all of the following are the examples of the rigid
fixation except</td><td width="200" style="border:solid 1px black; border-
collapse:collapse ">A: A large compression
plate placed at the inferior border of a body fracture combined with an arch
bar<br><br>B: Two lag screws inserted across a symphysis fracture<br><br>C: Champys
technique<br><br>D: none of the above<br><br></td><td width="50"
style="border:solid 1px black; border-collapse:collapse ">C</td><td width="250"
style="border:solid 1px black; border-collapse:collapse ">Reference: Peterson
Principals of Oral Surgery 2nd edition page number: 373 fig: 20. 4
Explaination:
The Champy method of treating angle fractures using a single, noncompression
miniplate attached with 2.0 mm monocortical screws. Because this plate is placed in
the most biomechanically advantageous area for this region (superior border), a
small plate can neutralize the functional forces and permit active use of the
mandible during the healing process.However, although this technique is
functionally stable,interfragmentary motion probably occurs to some extent during
function. It is therefore not rigid fixation.</td></tr><tr valign=top height="15"
style="background-color:White;"><td width="30" style="border:solid 1px black;
border-collapse:collapse ">32</td><td width="250" style="border:solid 1px black;
border-collapse:collapse ">which of the following critical analysis guide is used
kimprove the review of RCT</td><td width="200" style="border:solid 1px black;
border-collapse:collapse ">A: CONSORT<br><br>B: QUORUM<br><br>C: CASP<br><br>D: A
and C<br><br></td><td width="50" style="border:solid 1px black; border-
collapse:collapse ">D</td><td width="250" style="border:solid 1px black; border-
collapse:collapse ">reference: carranza 10th edition page no : 19
Explanation:
1) CONSORTstatement (Consolidated Standards of Reporting Trials)- To improve the
reporting and review of RCTs.
2) QUOROM (Quality of Reporting of Meta-Analyses)- To improve the reporting and
review of SRs
3) CASP(Critical Appraisal SkillsProgram): To review RCTs,SRs, and several other
types of studies.</td></tr><tr valign=top height="15" style="background-
color:#EFF3FB;"><td width="30" style="border:solid 1px black; border-
collapse:collapse ">33</td><td width="250" style="border:solid 1px black; border-
collapse:collapse ">The dermis may be classified as what type of connective tissue?
</td><td width="200" style="border:solid 1px black; border-collapse:collapse ">A:
Modified elastic tissue<br><br>B: 2. Reticular connective tissue<br><br>C: Dense
regular connective tissue<br><br>D: Dense irregular connective
tissue<br><br></td><td width="50" style="border:solid 1px black; border-
collapse:collapse ">D</td><td width="250" style="border:solid 1px black; border-
collapse:collapse ">Reference:Essentials of medical physiology, Sembulingam, 2nd
ed, pg no: 256
Explanation:The dermis is the layer of skin beneath the epidermis that consists of
dense irregular connective tissue and cushions the body from stress and strain. The
dermis provides tensile strength and elasticity to the skin through an
extracellular matrix composed of collagen fibrils, microfibrils, and elastic
fibers, embedded in proteoglycans.
It harbors many Mechanoreceptors (nerve endings) that provide the sense of touch
and heat. It also contains the hair follicles, sweat glands, sebaceous glands,
apocrine glands, lymphatic vessels and blood vessels. The blood vessels in the
dermis provide nourishment and waste removal from its own cells as well as for the
epidermis.</td></tr><tr valign=top height="15" style="background-color:White;"><td
width="30" style="border:solid 1px black; border-collapse:collapse ">34</td><td
width="250" style="border:solid 1px black; border-collapse:collapse ">true about
cemental repair</td><td width="200" style="border:solid 1px black; border-
collapse:collapse ">A: cemental repair needs viable pulp<br><br>B: cemental repair
needs presence of viable connective tissue<br><br>C: Cementum repair
can occur in devitalized as well as vital teeth.<br><br>D: b and c<br><br></td><td
width="50" style="border:solid 1px black; border-collapse:collapse ">D</td><td
width="250" style="border:solid 1px black; border-collapse:collapse ">reference:
carranza 10th edition page no : 78
Explanation:Cementum repair requires the presence of viable connective tissue. If
epithelium proliferates into an area of resorption, repair will not take place.
Cementum repair can occur in devitalized as well as vital teeth.</td></tr><tr
valign=top height="15" style="background-color:#EFF3FB;"><td width="30"
style="border:solid 1px black; border-collapse:collapse ">35</td><td width="250"
style="border:solid 1px black; border-collapse:collapse ">the HLA antigen that
appears to be less prevalant in agreesive periodontitis</td><td width="200"
style="border:solid 1px black; border-collapse:collapse ">A: HLA-A9<br><br>B: HLA-
B15<br><br>C: HLA-A2<br><br>D: any of the above<br><br></td><td width="50"
style="border:solid 1px black; border-collapse:collapse ">C</td><td width="250"
style="border:solid 1px black; border-collapse:collapse ">reference: carranza 10th
edition page no : 200
Explanation:
Two antigens that a ear to be consistently associated with diseases are HLA-A9 and
B15. The risk of disease in subjects with HLA-A9 or B15 is about 1.5 to 3.5 times
greate than in those lacking these antigens.
In contrast, th HLA-A2 ntigen appears to be less prevalent in patients with
aggressive periodontitiS than controls, suggesting that this antigen somehow may be
protective.</td></tr><tr valign=top height="15" style="background-color:White;"><td
width="30" style="border:solid 1px black; border-collapse:collapse ">36</td><td
width="250" style="border:solid 1px black; border-collapse:collapse ">Cyclic
ingress and egress of fluids at
restoration margin is called</td><td width="200" style="border:solid 1px black;
border-collapse:collapse ">A: imbibition<br><br>B: syneresis<br><br>C:
percolation<br><br>D: both a and b<br><br></td><td width="50" style="border:solid
1px black; border-collapse:collapse ">C</td><td width="250" style="border:solid
1px black; border-collapse:collapse ">Sturdevent 4th edtn page no. 138,
Explanation:
Cyclic ingress and egress of fluids at the restoration margin is called
percolation. Also Percolation along the margins of an amalgam restoration is due to
its difference in linear coefficient of thermal expansion from tooth structure
during intraoral temperature changes. Fluid influx occurs during cooling
(contraction). Fluid efflux occurs during heating (expansion).</td></tr><tr
valign=top height="15" style="background-color:#EFF3FB;"><td width="30"
style="border:solid 1px black; border-collapse:collapse ">37</td><td width="250"
style="border:solid 1px black; border-collapse:collapse ">The concept of phosphoric
acid-etching of dentin before application of a phosphate ester-type bonding agent
was introduced by</td><td width="200" style="border:solid 1px black; border-
collapse:collapse ">A: Bowden<br><br>B: Starsky<br><br>C: Bunocore<br><br>D:
Fusayama<br><br></td><td width="50" style="border:solid 1px black; border-
collapse:collapse ">D</td><td width="250" style="border:solid 1px black; border-
collapse:collapse ">Sturdevent 4th edtn page no. 244,
Explanation:
The concept of phosphoric acid-etching of dentin before application of a phosphate
ester-type bonding agent was introduced by Fusayama and others in
1979.</td></tr><tr valign=top height="15" style="background-color:White;"><td
width="30" style="border:solid 1px black; border-collapse:collapse ">38</td><td
width="250" style="border:solid 1px black; border-collapse:collapse ">valuable
observations in
diagnosing the condition of vital pulp exposure in the primary pulp</td><td
width="200" style="border:solid 1px black; border-collapse:collapse ">A: The size
of the exposure<br><br>B: The appearance of the pulp<br><br>C: The amount of
bleeding<br><br>D: all of the above<br><br></td><td width="50" style="border:solid
1px black; border-collapse:collapse ">D</td><td width="250" style="border:solid
1px black; border-collapse:collapse ">ref mcdonald pg no 396:The size of the
exposure, the appearance of the pulp, and the amount of bleeding are valuable
observations in diagnosing the condition of the primary pulp. For this reason the
use of a rubber dam to isolate the tooth is extremely important; in addition, with
the rubber dam the area can be kept clean and the work can be done more
efficient.The most favorable condition for vital pulp therapy is the small pinpoint
exposure surrounded by sound dentin. However, a true carious exposure, even of
pinpoint size, will be accompanied by inflammation of the
pulp, the degree of which is usually directly related to the
size of the exposure</td></tr><tr valign=top height="15" style="background-
color:#EFF3FB;"><td width="30" style="border:solid 1px black; border-
collapse:collapse ">39</td><td width="250" style="border:solid 1px black; border-
collapse:collapse ">tell-show do technique was introduced by</td><td width="200"
style="border:solid 1px black; border-collapse:collapse ">A: addelston<br><br>B:
craig<br><br>C: christopher<br><br>D: keith moore<br><br></td><td width="50"
style="border:solid 1px black; border-collapse:collapse ">A</td><td width="250"
style="border:solid 1px black; border-collapse:collapse ">ref mcdonald pg no 47:
addelston developed tell-show-do technique, which is one of the most common
behaviour shaping tool used in pedodontics</td></tr><tr valign=top height="15"
style="background-color:White;"><td width="30" style="border:solid 1px black;
border-collapse:collapse ">40</td><td width="250" style="border:solid 1px black;
border-collapse:collapse ">true about action of fluoride</td><td width="200"
style="border:solid 1px black; border-collapse:collapse ">A: production or
composition of enamel matrix is altered during ameloblastic secretory
stage<br><br>B: interference in the initial calcification stages<br><br>C:
ameloblast function is altered<br><br>D: all of the above<br><br></td><td
width="50" style="border:solid 1px black; border-collapse:collapse ">D</td><td
width="250" style="border:solid 1px black; border-collapse:collapse ">ref cameron
text book of peadiatric dentistry pg no 42:the interence in initial calcification
process caused by change in the ion transport mechanism, ameloblast function may be
altered. Fluoride toxicity can affect the nucleation and crystal growth durin all
stages.</td></tr><tr valign=top height="15" style="background-color:#EFF3FB;"><td
width="30" style="border:solid 1px black; border-collapse:collapse ">41</td><td
width="250" style="border:solid 1px black; border-collapse:collapse ">most commonly
used sedative by the pediatric dentists is</td><td width="200" style="border:solid
1px black; border-collapse:collapse ">A: Promethazin<br><br>B:
Hydroxyzine<br><br>C: nitrous oxide
and oxygen<br><br>D: Midazolam<br><br></td><td width="50" style="border:solid 1px
black; border-collapse:collapse ">C</td><td width="250" style="border:solid 1px
black; border-collapse:collapse ">Reference: Mc donalds page 294
explanation:
Eighty-five percent of pediatric dentists use nitrous oxide and oxygen for sedation
of patients. This makes it the most frequently used sedative agent.</td></tr><tr
valign=top height="15" style="background-color:White;"><td width="30"
style="border:solid 1px black; border-collapse:collapse ">42</td><td width="250"
style="border:solid 1px black; border-collapse:collapse ">functional matrix theory
revisited was given by</td><td width="200" style="border:solid 1px black; border-
collapse:collapse ">A: melvin moss<br><br>B: scott<br><br>C: seicher<br><br>D: van-
limborg<br><br></td><td width="50" style="border:solid 1px black; border-
collapse:collapse ">A</td><td width="250" style="border:solid 1px black; border-
collapse:collapse ">ref proffit 4th edition pg no 49:functional matrix theory
revisited was given by melvin moss. This theory was put forth to overcome the draw
backs of the functional matrix in explaining the role of genetics and the role of
functonal demand on the structure which help them to grow.Scott gave nasal septum
theory or the cartilagenous theory, Seicher gave the sutural theory and Van-
Limborgh gave the Epigenetic theory.</td></tr><tr valign=top height="15"
style="background-color:#EFF3FB;"><td width="30" style="border:solid 1px black;
border-collapse:collapse ">43</td><td width="250" style="border:solid 1px black;
border-collapse:collapse ">The primary tooth having the greatest faciolingual
diameter of its crown is the</td><td width="200" style="border:solid 1px black;
border-collapse:collapse ">A: maxillary first molar.<br><br>B: maxillary second
molar.<br><br>C: mandibular first molar.<br><br>D: mandibular second
molar.<br><br></td><td width="50" style="border:solid 1px black; border-
collapse:collapse ">B</td><td width="250" style="border:solid 1px black; border-
collapse:collapse ">Reference: Wheelers, Dental anatomy, pg no: 58
Explanation: primary teeth with labio-lingual diameter at the cervix
Upper Teeth
Central Incisor 4.0
Lateral Incisor3.7
Canine 5.5
First Molar 6.9
Second Molar 8.3
Lower Teeth
Central Incisor 3.5
Lateral Incisor 3.5
Canine 4.0
First Molar 5.3
Second Molar 6.4</td></tr><tr valign=top height="15" style="background-
color:White;"><td width="30" style="border:solid 1px black; border-
collapse:collapse ">44</td><td width="250" style="border:solid 1px black; border-
collapse:collapse ">which of the following statement is true</td><td width="200"
style="border:solid 1px black; border-collapse:collapse ">A: space is easier to
rergain in the maxillary arch<br><br>B: space is easier to regain in the mandibular
arch<br><br>C: both of the above<br><br>D: none of the above<br><br></td><td
width="50" style="border:solid 1px black; border-collapse:collapse ">A</td><td
width="250" style="border:solid 1px black; border-collapse:collapse ">ref proffit
4th edition pg no 476: space is easier to regain in the maxillary than in the
mandibular arch,because of the increased anchorage for removable appliance afforded
by the palatal vault and possibilty for use of extra oral force.</td></tr><tr
valign=top height="15" style="background-color:#EFF3FB;"><td width="30"
style="border:solid 1px black; border-collapse:collapse ">45</td><td width="250"
style="border:solid 1px black; border-collapse:collapse ">deep bite results due
to</td><td width="200" style="border:solid 1px black; border-collapse:collapse
">A: supra eruption of anterior<br><br>B: infra occlusion of molar<br><br>C: both
of the above<br><br>D: none of the above<br><br></td><td width="50"
style="border:solid 1px black; border-collapse:collapse ">C</td><td width="250"
style="border:solid 1px black; border-collapse:collapse ">ref proffit 4th edition
pg no 226 and 117: the cause of dental deep bite is due to supra eruption of
anterior teeth anf infra occlusion of posterior teeth. Skeletal deep bite is due to
rotation of jaws in the anterior direction.</td></tr><tr valign=top height="15"
style="background-color:White;"><td width="30" style="border:solid 1px black;
border-collapse:collapse ">46</td><td width="250" style="border:solid 1px black;
border-collapse:collapse ">if the preliminary cast prepared by using the impression
taken by impression compound is used to fabricate the complete denture, the denture
will be</td><td width="200" style="border:solid 1px black; border-
collapse:collapse ">A: the one with underextended borders<br><br>B: the one with
the overextended borders<br><br>C: the one with perfect fit<br><br>D: perfect fit
but with rough surfaces<br><br></td><td width="50" style="border:solid 1px black;
border-collapse:collapse ">B</td><td width="250" style="border:solid 1px black;
border-collapse:collapse ">reference : Nallaswamy textbook of prosthodontics page
no 70
explanation
when we make a primary impression using a rigid, high fusing material like
impression compound the soft tissues in the palate and in the sulcus usually gets
displaced.hence we get the overextended impression , if we fabricate the denture on
the cast made from such an impression, we may end up fabricating one with over
extended borders.</td></tr><tr valign=top height="15" style="background-
color:#EFF3FB;"><td width="30" style="border:solid 1px black; border-
collapse:collapse ">47</td><td width="250" style="border:solid 1px black; border-
collapse:collapse ">all of the following features of the anterior posterior strap
type major connectors is true except</td><td width="200" style="border:solid 1px
black; border-collapse:collapse ">A: Parallelogram shaped
and open in center portion.<br><br>B: Relatively broad
anterior and posterior palatal straps.Lateral palatal straps narrow<br><br>C:
anterior border of Anterior palatal strap should be placed anteriorly than anterior
rests.<br><br>D: posterior border of the Posterior palatal connector located at
junction of hard and soft palates<br><br></td><td width="50" style="border:solid
1px black; border-collapse:collapse ">C</td><td width="250" style="border:solid
1px black; border-collapse:collapse ">Reference: Mc Crackens RPD page no 63
Explanation:
(1) Parallelogram shaped and open in center portion.
(2) Relatively broad (8 to 10 mm) anterior and posterior palatal straps.
(3) Lateral palatal straps (7 to 9 mm) narrow and parallel to curve of arch;
minimum of 6 mm from gingival crevices of remaining teeth.
(4) Anterior palatal strap: anterior border not placed farther anteriorly than
anterior rests and never closer than 6 mm to lingual gingival crevices; follows the
valleys of the rugae at right angles to the median palatal suture. Posterior
border, if in rugae area, follows valleys of rugae at right angles to the median
palatal suture.
(5) Posterior palatal connector: posterior border located at junction of hard and
soft palates and at right angles to median palatal suture and extended to hamular
notch area(s) on distal extension side(s).
(6) Anatomic replica or matte surface.
</td></tr><tr valign=top height="15" style="background-color:White;"><td width="30"
style="border:solid 1px black; border-collapse:collapse ">48</td><td width="250"
style="border:solid 1px black; border-collapse:collapse ">repositioning a cast on a
surveyor at any time can involve a human error of approximately</td><td
width="200" style="border:solid 1px black; border-collapse:collapse ">A: 2
mm<br><br>B: 0.02 mm<br><br>C: 0.2 mm<br><br>D: 1 mm<br><br></td><td width="50"
style="border:solid 1px black; border-collapse:collapse ">C</td><td width="250"
style="border:solid 1px black; border-collapse:collapse ">Reference: Mc Crackens
RPD page no 178
Explanation:
It must be remembered that repositioning a cast on a surveyor at any time can
involve a certain amount of human error. It has been estimated that an error of 0.
2 mm can be anticipated in reorienting a cast with three reference points on its
base. This reorientation error can influence the placement of appropriate blockout
wax and may result in ineffective placement of direct retainers into prescribed
undercuts and improper contacts of minor connectors to guiding planes. Therefore
reorientation of the cast to surveyor by any method must be done with great
care.</td></tr><tr valign=top height="15" style="background-color:#EFF3FB;"><td
width="30" style="border:solid 1px black; border-collapse:collapse ">49</td><td
width="250" style="border:solid 1px black; border-collapse:collapse ">for the
preparation of tooth to receive the crown/ retainer at no point of time the tooth
should be reduced beyond the central core ,the central core is a cylinder of pulp
with surrounding</td><td width="200" style="border:solid 1px black; border-
collapse:collapse ">A: 2 mm of dentine<br><br>B: 1 mm of dentine<br><br>C: 3 mm on
dentine and 0.5 mm of enamel<br><br>D: 1 mm of enamel<br><br></td><td width="50"
style="border:solid 1px black; border-collapse:collapse ">B</td><td width="250"
style="border:solid 1px black; border-collapse:collapse ">reference: Nallaswamy FPD
page no: 553
Explanation:
• At no point of time the tooth should be reduced beyond the central core (the
central core is a cylinder of pulp with 1mm of surrounding dentin)
• For all restorations except full veneer crowns, all the tooth surfaces should be
uniformly reduced.
• If the caries extends to the central core then it should be scooped out and GIC
restoration should be done in the scooped out area</td></tr><tr valign=top
height="15" style="background-color:White;"><td width="30" style="border:solid 1px
black; border-collapse:collapse ">50</td><td width="250" style="border:solid 1px
black; border-collapse:collapse ">reverse smoking is predominantly found in
:</td><td width="200" style="border:solid 1px black; border-collapse:collapse ">A:
Andhra Pradesh<br><br>B: Goa<br><br>C: Orissa<br><br>D: Assam<br><br></td><td
width="50" style="border:solid 1px black; border-collapse:collapse ">A</td><td
width="250" style="border:solid 1px black; border-collapse:collapse ">"Reference:
Soben Peter,3rd ed , page no 476:
Explanation: reverse smoking is predominantly found in A.P. vishakhapatnam
</td></tr><tr valign=top height="15" style="background-color:#EFF3FB;"><td
width="30" style="border:solid 1px black; border-collapse:collapse ">51</td><td
width="250" style="border:solid 1px black; border-collapse:collapse ">expanded
function dental auxiliary (EFDA) in philadelphia are called</td><td width="200"
style="border:solid 1px black; border-collapse:collapse ">A: dental aide<br><br>B:
technotherapists<br><br>C: denturists<br><br>D: quacks<br><br></td><td width="50"
style="border:solid 1px black; border-collapse:collapse ">B</td><td width="250"
style="border:solid 1px black; border-collapse:collapse ">"Reference: Soben
Peter,4th ed , page no 418:
Explanation:"the first large scale applications of the expanded duty principle were
made in philadelphia , they are called technotherapists
</td></tr><tr valign=top height="15" style="background-color:White;"><td width="30"
style="border:solid 1px black; border-collapse:collapse ">52</td><td width="250"
style="border:solid 1px black; border-collapse:collapse ">All of the following
cranial nerves contain Somatic Efferents, Except:</td><td width="200"
style="border:solid 1px black; border-collapse:collapse ">A: VII Nerve (Facial
nerve)<br><br>B: III Nerve (Occulomotor nerve)<br><br>C: IV Nerve (Trochlear
nerve)<br><br>D: VI Nerve (Abducent nerve)<br><br></td><td width="50"
style="border:solid 1px black; border-collapse:collapse ">A</td><td width="250"
style="border:solid 1px black; border-collapse:collapse ">General Somatic Efferents
• Cranial Nerves that supply skeletal muscles of somatic origin contain somatic
efferents
• These cranial nerves contain Somatic Efferent nuclei that supply these muscles.
• Skeletal muscles of somatic origin include the tongue muscles and the muscles
that move the eyeball (obliques & Recti)
Cranial Nerves Containing Somatic Efferents: XII, III, IV and VI
Bronchial Efferents or / Special Visceral Efferents (SVE): supply the muscles
derrived from branchial arches
V (Masticatory nucleus)
VII (Facial nucleus)
IX, X and XI (Ambiguous nucleus)
XI (Spinal acessary nucleus)

General visceral Efferents (GVEor VE) viscera that receive parasympathetic supply
III (Edinger westphial nucleus
VII (Superior salivatory nucleus)
IX (Inferior salivatory nucleus)
X (Dorsal motor nucleus)</td></tr><tr valign=top height="15" style="background-
color:#EFF3FB;"><td width="30" style="border:solid 1px black; border-
collapse:collapse ">53</td><td width="250" style="border:solid 1px black; border-
collapse:collapse ">Which of this part of vertebral canal will show secondary
curves with concavity backwards?</td><td width="200" style="border:solid 1px
black; border-collapse:collapse ">A: Cervical<br><br>B: Thoracic<br><br>C:
Sacral<br><br>D: Coccyx<br><br></td><td width="50" style="border:solid 1px black;
border-collapse:collapse ">A</td><td width="250" style="border:solid 1px black;
border-collapse:collapse ">Primary Curves: Embryonic and Fetal Curvatures
• Embryonic body appears flexed
• Curves are convex dorsally (concave anteriorly)
• Primarily in the thoracic and pelvic region.

Secondary Curves: Adult curvatures


• Cervical spine ( Lordosis) Convex anteriorly, Concave posteriorly
• Thoracic curve (Kyphosis) Convex posteriorly, Concave anteriorly
• Lumbar curve (Lordosis) Convex anteriorly,Concave posteriorly
• Pelvic curve (Kyphosis) Convex posteriorly, (sacrococcygeal) Concave
anteriorly</td></tr><tr valign=top height="15" style="background-color:White;"><td
width="30" style="border:solid 1px black; border-collapse:collapse ">54</td><td
width="250" style="border:solid 1px black; border-collapse:collapse ">Each of the
following is an effect of parathyroid hormone EXCEPT one. Which one is this
EXCEPTION?</td><td width="200" style="border:solid 1px black; border-
collapse:collapse ">A: A Stimulation of 1-alpha-hydroxylase in kidney<br><br>B:
Stimulation of osteoclastic activity in bone<br><br>C: Stimulation of calcium
reabsorption by kidney<br><br>D: Inhibition of intestinal absorption of
calcium<br><br></td><td width="50" style="border:solid 1px black; border-
collapse:collapse ">D</td><td width="250" style="border:solid 1px black; border-
collapse:collapse ">Reference:Guyton, Physiology, 11th ed, pg no:343
Explanation:
Parathyroid Hormone Increases Calcium Reabsorption.
Parathyroid hormone is one of the most important calcium-regulating hormones in the
body. Its principal action in the kidneys is to increase tubular reabsorption of
calcium, especially in the distal tubules and perhaps also in the loops of Henle.
Parathyroid hormone also has other actions, including inhibition of phosphate
reabsorption by the proximal tubule and stimulation of magnesium reabsorption by
the loop of Henle.
the conversion of 25- hydroxycholecalciferol to 1,25-dihydroxycholecalciferol
requires PTH.As PTH stimulates 1 alpha hydroxylase in kidney for the conversion to
take place. In the absence of PTH, almost none of the 1,25-dihydroxycholecalciferol
is formed. Therefore, PTH exerts a potent influence in determiningthe functional
effects of vitamin D in the body.
PTH also stimulates osteoclastic activity and increases intestinal absorption of
calcium to increase calcium level in the body</td></tr><tr valign=top height="15"
style="background-color:#EFF3FB;"><td width="30" style="border:solid 1px black;
border-collapse:collapse ">55</td><td width="250" style="border:solid 1px black;
border-collapse:collapse ">The human plasma lipoprotein containing the highest
percentage of triacylglycerol by weight is:</td><td width="200"
style="border:solid 1px black; border-collapse:collapse ">A: VLDL<br><br>B:
Chylomicrons<br><br>C: HDL<br><br>D: LDL<br><br></td><td width="50"
style="border:solid 1px black; border-collapse:collapse ">B</td><td width="250"
style="border:solid 1px black; border-collapse:collapse ">Lipoproteins may be
differentiated on the basis of their lipid & protein compositions. The lipoproteins
containing the largest percentage of triacylglycerol are Chylomicrons</td></tr><tr
valign=top height="15" style="background-color:White;"><td width="30"
style="border:solid 1px black; border-collapse:collapse ">56</td><td width="250"
style="border:solid 1px black; border-collapse:collapse ">hemostasis
involves</td><td width="200" style="border:solid 1px black; border-
collapse:collapse ">A: vasoconstriction<br><br>B: formation of platelet
plug<br><br>C: coagulation of blood<br><br>D: all of the above<br><br></td><td
width="50" style="border:solid 1px black; border-collapse:collapse ">D</td><td
width="250" style="border:solid 1px black; border-collapse:collapse ">Ref:
Sembulingam, 2nd ed, pg no: 82
Exp: HEMOSTASIS occurs in 3 stages:
a. Vasoconstriction
b. coagulation of the blood
c. formation of platelet plug</td></tr><tr valign=top height="15"
style="background-color:#EFF3FB;"><td width="30" style="border:solid 1px black;
border-collapse:collapse ">57</td><td width="250" style="border:solid 1px black;
border-collapse:collapse ">A patient with Renal cell carcinoma is on treatment with
Erythropoetin. Erythropoetin use may be associated with all of the following
Except:</td><td width="200" style="border:solid 1px black; border-
collapse:collapse ">A: Decreased reticulocytes<br><br>B: Increased
hemtocrit<br><br>C: Increased hemoglobin<br><br>D: Eliminates need for
transfusion<br><br></td><td width="50" style="border:solid 1px black; border-
collapse:collapse ">A</td><td width="250" style="border:solid 1px black; border-
collapse:collapse ">Treatment with erythropoetin results in an increase in the
Reticulocyte count (reticulocytes are not decreased).

Erythropoetin acts on Erythroid precursors in the bone marrow and stimulates the
rate of red cell production
This results in:
Increased Reticulocyte count
Increased Haematocrit
Increased oxygen carrying capacity of blood (Increased Haemoglobin)</td></tr><tr
valign=top height="15" style="background-color:White;"><td width="30"
style="border:solid 1px black; border-collapse:collapse ">58</td><td width="250"
style="border:solid 1px black; border-collapse:collapse ">A 30 year old female
patient presents with non progressive dysphagia, for both solids and liquids. The
characteristic finding on baruim swallow that will confirm the probable diagnosis
is:</td><td width="200" style="border:solid 1px black; border-collapse:collapse
">A: Dilated esophagus with narrow lower end (Rat tail esophagus)<br><br>B:
Multiple sacculations and pseudodiverticulae (Corkscrew esophagus)<br><br>C: Narrow
and irregular esophageal lumen<br><br>D: Stricture ulcer in the
esophagus<br><br></td><td width="50" style="border:solid 1px black; border-
collapse:collapse ">B</td><td width="250" style="border:solid 1px black; border-
collapse:collapse ">
• The presence of non progressive dysphagia to both solids and liquids suggests the
diagnosis of diffuse esophageal
spasm. The charachteristic barium swallow findings in diffuse esophageal spasm is
the corkscrew esophagus
• Rat tail esophagus with a dilated proximal esophagus and narrow lower end
suggests a diagnosis of Achlasia
cardia. Achlasia is associated with Progressive dysphagia to both solids and
liquids</td></tr><tr valign=top height="15" style="background-color:#EFF3FB;"><td
width="30" style="border:solid 1px black; border-collapse:collapse ">59</td><td
width="250" style="border:solid 1px black; border-collapse:collapse ">A 28 year old
patient has a FPD in 11, 12, 21 and 22 region following severe trauma. He is quite
happy with the prosthesis except for the black triangles which are present in the
area of gingival embrasures and is of esthetic concern to the patient? How can you
modify the prosthesis to enhance the patients esthetics?</td><td width="200"
style="border:solid 1px black; border-collapse:collapse ">A: use pink porcelain on
the old FPD, hence covering the gingival embrasures this enhancing the
look.<br><br>B: fabricate a new prosthesis<br><br>C: Andrews bridge<br><br>D:
Either A or C<br><br></td><td width="50" style="border:solid 1px black; border-
collapse:collapse ">D</td><td width="250" style="border:solid 1px black; border-
collapse:collapse ">Reference: Nallaswamy, Textbook of prosthodontics, pg no: 515
Explanation:pontic modifications:
if edentulous space is resorbed, large gingival embrasures (black triangles appear)
these can be treated by:
a.pink porcelain
b. or by fabricating Andrews bridge
Andrews bridge: Fixed removable modified fpd. It has afixed bar connected to the
lingual surface of the retainer whereas the removale prosthesis is attached onto
the bar.
</td></tr><tr valign=top height="15" style="background-color:White;"><td width="30"
style="border:solid 1px black; border-collapse:collapse ">60</td><td width="250"
style="border:solid 1px black; border-collapse:collapse ">Degeneration of the
basement membrane is mediated by:</td><td width="200" style="border:solid 1px
black; border-collapse:collapse ">A: Oxidases<br><br>B: Elastases<br><br>C:
Hydroxylases<br><br>D: Metallo proteinase<br><br></td><td width="50"
style="border:solid 1px black; border-collapse:collapse ">D</td><td width="250"
style="border:solid 1px black; border-collapse:collapse ">Basement membrane is a
component of ECM and largely consists of collagen (Type IV).
Matrix Mettaloproteins MMP9 and MMP2 are collagenases that cleave type IV collagen
of epithelial and vascular
basement membranes

Matrix metalloproteinases (MMP) is a family of enzymes which includes more than 20


members
These include enzyme that are involved in degradationof Basement Mebrane such as:
• MMP 2 & 9, (Gelatinases/Collagenases) which degrade amorphous collagen and
fibronectin (Basement Membrane)
• MMP 3, 10 & 11 (Stromelysins) which degrade proteoglycans, laminin, fibronetin
and amorphous collagens
(Basement membrane components)</td></tr><tr valign=top height="15"
style="background-color:#EFF3FB;"><td width="30" style="border:solid 1px black;
border-collapse:collapse ">61</td><td width="250" style="border:solid 1px black;
border-collapse:collapse ">Thoracic outlet syndrome is primarily diagnosed
by:</td><td width="200" style="border:solid 1px black; border-collapse:collapse
">A: Clinical Evaluation<br><br>B: CTscan<br><br>C: MRl<br><br>D:
Angiography<br><br></td><td width="50" style="border:solid 1px black; border-
collapse:collapse ">A</td><td width="250" style="border:solid 1px black; border-
collapse:collapse ">Explanation:
ThoracicOutlet Compression Syndrome (TOS)
►TDS is a symptom complex resulting from compression of the neurovascular bundle
(artery, vein or nerve) at the thoracic outlet
►Depending on the structures affected, TOS may be divided into Vascular forms
(Arterial and lor Venous) and Neurogenic forms
►Compression resulting from TOS is dynamic and hence is best evaluated clinically
by mechanical provocative maneuvers

ProvocativeTests:
1) (ScaleneTest)Adson test: Patient is instructed to: take breath and hold it,
extend the neck fully, turn the face towards the side---Obliteration or the
diminution of the radila pulse suggest the diagnosis.
2) MilitaryPosition)CostoclavicularTest: pateint instructed to Draw shoulders
downwards---Obliteration of radial pulse or reproduction of symptoms indicates
compression

3) Hyperabduction Test: Hyperabduct (Raise) the arm to 180 degrees---Obliteration


or diminution of Radial pulse suggests the diagnosis

4) RoosTest (Arm claudication test) Draw shoulders backwards,Rise arms to


horizontal position with elbows flexed to 90 degrees, Exercise the hands---Numbness
or pain in the hands with exercise suggests the diagnosis</td></tr><tr valign=top
height="15" style="background-color:White;"><td width="30" style="border:solid 1px
black; border-collapse:collapse ">62</td><td width="250" style="border:solid 1px
black; border-collapse:collapse ">Based on Epidemiological studies, which of the
following has been found to be most protective against Carcinoma Colon</td><td
width="200" style="border:solid 1px black; border-collapse:collapse ">A: High fiber
diet<br><br>B: Low fat diet<br><br>C: Low selenium diet<br><br>D: Low protein
diet<br><br></td><td width="50" style="border:solid 1px black; border-
collapse:collapse ">A</td><td width="250" style="border:solid 1px black; border-
collapse:collapse ">1) Epidemiologic,case control and animal studies suggest that
dietary fibers are protective against the development of coloncancer
2) Fat is primarily related as a promoting agent such that high fat in diet is
believed to promote carcinogenesis. Low Fat has not been shown to have a protective
role in carcinogenesis although it may reduce the incidence of
coloniccarcinoma indirectly by reducing a promoting effect .
3) selenium is protective , so low selenium diet has promotive effect.</td></tr><tr
valign=top height="15" style="background-color:#EFF3FB;"><td width="30"
style="border:solid 1px black; border-collapse:collapse ">63</td><td width="250"
style="border:solid 1px black; border-collapse:collapse ">The prognosis of
rhabdomyosarcoma is likely to be poor if the site of the tumour is:</td><td
width="200" style="border:solid 1px black; border-collapse:collapse ">A:
Orbit<br><br>B: Para testicular<br><br>C: Extremity<br><br>D: Urinary
bladder<br><br></td><td width="50" style="border:solid 1px black; border-
collapse:collapse ">C</td><td width="250" style="border:solid 1px black; border-
collapse:collapse ">RMS arising in the extremities is more likely to be identified
in the second decade of life. Almost one half of these extremity primaries contain
alveolar elements that are associated with a more aggressive tumor biology
Primary tumor sites with better prognosis:
• Orbit
• Genitourinary (paratesticular)
• Prostate</td></tr><tr valign=top height="15" style="background-color:White;"><td
width="30" style="border:solid 1px black; border-collapse:collapse ">64</td><td
width="250" style="border:solid 1px black; border-collapse:collapse ">Most common
mode of transmission of Pasturella Multocida is:</td><td width="200"
style="border:solid 1px black; border-collapse:collapse ">A: Animal Bites or
scratches<br><br>B: Aerosols or dust<br><br>C: Contaminated tissue<br><br>D: Human
to human<br><br></td><td width="50" style="border:solid 1px black; border-
collapse:collapse ">A</td><td width="250" style="border:solid 1px black; border-
collapse:collapse ">Reference:Ananthnarayanan 7th edition page no :331
Explanation:
Modes of infection of P. Multocida:
►Direct Inoculation Through Skin (most common)
►Through the Respiratory Tract (second most common)</td></tr><tr valign=top
height="15" style="background-color:#EFF3FB;"><td width="30" style="border:solid
1px black; border-collapse:collapse ">65</td><td width="250" style="border:solid
1px black; border-collapse:collapse ">Which of the following statements about
Prions is true:</td><td width="200" style="border:solid 1px black; border-
collapse:collapse ">A: They are infectious proteins<br><br>B: They are made up of
bacteria and virus<br><br>C: They have rich nuclear material<br><br>D: They can be
cultured in cell free media<br><br></td><td width="50" style="border:solid 1px
black; border-collapse:collapse ">A</td><td width="250" style="border:solid 1px
black; border-collapse:collapse ">Prions are infectious protein particals, causing
diseases such as japanese encephalitis , kuru
they are most resistant to disinfectants.</td></tr><tr valign=top height="15"
style="background-color:White;"><td width="30" style="border:solid 1px black;
border-collapse:collapse ">66</td><td width="250" style="border:solid 1px black;
border-collapse:collapse ">Which of the following statements is NOT correct
regarding, resistance to Methicillin in MRSA</td><td width="200"
style="border:solid 1px black; border-collapse:collapse ">A: Resistance is produced
as a result of altered PBPs<br><br>B: Resistance may be produced by hyperproduction
of Beta Lactamase<br><br>C: Resistance is primarily mediated Itransmitted via
plasmids<br><br>D: Resistance may be missed at incubation temperature of 37°C
during susceptibility testing<br><br></td><td width="50" style="border:solid 1px
black; border-collapse:collapse ">C</td><td width="250" style="border:solid 1px
black; border-collapse:collapse ">Explanation:
1) Resistance is not primarily mediated via plasmids. Resistance is primarily
mediated via chromosomal MeC-A gene which codes for altered PBP - 2A.
2) Borderline resistanceto methicillin may be produced as a result of hyper
production of Beta Lactamases, but this is not the primary mechanism.
3) Low resistance to MRSA (heteroresistance) may be missed at 37 degrees C and
temperature of incubation )no higher than 35 degrees C is recommended. Expression
of Resistance is enhanced by lowering the temperature of incubation to less than
35° (rather than 37° whichis used routinely)</td></tr><tr valign=top height="15"
style="background-color:#EFF3FB;"><td width="30" style="border:solid 1px black;
border-collapse:collapse ">67</td><td width="250" style="border:solid 1px black;
border-collapse:collapse ">Ethosuximide is used in the treatment of:</td><td
width="200" style="border:solid 1px black; border-collapse:collapse ">A: Tonic-
clonic seizure<br><br>B: Absence seizure<br><br>C: Myoclonic seizure<br><br>D:
Simple partial seizure<br><br></td><td width="50" style="border:solid 1px black;
border-collapse:collapse ">B</td><td width="250" style="border:solid 1px black;
border-collapse:collapse ">Reference: KD tripathi page no:374, 5th edition
explanation:
1) Primary Generalized tonic clonic: first line- Valproic acid, Lamotrigine
2) Partial: first line-Carbamazepine, Phenytoin, Lamotrigene, Val roic acid
3) Absence: first line- Valproic acid, Ethosuximide
4) Atypical Absence, Myoclonic Atonic: first line- Valproic acid.</td></tr><tr
valign=top height="15" style="background-color:White;"><td width="30"
style="border:solid 1px black; border-collapse:collapse ">68</td><td width="250"
style="border:solid 1px black; border-collapse:collapse ">The blood pressure
measured by a sphygmomanometer:</td><td width="200" style="border:solid 1px black;
border-collapse:collapse ">A: Is lower than the intraarterial pressure<br><br>B: Is
higher than the intraarterial pressure<br><br>C: Is same as the intraarterial
pressure<br><br>D: Is the same with different cuff sizes<br><br></td><td
width="50" style="border:solid 1px black; border-collapse:collapse ">B</td><td
width="250" style="border:solid 1px black; border-collapse:collapse ">• The
sphygmomanometer will give accurate readings only if no pressure (zero mm. of
mercury) is dissipated in
compressing the soft tissues overlying the artery. However in practice, some
pressure is always dissipated in compressing the blanket of soft tissues overlying
the artery and hence blood pressure measured by the sphygmomanometer are slightly
higher than true intra arterial pressures especially in elderly and obese
• The amount by which the sphygmomanometer overestimates the true blood pressure is
influenced by the amount of
overlying fat and the compressibility of the vessles.

OTHER FACTS:
• Small (Narrow) cuff leads to falsely high blood pressure recordings
(overestimation)
• Large (wide) cuff leads to falsely lower blood pressure recordings (under
estimation)

Note
• More accurate pressure can be recorded in obese individuals by using a wider cuff
size
• More accurate pressure can be recorded in the thigh by using a wider cuff
size</td></tr><tr valign=top height="15" style="background-color:#EFF3FB;"><td
width="30" style="border:solid 1px black; border-collapse:collapse ">69</td><td
width="250" style="border:solid 1px black; border-collapse:collapse ">Which of the
following hormone is not secret ed by the kidney:</td><td width="200"
style="border:solid 1px black; border-collapse:collapse ">A: Renin<br><br>B:
Angiotensin I<br><br>C: Erythropoietin<br><br>D: 1,25DHCC<br><br></td><td
width="50" style="border:solid 1px black; border-collapse:collapse ">B</td><td
width="250" style="border:solid 1px black; border-collapse:collapse ">The kidneys
produce three hormones, 1, 25 Dihydrochole calciferol, Renin and Erythropoetin.
Angiotensin I is not secreted by any organ. It is formed by the action of Renin
(secreted by kidney) on Angiotensinogen (secreted by Liver).</td></tr><tr
valign=top height="15" style="background-color:White;"><td width="30"
style="border:solid 1px black; border-collapse:collapse ">70</td><td width="250"
style="border:solid 1px black; border-collapse:collapse ">The latest WHO protocol
for classification and measurement of disability is:</td><td width="200"
style="border:solid 1px black; border-collapse:collapse ">A: ICIDH<br><br>B:
ICF<br><br>C: WHODAS<br><br>D: DALY<br><br></td><td width="50" style="border:solid
1px black; border-collapse:collapse ">B</td><td width="250" style="border:solid
1px black; border-collapse:collapse ">The latest WHO initiative for classification
of disability is the International Classification of Functioning, Disability and
Health which is termed as ICF</td></tr><tr valign=top height="15"
style="background-color:#EFF3FB;"><td width="30" style="border:solid 1px black;
border-collapse:collapse ">71</td><td width="250" style="border:solid 1px black;
border-collapse:collapse ">Psychrometer is used to measure:</td><td width="200"
style="border:solid 1px black; border-collapse:collapse ">A: Humidity<br><br>B: Air
velocity<br><br>C: Room temperature<br><br>D: Radiant temperature<br><br></td><td
width="50" style="border:solid 1px black; border-collapse:collapse ">A</td><td
width="250" style="border:solid 1px black; border-collapse:collapse ">Psychrometer
is a device to measure humidity.
Psychrometer (Synonym: Wet and Dry bulb thermometer / Hygrometer)</td></tr><tr
valign=top height="15" style="background-color:White;"><td width="30"
style="border:solid 1px black; border-collapse:collapse ">72</td><td width="250"
style="border:solid 1px black; border-collapse:collapse ">Which of the following is
the most radiosensitive phase of the cell cycle?</td><td width="200"
style="border:solid 1px black; border-collapse:collapse ">A: G2M<br><br>B:
G2<br><br>C: S<br><br>D: GI<br><br></td><td width="50" style="border:solid 1px
black; border-collapse:collapse ">A</td><td width="250" style="border:solid 1px
black; border-collapse:collapse ">Most sensitive phase is the junction of G2M phase
Susceptibility orvarious phases orcell cycle to radiation: G2M> G2 >= M > G1 >
Early S > Late S
Facts to remember:
• Dividing part of cells are most sensitive to RT
• Non dividing cells are relatively resistant
• Hypoxic cells are relatively resistant
• Phase of cell cycle that is most sensitive to radiation: G2M > M
• Phase of cell cycle that is most resistant to radiation: End of Sphase
• Phase of cell cyle in which radiation exposure leads to chromosomal aberration:
G1
• Phase of cell cycle in which radiation exposure leads to chromatid aberration:
G2</td></tr><tr valign=top height="15" style="background-color:#EFF3FB;"><td
width="30" style="border:solid 1px black; border-collapse:collapse ">73</td><td
width="250" style="border:solid 1px black; border-collapse:collapse ">The cell
bodies of proprioceptive nerves carrying information from the periodontal ligaments
are located in the</td><td width="200" style="border:solid 1px black; border-
collapse:collapse ">A: nucleus ambiguus.<br><br>B: spinal nucleus of V.<br><br>C:
main sensory nucleus of V.<br><br>D: mesencephalic nucleus of V.<br><br></td><td
width="50" style="border:solid 1px black; border-collapse:collapse ">D</td><td
width="250" style="border:solid 1px black; border-collapse:collapse ">Various
structures and functions of cranial nerve (CN) V (trigeminal) include the
trigeminal ganglion, near the cavernous sinus, which sends out the maxillary,
mandibular, and ophthalmic nerves. The mesencephalic nucleus receives information
about proprioception from the muscles of mastication, TMJ joint, and periodontal
ligament. The main sensory nucleus of CN V (choice C) lies near the motor nucleus
and receives tactile sensation. The spinal nucleus of CN V (choice B) receives pain
and thermal sensation. Nucleus ambiguus (choice A) is located in the pons
and sends fibers to CN IX, X, and XI.</td></tr><tr valign=top height="15"
style="background-color:White;"><td width="30" style="border:solid 1px black;
border-collapse:collapse ">74</td><td width="250" style="border:solid 1px black;
border-collapse:collapse ">In the posterior mediastinum, the thoracic duct usually
lies</td><td width="200" style="border:solid 1px black; border-collapse:collapse
">A: anterior to the phrenic nerves.<br><br>B: posterior to the
esophagus.<br><br>C: on the anterolateral surface of trachea.<br><br>D: does not
reach<br><br></td><td width="50" style="border:solid 1px black; border-
collapse:collapse ">B</td><td width="250" style="border:solid 1px black; border-
collapse:collapse ">The thoracic duct runs from the cisterna chyli (between the
aorta and azygos vein on the bodies of the first two lumbar vertebrae) through the
aortic opening of the diaphragm to the posterior mediastinum, along the vertebral
column, behind the esophagus, and through the superior mediastinum. It arches over
the apex of the pleura, descends across the left subclavian artery, and enters the
junction of the left internal jugular and subclavian veins. It is found posterior
and medial to the phrenic nerve and posterior to the trachea.</td></tr><tr
valign=top height="15" style="background-color:#EFF3FB;"><td width="30"
style="border:solid 1px black; border-collapse:collapse ">75</td><td width="250"
style="border:solid 1px black; border-collapse:collapse ">If an axonal membrane
transiently becomes very permeable to Na+ ions, then the membrane potential of the
cell will approach</td><td width="200" style="border:solid 1px black; border-
collapse:collapse ">A: -70 mV.<br><br>B: -60 mV.<br><br>C: -50 mV.<br><br>D: +60
mV.<br><br></td><td width="50" style="border:solid 1px black; border-
collapse:collapse ">D</td><td width="250" style="border:solid 1px black; border-
collapse:collapse ">► The diffusion of an ion through a membrane, without the
concomitant diffusion of its counter-ion, creates a membrane potential. Na+ ions
are present in higher concentration in the extracellular fluid than in axoplasm. If
the axonal membrane becomes permeable to Na+, the inward diffusion of this positive
ion, without similar diffusion of its negative counter-ion, will make the inside of
the neuron electrically positive in relation to the extracellular fluid. Thus, the
membrane potential will have a positive value.
► It is just such a transient increase in membrane permeability to Na+ that is the
mechanism responsible for the neuronal action potential. During the impulse, the
membrane voltage goes from a resting potential of about –75 mV to a peak of about
+30
mV, approaching the equilibrium potential for sodium of about +60 mV.</td></tr><tr
valign=top height="15" style="background-color:White;"><td width="30"
style="border:solid 1px black; border-collapse:collapse ">76</td><td width="250"
style="border:solid 1px black; border-collapse:collapse ">The most potent propionic
acid derivative is</td><td width="200" style="border:solid 1px black; border-
collapse:collapse ">A: ibuprofen<br><br>B: naproxen<br><br>C:
flurbiprofen<br><br>D: ketoprofen<br><br></td><td width="50" style="border:solid
1px black; border-collapse:collapse ">B</td><td width="250" style="border:solid
1px black; border-collapse:collapse ">Reference: Tripathi, Essentials of
Pharmacology,6th edition, pg no: 193
Explanation: Naproxen is the most potent propionic acid derivative.
Ibuprofen is the most used over the counter drugs.It also has a high theraputic
index.</td></tr><tr valign=top height="15" style="background-color:#EFF3FB;"><td
width="30" style="border:solid 1px black; border-collapse:collapse ">77</td><td
width="250" style="border:solid 1px black; border-collapse:collapse ">Most
endogenous cholesterol in the liver is usually converted into which of the
following?</td><td width="200" style="border:solid 1px black; border-
collapse:collapse ">A: Glucose<br><br>B: Steroids<br><br>C: Cholic acid<br><br>D:
Oxaloacetate<br><br></td><td width="50" style="border:solid 1px black; border-
collapse:collapse ">C</td><td width="250" style="border:solid 1px black; border-
collapse:collapse ">►Cholic acid, one of the bile acids, is synthesized by the
liver from cholesterol. In the conversion to cholic acid, hydroxyl groups and a
carboxyl group are added to the steroid nucleus of cholesterol. Bile acids are
important in the intestinal absorption of lipids.
►Neither glucose (choice A), oxaloacetate (choice D), nor ketone bodies (choice E)
are
derived from cholesterol. Steroids (choice B) are synthesized from cholesterol but
are not
produced in the liver. They are produced in the gonads and adrenal
glands.</td></tr><tr valign=top height="15" style="background-color:White;"><td
width="30" style="border:solid 1px black; border-collapse:collapse ">78</td><td
width="250" style="border:solid 1px black; border-collapse:collapse ">Which of the
following substances is LEAST polar?</td><td width="200" style="border:solid 1px
black; border-collapse:collapse ">A: Ethanol<br><br>B: Cholesterol<br><br>C:
Palmitic acid<br><br>D: Glycocholic acid<br><br></td><td width="50"
style="border:solid 1px black; border-collapse:collapse ">B</td><td width="250"
style="border:solid 1px black; border-collapse:collapse ">► Cholesterol is
considered a nonpolar and very hydrophobic lipid.The other three choices are less
hydrophobic and more polar than cholesterol, and are therefore incorrect.
► Ethanol (choice A) is a simple alcohol molecule and the most polar of the group.
Palmitic acid (choice C) is a saturated, straight-chain fatty acid that is nonpolar
but less so than cholesterol. Glycocholic acid (choice D) is a conjugated bile
acid.</td></tr><tr valign=top height="15" style="background-color:#EFF3FB;"><td
width="30" style="border:solid 1px black; border-collapse:collapse ">79</td><td
width="250" style="border:solid 1px black; border-collapse:collapse ">Syneresis and
imbibition are inherent properties that affect the dimensions of which of the
following impression materials (a) Silicones (b) Polysulfide rubbers (c) Reversible
hydrocolloids (d) Irreversible hydrocolloids (e) Impression modeling
compounds</td><td width="200" style="border:solid 1px black; border-
collapse:collapse ">A: (a) and (b) only<br><br>B: (a). (b) and (e)<br><br>C: (c)
and (d)<br><br>D: (c) and (e)<br><br></td><td width="50" style="border:solid 1px
black; border-collapse:collapse ">C</td><td width="250" style="border:solid 1px
black; border-collapse:collapse ">Ref: Craig, Ed: 11th,Pg no: 344, phillips, 10th
ed, pg no 161 and 147, Expl: As with alginate impressions, agar hydrocolloid
impressions are best stored in 100% relative humidity if the gypsum models cannot
be prepared immediately. Even in 100% humidity they can be stored for only limited
times, such as 1 hour, without shrinkage of the impression material caused by
syneresis. The best procedure is to pour up the impression immediately after
removing, rinsing, disinfecting, and superficial drying.Synerisis and imbibition
deos not occur in Impression compound. But rubber materials like polysulphide
aremuch more stable dimentionally when they are stored in air than the hydrocooliod
imp. materials. Addition silicones are the most dimentional stable imp. material of
all existing imp.materials.</td></tr><tr valign=top height="15" style="background-
color:White;"><td width="30" style="border:solid 1px black; border-
collapse:collapse ">80</td><td width="250" style="border:solid 1px black; border-
collapse:collapse ">In cobalt-chromium alloys, the constituent responsible for
corrosion resistance is</td><td width="200" style="border:solid 1px black; border-
collapse:collapse ">A: silver<br><br>B: nickel.<br><br>C: chromium.<br><br>D:
cobalt.<br><br></td><td width="50" style="border:solid 1px black; border-
collapse:collapse ">C</td><td width="250" style="border:solid 1px black; border-
collapse:collapse ">Ref: Craig, Ed: 11th, Pg no: 481, Expl: Chromium is
responsible for the tarnish and corrosion resistance of these alloys. When the
chromium content of an alloy is higher than 30%, the alloy is more difficult to
cast. With this percentage of chromium, the alloy also forms a brittle phase, known
as the sigma (0) phase. Therefore cast base-metal dental alloys should not contain
more than 28% or 29% chromium. In general, cobalt and nickel, up to a certain
percentage, are interchangeable elements. Cobalt increases the elastic modulus,
strength, and hardness of the alloy more than does nickel.</td></tr><tr valign=top
height="15" style="background-color:#EFF3FB;"><td width="30" style="border:solid
1px black; border-collapse:collapse ">81</td><td width="250" style="border:solid
1px black; border-collapse:collapse ">Viruses may cause disease by</td><td
width="200" style="border:solid 1px black; border-collapse:collapse ">A: lysing
many cells of the host<br><br>B: transforming cells to malignant cells<br><br>C:
disrupting the normal defence mechanism<br><br>D: all of the above<br><br></td><td
width="50" style="border:solid 1px black; border-collapse:collapse ">D</td><td
width="250" style="border:solid 1px black; border-collapse:collapse ">Ref:
ananthanarayan, Ed: 8th ,Pg no:444, Expl: At the cellular level, the virus causes
cytocidal effect or cytolytisisEx: poliovirus. Other may produce cellular
proliferation or malignancy Ex: oncogenic viruses. In some instances it causes
cytopathic effects.</td></tr><tr valign=top height="15" style="background-
color:White;"><td width="30" style="border:solid 1px black; border-
collapse:collapse ">82</td><td width="250" style="border:solid 1px black; border-
collapse:collapse ">An antibacterial substance found in saliva, tears, and eggs
white is</td><td width="200" style="border:solid 1px black; border-
collapse:collapse ">A: albumin<br><br>B: isozyme<br><br>C: amylase<br><br>D:
lysozyme<br><br></td><td width="50" style="border:solid 1px black; border-
collapse:collapse ">D</td><td width="250" style="border:solid 1px black; border-
collapse:collapse ">Ref: Guyton, Ed: 11th, Pg no: 794, Expl: Saliva contains
several factors that destroy bacteria. One of these is thiocyanate ions and another
is several proteolytic enzymes—most important, lysozyme—that (a) attack the
bacteria, (b) aid the thiocyanate ions in entering the bacteria where these ions in
turn become bactericidal, and (c) digest food particles, thus helping further to
remove the bacterial metabolic support. Saliva contains two major types of protein
secretion: (1) a serous secretion that contains ptyalin (ana-amylase), which is an
enzyme for digesting starches, and (2) mucus secretion that contains mucin for
lubricating and for surface protective purposes.
</td></tr><tr valign=top height="15" style="background-color:#EFF3FB;"><td
width="30" style="border:solid 1px black; border-collapse:collapse ">83</td><td
width="250" style="border:solid 1px black; border-collapse:collapse ">If the pH of
an area is lower than normal body pH, the membrane theory of local anesthetic
action predicts that the local anesthetic activity would be</td><td width="200"
style="border:solid 1px black; border-collapse:collapse ">A: greater, owing to an
incre.1se in the free-base form of the drug.<br><br>B: greater, owing to an
increase in the cationic form of the drug.<br><br>C: less, owing to an increase in
the free-base form of the drug .<br><br>D: less, owing to a decrease in the free-
base form of the drug.<br><br></td><td width="50" style="border:solid 1px black;
border-collapse:collapse ">D</td><td width="250" style="border:solid 1px black;
border-collapse:collapse ">Ref: Monheims, Ed: 7th, Pg no: 131, Expl: The local
aneasthetic solution is water soluble. The salts of local aneasthetic compounds
exist as both uncharged molecules , also called free base, and positiively charged
molecules, called the cation, in equlibrium with each other. This relative
proportion between the free base and the cation depends on the pH of the solution
and the pKa (dissociation constant). Since pKa is constant for any specific
compound, the relative proportion of free base and the cation depends on the pH of
the solution.If the pH is low, more cation will be present than free base( fat
soluble) form. The free base forms is responsible for the optimal diffusion through
the nerve sheath.Thus deprotonation is prevented in acidic pH and the LA solution
fails to penetrate the nerve.</td></tr><tr valign=top height="15"
style="background-color:White;"><td width="30" style="border:solid 1px black;
border-collapse:collapse ">84</td><td width="250" style="border:solid 1px black;
border-collapse:collapse ">The most controllable route for administration of a
general anesthetic is</td><td width="200" style="border:solid 1px black; border-
collapse:collapse ">A: rectal.<br><br>B: inhalation.<br><br>C:
intramuscular.<br><br>D: intravenous.<br><br></td><td width="50"
style="border:solid 1px black; border-collapse:collapse ">B</td><td width="250"
style="border:solid 1px black; border-collapse:collapse ">Ref: Tripathi, Ed: 5th,
Pg no: 9, Expl: Inhalational GA is absorbed from the alveoli and the action is
very rapid. When discontinued the drug diffues back and is rapidly eliminated in
expired air. Thus, controlled administration is possible with moment to moment
adjustment.</td></tr><tr valign=top height="15" style="background-
color:#EFF3FB;"><td width="30" style="border:solid 1px black; border-
collapse:collapse ">85</td><td width="250" style="border:solid 1px black; border-
collapse:collapse ">The fluid that leaks out of vessels in noninflammatory
conditions, such as cardiac failure, is</td><td width="200" style="border:solid
1px black; border-collapse:collapse ">A: exudate<br><br>B: ecchymoses<br><br>C:
metachysis<br><br>D: transudate<br><br></td><td width="50" style="border:solid 1px
black; border-collapse:collapse ">D</td><td width="250" style="border:solid 1px
black; border-collapse:collapse ">Ref: Harsh Mohan, Ed: 3rd, Pg no: 189, 201. Expl:
Transudate is the filtrateof blood plasma without changes in endothelial
permeability. It is a noninflammatory oedema Ex: Oedema in congestive cardiac
failure. Where as Exudate is oedema of inflamed tissue associated with increased
vascular permeability. Ex: Purulent exudate such as pus. Ecchymoses is Large
extravasations of blood into the skin and mucous membranes seen in haemorrhagic
disorders.</td></tr><tr valign=top height="15" style="background-color:White;"><td
width="30" style="border:solid 1px black; border-collapse:collapse ">86</td><td
width="250" style="border:solid 1px black; border-collapse:collapse ">the hormones
responsible for blood calcium level are</td><td width="200" style="border:solid
1px black; border-collapse:collapse ">A: parathormone<br><br>B:
calcitonin<br><br>C: 1,25 dihydroxycalciferol<br><br>D: all of the
above<br><br></td><td width="50" style="border:solid 1px black; border-
collapse:collapse ">D</td><td width="250" style="border:solid 1px black; border-
collapse:collapse ">Ref: Sembulingam, 2nd ed, pg no: 301
Exp: all the hormones are responsible for regulating blood calcium level:
CALCITONIN: By decreasing bone resorption
PARATHORMONE: By increasing bone resorption
1,25 DIHYDROXYCALCIFEROL: By increasing calcium absorption from
intestine</td></tr><tr valign=top height="15" style="background-color:#EFF3FB;"><td
width="30" style="border:solid 1px black; border-collapse:collapse ">87</td><td
width="250" style="border:solid 1px black; border-collapse:collapse ">On the crown
of the maxillary canine, the height of contour is normally located in the cervical
third of which of the following surfaces?</td><td width="200" style="border:solid
1px black; border-collapse:collapse ">A: Labial<br><br>B: Lingual<br><br>C: Both
labial and lingual<br><br>D: Neither labial nor lingual<br><br></td><td width="50"
style="border:solid 1px black; border-collapse:collapse ">C</td><td width="250"
style="border:solid 1px black; border-collapse:collapse ">► The height of contour,
or crest of curvature, is the greatest elevation
of the tooth, either incisocervically or occlusocervically, on a specific surface
of the crown. The labial and lingual surfaces of a tooth also have a height of
contour that is easily seen when viewing the tooth’s profile from the proximal.
►On the crown of the maxillary canine, the height of contour is normally located in
the cervical third of both labial and lingual surfaces, similar to all anterior
teeth. Choice D, neither labial or lingual surface, is incorrect.
► For posterior teeth, the height of contour for the crown’s labial (choice A)
surface is in the cervical third.</td></tr><tr valign=top height="15"
style="background-color:White;"><td width="30" style="border:solid 1px black;
border-collapse:collapse ">88</td><td width="250" style="border:solid 1px black;
border-collapse:collapse ">The non-working pathway of the maxillary cusps on the
mandibular posterior teeth is toward the</td><td width="200" style="border:solid
1px black; border-collapse:collapse ">A: distofacial.<br><br>B:
distolingual.<br><br>C: mesiofacial.<br><br>D: mesiolingual.<br><br></td><td
width="50" style="border:solid 1px black; border-collapse:collapse ">A</td><td
width="250" style="border:solid 1px black; border-collapse:collapse ">►When the
mandible moves into lateral excursive movement, the
nonworking-side condyle moves forward and down, and the working side rotates. This
causes the maxillary teeth to follow a pathway that is facial and distal to the
mandibular molars on the nonworking side.
► ( refer: diagrams in the Aim MDS Keynotes/ Videos Dental anatomy) for more
path</td></tr><tr valign=top height="15" style="background-color:#EFF3FB;"><td
width="30" style="border:solid 1px black; border-collapse:collapse ">89</td><td
width="250" style="border:solid 1px black; border-collapse:collapse ">The principal
fibrous elements of the periodontal ligament in adults consist chiefly of</td><td
width="200" style="border:solid 1px black; border-collapse:collapse ">A: elastic
fibers<br><br>B: collagen fibers<br><br>C: reticular fibers<br><br>D: a mixture of
elastic and collagen fibers<br><br></td><td width="50" style="border:solid 1px
black; border-collapse:collapse ">B</td><td width="250" style="border:solid 1px
black; border-collapse:collapse ">Ref: Orbans, Ed: 10th, Pg no: 216, Expl: The
fibers in periodontal ligament are the collagen and the oxytalan fibers. The
elastic fibers are restricted to the walls of the blood vessels.The majority of the
fibers are collagen ( predominently Type I collagen , but type III collagen is also
present).</td></tr><tr valign=top height="15" style="background-color:White;"><td
width="30" style="border:solid 1px black; border-collapse:collapse ">90</td><td
width="250" style="border:solid 1px black; border-collapse:collapse ">The principal
types of nerves in the dental pulp are</td><td width="200" style="border:solid 1px
black; border-collapse:collapse ">A: parsympathetic and afferent fibers<br><br>B:
sympathetic and afferent fibers<br><br>C: parasympathetic fibers only<br><br>D:
sympathetic fibers only<br><br></td><td width="50" style="border:solid 1px black;
border-collapse:collapse ">B</td><td width="250" style="border:solid 1px black;
border-collapse:collapse ">Ref: Orbans, Ed: 10th, Pg no: 156, Expl: The majority of
the nerves that enter the pulp are nonmyelinated and are sympathetic in nature. The
bundles of nerve fibers enter the tooth through apical foramen and proceed to the
coronal area. The peripheral axons form a network of nerves located adjacent to the
cell-rich zone. This is termed as the plexus of Rashkow or the parietal layer of
nerves. The large myelinated fibers carry the sensation of pain that may be caused
by external stimuli.</td></tr><tr valign=top height="15" style="background-
color:#EFF3FB;"><td width="30" style="border:solid 1px black; border-
collapse:collapse ">91</td><td width="250" style="border:solid 1px black; border-
collapse:collapse ">In the healing of an ulcer, the epithelium that eventually will
cover the defects is derived from</td><td width="200" style="border:solid 1px
black; border-collapse:collapse ">A: intact epithelium at the ulcer
margin<br><br>B: metaplasia of fibroblasts to epithelial cells<br><br>C:
transformation of fibrinous exudate to epithelium<br><br>D: transformation of
endothelial cells to epithelial cells<br><br></td><td width="50"
style="border:solid 1px black; border-collapse:collapse ">A</td><td width="250"
style="border:solid 1px black; border-collapse:collapse ">Ref: Shafers, Ed: 5th, Pg
no: 816 , Expl:The replacement of the lost tissue in ulcer is by granulation
tissue filling the defect followed by epithelialization. The epidermal growth
factor , produced by the epithelium around the damaged area helps in regeneration
of the epithelial tissue. However, the fibroblast growth factor released by
macrophages mediates the fibroblast activity and the endothelial growth factor
triggers the formation of the blood vessels.</td></tr><tr valign=top height="15"
style="background-color:White;"><td width="30" style="border:solid 1px black;
border-collapse:collapse ">92</td><td width="250" style="border:solid 1px black;
border-collapse:collapse ">In erythema multiforme, typical oral involvement
consists of</td><td width="200" style="border:solid 1px black; border-
collapse:collapse ">A: fissured tongue<br><br>B: acute gingivitis<br><br>C: diffuse
mucosal ulcerations<br><br>D: white straitions on the buccal mucosa<br><br></td><td
width="50" style="border:solid 1px black; border-collapse:collapse ">C</td><td
width="250" style="border:solid 1px black; border-collapse:collapse ">Ref: Shafers,
Ed: 5th, Pg no: 1112, 124 . Expl: Oral mucous membrane lesions are macules ,
papules, or vesicles which erode or ulcerate and bleed freely. Tongue, B. mucosa
and gingiva are the most common places in o. cavity.Bulls eye shaped lesions on
skin is a charcteristic feature of E. multiforme.
Fissured tongue is associated with Downs syndrome and Melkerson-Rosenthal syndrome.
White straitions are usually seen in leukoplakia, lichen planus, chronic cheek
bite,etc.</td></tr><tr valign=top height="15" style="background-color:#EFF3FB;"><td
width="30" style="border:solid 1px black; border-collapse:collapse ">93</td><td
width="250" style="border:solid 1px black; border-collapse:collapse ">Which of the
following dental findings is frequently observed in cases of Pagets disease of
bone</td><td width="200" style="border:solid 1px black; border-collapse:collapse
">A: Hypercementosis<br><br>B: Apical root resoption<br><br>C: Internal resorption
of the pulp<br><br>D: Widening of the periodontal ligament<br><br></td><td
width="50" style="border:solid 1px black; border-collapse:collapse ">A</td><td
width="250" style="border:solid 1px black; border-collapse:collapse ">Ref: White &
Pharoah, Ed: 5th, Pg no: 367, Ingle, 5thed, pgno 139 ,Expl:Hypercementosis occurs
in patients with Pagets disease of bone and with hyperpituitarism (gigantism and
acromegaly). The effects of inflammation on surrounding cancellous bone include
stimulation of bone formation, resulting in a sclerotic pattern, or bone
resorption, resulting in radiolucency. The periodontal ligament space involved in
the lesion will be widened; this widening is greatest at the source of the
inflammation. For example, with periapical lesions the widening is greatest around
the apical region of the root; in periodontal disease the widening is greatest at
the alveolar crest. With chronic infections, root resorption may occur and cortical
boundaries may be resorbed. Internal resorption of pulp is seen in chronic pulpal
inflammation, it also occurs as an idiopathic dystrophic change. Trauma in the form
of an accidental blow, or traumatic cavity preparation, has often been indicted as
a triggering mechanism for internal resorption.</td></tr><tr valign=top height="15"
style="background-color:White;"><td width="30" style="border:solid 1px black;
border-collapse:collapse ">94</td><td width="250" style="border:solid 1px black;
border-collapse:collapse ">Multiple periapical radiolucencies are common in
patients with which of the following conditions?</td><td width="200"
style="border:solid 1px black; border-collapse:collapse ">A: Taurodontism<br><br>B:
Dentinal dysplasia<br><br>C: Amelogenesis imperfecta<br><br>D: Dentinogenesis
imperfecta<br><br></td><td width="50" style="border:solid 1px black; border-
collapse:collapse ">B</td><td width="250" style="border:solid 1px black; border-
collapse:collapse ">Ref: White & Pharoah, Ed: 5th, Pg no: 348 -49, Expl:
Association of these periapical radiolucencies with noncarious teeth is an
important feature for recognition of this particular entity.The differential
diagnosis for dentin dysplasia may include only one other entity, dentinogenesis
imperfecta. Because these two conditions seem to fortn a continuum, their
differentiation may be difficult at first. Both entities can produce altered color
and occluded pulp chambersfinding a thistle-tube-shaped pulp chamber ina
singlerooted tooth strengthens the probability of dentin dysplasia If the roots are
short and narrow, the condition is likely to be dentinogenesis imperfecta. On the
other hand, normal-appearing roots or practically no roots at all should suggest
dentin dysplasia. Periapical rarefying osteitis in association with noncarious
teeth are more commonly seen in dentin dysplasia.</td></tr><tr valign=top
height="15" style="background-color:#EFF3FB;"><td width="30" style="border:solid
1px black; border-collapse:collapse ">95</td><td width="250" style="border:solid
1px black; border-collapse:collapse ">Which of the following is an advantage of IV
administration of a drug?</td><td width="200" style="border:solid 1px black;
border-collapse:collapse ">A: it eliminates side effects<br><br>B: minimal skill is
necessary<br><br>C: it allows titration of the drug<br><br>D: sedatives drugs are
compatible with IV<br><br></td><td width="50" style="border:solid 1px black;
border-collapse:collapse ">C</td><td width="250" style="border:solid 1px black;
border-collapse:collapse ">Ref:Tripathi, Ed: 5th, Pg no: 10, Expl: one big
advantage of IV route is titration of the dose with the response is possible. One
of the main disadvantage is that it is the most risky route - vital organs like
Heart, brain, etc get exposed to high concentration of the drug.</td></tr><tr
valign=top height="15" style="background-color:White;"><td width="30"
style="border:solid 1px black; border-collapse:collapse ">96</td><td width="250"
style="border:solid 1px black; border-collapse:collapse ">The most common cause of
postextraction bleeding is</td><td width="200" style="border:solid 1px black;
border-collapse:collapse ">A: failure of the patient to follow postextraction
instructions<br><br>B: congenital coagulation factor deficiency ( eg:
hemophilia)<br><br>C: acquired coagulation factor deficiency (eg: anticoagulants
like Coumadin)<br><br>D: inhibition of ADP release by platelets because of
ingestion of analgesics containig aspirin<br><br></td><td width="50"
style="border:solid 1px black; border-collapse:collapse ">A</td><td width="250"
style="border:solid 1px black; border-collapse:collapse ">Ref:, Ed:, Pg no:, Expl:
With proper medical and drug history B, C and D are ruled out. Where as option A is
common because the patient is anxious and may not follow the post operative
instructions.</td></tr><tr valign=top height="15" style="background-
color:#EFF3FB;"><td width="30" style="border:solid 1px black; border-
collapse:collapse ">97</td><td width="250" style="border:solid 1px black; border-
collapse:collapse ">A group of antibiotics related both structurally and by mode of
action to the penicillins is</td><td width="200" style="border:solid 1px black;
border-collapse:collapse ">A: cycloserines.<br><br>B: cephalosporins.<br><br>C:
chloramphenicols.<br><br>D: polymyxins<br><br></td><td width="50"
style="border:solid 1px black; border-collapse:collapse ">B</td><td width="250"
style="border:solid 1px black; border-collapse:collapse ">
Reference:Elander, R. P. (2003). "Industrial production of beta-lactam
antibiotics". Applied microbiology and biotechnology 61 (5–6): 385–392.
Explanation: Both penicillins and cephalosporins belong to B lactam antibiotics.
The penicillins and cephalosporins which have so far proved clinically useful have
the general structures I and H respectively. The general similarity in structure of
the two fused ring systems is clearly due to common features in their biogenesis.
MODE OF ACTION: β-Lactam antibiotics are bacteriocidal, and act by inhibiting the
synthesis of the peptidoglycan layer of bacterial cell walls. The peptidoglycan
layer is important for cell wall structural integrity, especially in Gram-positive
organisms, being the outermost and primary component of the wall</td></tr><tr
valign=top height="15" style="background-color:White;"><td width="30"
style="border:solid 1px black; border-collapse:collapse ">98</td><td width="250"
style="border:solid 1px black; border-collapse:collapse ">A two-days old developing
plaque would consist primarily of</td><td width="200" style="border:solid 1px
black; border-collapse:collapse ">A: filamentous organism<br><br>B: gram-positive
cocci and rod-like organisms<br><br>C: a structureless, non-mineralized
pellicle<br><br>D: spirochetal organisms<br><br></td><td width="50"
style="border:solid 1px black; border-collapse:collapse ">B</td><td width="250"
style="border:solid 1px black; border-collapse:collapse ">Ref: Carranza, Ed: 11th,
Pg no: 143, Expl : Streptococci and actinomyces are the primary colonizers of
plaque. These organisma prepare a favourable envinronment for later( secondary
colonizers), which have a fastidius growth requirements.Ex: P. intermedia, P.
gingivalis.</td></tr><tr valign=top height="15" style="background-
color:#EFF3FB;"><td width="30" style="border:solid 1px black; border-
collapse:collapse ">99</td><td width="250" style="border:solid 1px black; border-
collapse:collapse ">The strength of dental investments for gold alloys is dependent
upon the amount of</td><td width="200" style="border:solid 1px black; border-
collapse:collapse ">A: silica<br><br>B: carbon<br><br>C: gypsum<br><br>D:
alumina<br><br></td><td width="50" style="border:solid 1px black; border-
collapse:collapse ">C</td><td width="250" style="border:solid 1px black; border-
collapse:collapse ">Ref: Phillips, Ed: 10th, Pg no: 483, Expl: The compressive
strength of the investment mold is the primary factor to be considered, in
addition to the expansion . The comp. strength is increased according to the
amount and type of gypsum binder present,Eg : the use of alpha hemihydrate instead
of plaster definitely increases the compressive strength of the
investment.</td></tr><tr valign=top height="15" style="background-color:White;"><td
width="30" style="border:solid 1px black; border-collapse:collapse ">100</td><td
width="250" style="border:solid 1px black; border-collapse:collapse ">When a liquid
wets a solid completely, the contact angle between the liquid and the solid
is</td><td width="200" style="border:solid 1px black; border-collapse:collapse
">A: 0degrees<br><br>B: 45degrees<br><br>C: 90degrees<br><br>D: 135
degrees<br><br></td><td width="50" style="border:solid 1px black; border-
collapse:collapse ">A</td><td width="250" style="border:solid 1px black; border-
collapse:collapse ">Ref:, Ed:, Pg no:, Expl: the spreading of liquids on solids, or
the tendency for wetting surfaces, by measuring the angle of contact between the
liquid and the solid surface.The greater the tendency to wet the surface, the lower
the contact angle, until complete wetting occurs at an angle equal to
zero.</td></tr><tr valign=top height="15" style="background-color:#EFF3FB;"><td
width="30" style="border:solid 1px black; border-collapse:collapse ">101</td><td
width="250" style="border:solid 1px black; border-collapse:collapse ">A patient
recently received a blow to the mouth. Radiographs show a horizontal mid root
fracture of a maxillary central incisor. The tooth is not mobile and is
asymptomatic; however, it does not respond to pulp testing. No radiographic lesion
is present. Proper treatment is to</td><td width="200" style="border:solid 1px
black; border-collapse:collapse ">A: institute root canal treatment to include both
segments ofthe tooth.<br><br>B: extract the coronal segment of the tooth and remove
surgically the apical segment<br><br>C: remove surgically the apical segment of the
tooth and retrofill the coronal segment<br><br>D: render no treatment at this time.
The tooth should be periodically rechecked clinically a:ld
radiographically.<br><br></td><td width="50" style="border:solid 1px black;
border-collapse:collapse ">D</td><td width="250" style="border:solid 1px black;
border-collapse:collapse ">Ref: Ingle, Ed: 5th, Pg no: 812, Expl: Root fractures
are not always horizontal; in fact, probably more often than not, the angulation of
fractures is diagonal. Treatment: If there is no mobility and the tooth is
symptomless, the fracture is likely to be in the apical one-third of the root, and
no treatment is necessary. If the coronal fragment is mobile, treatment is
indicated. The initial treatment consists of repositioning the coronal segment (if
it is displaced) and then stabilizing the tooth to allow healing of the periodontal
ligament supporting the coronal segment.Repositioning can be as simple as pushing
the tooth into place with finger pressure, or orthodontic intervention may be
required to move the displaced segment into properalignment and Splinting. The
splint should allow for functional movement of the tooth to promote healing, and
the length of stabilization time is 4 to 6 weeks. Following initial treatment by
reduction and stabilization, repair by calcific and/or fibrous deposition is very
likely. About 80% of properly treated root fractures heal
successfully.</td></tr><tr valign=top height="15" style="background-
color:White;"><td width="30" style="border:solid 1px black; border-
collapse:collapse ">102</td><td width="250" style="border:solid 1px black; border-
collapse:collapse ">Which of the following is the best space maintainer?</td><td
width="200" style="border:solid 1px black; border-collapse:collapse ">A: Nance
holding arch<br><br>B: A band and loop appliance<br><br>C: Removable accrylic
appliance<br><br>D: A pulpectomized primary molar<br><br></td><td width="50"
style="border:solid 1px black; border-collapse:collapse ">D</td><td width="250"
style="border:solid 1px black; border-collapse:collapse ">Ref: Ingle, Ed: 5th, Pg
no: 889, Expl: The treatment objectives in nonvital pulp therapy for primary teeth
are to (1) maintain the tooth free of infection, (2) biomechanically cleanse and
obturate the root canals, (3) promote physiologic root resorption, and (4) hold the
space for the erupting permanent tooth. The treatment of choice to achieve these
objectives is pulpectomy, which involves the removal of necrotic pulp tissue
followed by filling the root canals with a resorbable cement. Indications for this
procedure include teeth with poor chance of vital pulp treatment success, strategic
importance with respect to space maintenance, absence of severe root resorption,
absence of surrounding bone loss from infection, and expectation of
restorability.</td></tr><tr valign=top height="15" style="background-
color:#EFF3FB;"><td width="30" style="border:solid 1px black; border-
collapse:collapse ">103</td><td width="250" style="border:solid 1px black; border-
collapse:collapse ">Which of the following is least likely to resolve from
persistent long term thumb sucking?</td><td width="200" style="border:solid 1px
black; border-collapse:collapse ">A: A deep overbite<br><br>B: Protrusion of
maxillary incisors<br><br>C: Construction of maxilary arch<br><br>D: Rotation of
maxialry laetral incisors<br><br></td><td width="50" style="border:solid 1px
black; border-collapse:collapse ">A</td><td width="250" style="border:solid 1px
black; border-collapse:collapse ">Ref: Balaji, Ed: 2nd, Pg no: 107, 412, Expl: The
effects of thumb sucking are 1) Labial tipping of maxillary anteriors, 2) the
overjet increases due to proclination of maxillary anteriors, 3) anterior open
bite, the cheek muscles contract during thumb sucking resulting in a narrow
maxillary arch which predisposes to posterior cross bite. However, the etiology for
deep overbite is Skeletal which is genetic in origin or dental deep bite which due
to over eruption of anteriors or infra-occlusion of molars.</td></tr><tr valign=top
height="15" style="background-color:White;"><td width="30" style="border:solid 1px
black; border-collapse:collapse ">104</td><td width="250" style="border:solid 1px
black; border-collapse:collapse ">Which of the following conditions is usually
present in a Class-II, Divison-II, malocclusion?</td><td width="200"
style="border:solid 1px black; border-collapse:collapse ">A: Open bite<br><br>B:
Steep mandibular plane<br><br>C: Mesiocclusion of permanent first molars<br><br>D:
Lingual inclination of maxillary central incisors<br><br></td><td width="50"
style="border:solid 1px black; border-collapse:collapse ">D</td><td width="250"
style="border:solid 1px black; border-collapse:collapse ">Ref: Balaji, Ed: 2nd, Pg
no: 78, Expl: In Angels Class II Div 2 the upper central incisors are lingually
inclined and the lateral incisors are labially tipped overlapping the centrals. The
lingually inclined upper incisors gives the arch a squarish appearance and
mandibular gingival tissue is often traumatized . by excessive tipped upper
incisors. the patient exhibits normal perioral muscle activity.</td></tr><tr
valign=top height="15" style="background-color:#EFF3FB;"><td width="30"
style="border:solid 1px black; border-collapse:collapse ">105</td><td width="250"
style="border:solid 1px black; border-collapse:collapse ">Orthodontic correction of
which of the following is most easily retained?</td><td width="200"
style="border:solid 1px black; border-collapse:collapse ">A: Diastema<br><br>B:
Rotation<br><br>C: Expansion<br><br>D: Anterior crossbite<br><br></td><td
width="50" style="border:solid 1px black; border-collapse:collapse ">D</td><td
width="250" style="border:solid 1px black; border-collapse:collapse ">Ref: Balaji,
Ed: 2nd, Pg no: 445, Expl: Correction of anterior cross bites requires
no retention or a short trem retention. Where as , rotation, expansion and
diastema demands for prolonged or permanent retention</td></tr><tr valign=top
height="15" style="background-color:White;"><td width="30" style="border:solid 1px
black; border-collapse:collapse ">106</td><td width="250" style="border:solid 1px
black; border-collapse:collapse ">Which of the following results from addition of
more water initially to a mix of gypsum products(hemihydrate)?</td><td width="200"
style="border:solid 1px black; border-collapse:collapse ">A: more expansion, more
strength, more abrasion resistance<br><br>B: more expansion, less strength, less
abrasion resistance<br><br>C: less expansion, more strength, less abrasion
resistance<br><br>D: less expansion, less strength, less abrasion
resistance<br><br></td><td width="50" style="border:solid 1px black; border-
collapse:collapse ">D</td><td width="250" style="border:solid 1px black; border-
collapse:collapse ">Ref:Craig, Ed: 11th, Pg no:400,404,401. Expl: The compressive
strength is inversely related to the W/P ratio of the mix. The more water used to
make the mix, the lower the compressive strength. The W/P ratio of the mix also has
an effect, with an increase in the ratio reducing the setting expansion.The surface
hardness of unmodified gypsum materials
is related in a general way to their compressive strength. High compressive
strengths of the hardened mass correspond to high surface hardnesses. After the
final setting occurs, the surface hardness remains practically constant until most
excess water is evaporated from the surface, after which its increase is similar to
the increase in compressive strength..</td></tr><tr valign=top height="15"
style="background-color:#EFF3FB;"><td width="30" style="border:solid 1px black;
border-collapse:collapse ">107</td><td width="250" style="border:solid 1px black;
border-collapse:collapse ">The area of buccal frenum of a complete mandibular
denture must be thinned to allow for the contraction of which of the following
muscles</td><td width="200" style="border:solid 1px black; border-
collapse:collapse ">A: Caninus<br><br>B: masseter<br><br>C: zygomaticus<br><br>D:
triangularis<br><br></td><td width="50" style="border:solid 1px black; border-
collapse:collapse ">D</td><td width="250" style="border:solid 1px black; border-
collapse:collapse ">Ref: Bouchers, Ed: 9th, Pg no: 187, 128, Expl: The buccal frenm
connects as a continuos band through the modiolus at the corner of the mouth and on
up to the buccal frenum attachment of maxilla.Caninus muscle attaches beneath the
buccal frenum in maxillary arch and affects the position of buccal frenum in
maxillary arch . Masseter muscle decides the thickness of maxillary buccal flange
in denture.</td></tr><tr valign=top height="15" style="background-color:White;"><td
width="30" style="border:solid 1px black; border-collapse:collapse ">108</td><td
width="250" style="border:solid 1px black; border-collapse:collapse ">All of the
following are catagorised as secondary lymphoid organs except</td><td width="200"
style="border:solid 1px black; border-collapse:collapse ">A: Lymph nodes<br><br>B:
Spleen<br><br>C: Thymus<br><br>D: Subepithelial collections of
lymphocytes<br><br></td><td width="50" style="border:solid 1px black; border-
collapse:collapse ">C</td><td width="250" style="border:solid 1px black; border-
collapse:collapse ">Reference: Immunology: essential and fundamental By Sulabha
Pathak, Urmi Palan page no 90
Explanation : Primary lymphoid organs
The central or primary lymphoid organs generate lymphocytes from immature
progenitor cells.
The thymus and the bone marrow constitute the primary lymphoid tissues involved in
the production and early selection of lymphocytes.

</td></tr><tr valign=top height="15" style="background-color:#EFF3FB;"><td


width="30" style="border:solid 1px black; border-collapse:collapse ">109</td><td
width="250" style="border:solid 1px black; border-collapse:collapse ">Action of
styloglossus muscle</td><td width="200" style="border:solid 1px black; border-
collapse:collapse ">A: Protrusion<br><br>B: Sidewise<br><br>C: Upwards & backwards
pull of tongue<br><br>D: Retraction and depression tongue<br><br></td><td
width="50" style="border:solid 1px black; border-collapse:collapse ">C</td><td
width="250" style="border:solid 1px black; border-collapse:collapse ">ref:Chaurasia
5th edi pg no 272 table 17.1
exp : Styloglossus - origin from tip and part of anterior surface of styloid
process , inserts into the side of the ongue . Its action is to pull tongue upward
& backward .</td></tr><tr valign=top height="15" style="background-
color:White;"><td width="30" style="border:solid 1px black; border-
collapse:collapse ">110</td><td width="250" style="border:solid 1px black; border-
collapse:collapse ">Ceruminous glands present in the ear are:</td><td width="200"
style="border:solid 1px black; border-collapse:collapse ">A: Modified eccrine
glands<br><br>B: Modified apocrine glands<br><br>C: Mucous gland<br><br>D: Modified
holocrine glands<br><br></td><td width="50" style="border:solid 1px black; border-
collapse:collapse ">B</td><td width="250" style="border:solid 1px black; border-
collapse:collapse ">REF ; chaurasia 4th edtn page no. 256
EXP : Ceruminous glands are modified sweat glands.Which are nothing but
apocrine</td></tr><tr valign=top height="15" style="background-color:#EFF3FB;"><td
width="30" style="border:solid 1px black; border-collapse:collapse ">111</td><td
width="250" style="border:solid 1px black; border-collapse:collapse ">In cerebral
cortex the afferent fibre passes to subcortex region and represent as:</td><td
width="200" style="border:solid 1px black; border-collapse:collapse ">A:
Collateral<br><br>B: Projection<br><br>C: Commissural<br><br>D: O.
Assimilation<br><br></td><td width="50" style="border:solid 1px black; border-
collapse:collapse ">A</td><td width="250" style="border:solid 1px black; border-
collapse:collapse ">REF : Guyton 11th edtn pg no. 688
EXP : The axons from the giant Betz cells send short collaterals back to the cortex
itself. These collaterals are believed to inhibit adjacent regions of the cortex
when the Betz cells discharge, thereby “sharpening” the boundaries of the
excitatory signal.</td></tr><tr valign=top height="15" style="background-
color:White;"><td width="30" style="border:solid 1px black; border-
collapse:collapse ">112</td><td width="250" style="border:solid 1px black; border-
collapse:collapse ">Facial N. stimulation during testing of nerve indicated by
contraction of muscle</td><td width="200" style="border:solid 1px black; border-
collapse:collapse ">A: Temporalis<br><br>B: Masseter<br><br>C:
Sternoleidomastoid<br><br>D: Orbicularis oris<br><br></td><td width="50"
style="border:solid 1px black; border-collapse:collapse ">D</td><td width="250"
style="border:solid 1px black; border-collapse:collapse ">REF : Chaurasia 5th edi
pg no 57
EXP : Clinically , the facial nerve is examined by testing the following muscles :
1.Frontalis: Ask the patient to look upwards without moving his head, and look for
the normal horizontal wrinkles of the forehead
2. Corrugator supercilii: Frowning and making vertical wrinkles of the forehead
3. Orbicularis oculi: Tight closure of the eyes
4. Orbicularis oris: Whistling and pursing the mouth
5. Dilators of mouth: Showing the teeth.
6. Buccinator: Puffing the mouth and then blowing forcibly as in whistling
7. Platysma: Forcible pulling of the angles of the mouth downwards and backwards
forming prominent vertical folds of skin on the side of the neck. The platysma
contracts along with the risorius</td></tr><tr valign=top height="15"
style="background-color:#EFF3FB;"><td width="30" style="border:solid 1px black;
border-collapse:collapse ">113</td><td width="250" style="border:solid 1px black;
border-collapse:collapse ">Pudendal nerve supplying motor part to external
sphincter is derived from</td><td width="200" style="border:solid 1px black;
border-collapse:collapse ">A: L5-S1 roots<br><br>B: S1-S2 roots<br><br>C: L2-L3
roots<br><br>D: S2-S3 roots<br><br></td><td width="50" style="border:solid 1px
black; border-collapse:collapse ">D</td><td width="250" style="border:solid 1px
black; border-collapse:collapse ">REF : BDC, 4th ed, Vol 2, Pgno: 335 & 382
EXP : Pudendal nerve is derived from S2, S3, & S4,and is a brnach of the perineal
nerve.</td></tr><tr valign=top height="15" style="background-color:White;"><td
width="30" style="border:solid 1px black; border-collapse:collapse ">114</td><td
width="250" style="border:solid 1px black; border-collapse:collapse ">Membrane
fluidity is increased by:</td><td width="200" style="border:solid 1px black;
border-collapse:collapse ">A: Stearic acid<br><br>B: Palmitic acid<br><br>C:
Cholesterol<br><br>D: Linoleic acid<br><br></td><td width="50" style="border:solid
1px black; border-collapse:collapse ">D</td><td width="250" style="border:solid
1px black; border-collapse:collapse ">REF ; Harper 26th edtn pg no. 190
EXP : Unsaturated fatty acids in phospholipids of the cell membrane are important
in maintaining membrane fluidity. A high ratio of polyunsaturated fatty acids to
saturated fatty acids (P:S ratio) in the diet is a major factor in lowering plasma
cholesterol concentrations
and is considered to be beneficial in preventing coronary heart disease. Linolenic
Is Polyunsaturared fatty acid and.</td></tr><tr valign=top height="15"
style="background-color:#EFF3FB;"><td width="30" style="border:solid 1px black;
border-collapse:collapse ">115</td><td width="250" style="border:solid 1px black;
border-collapse:collapse ">In type specific plasma blood grouping all the following
mentioned groups can take the plasma except:</td><td width="200"
style="border:solid 1px black; border-collapse:collapse ">A: Plasma blood group AB
receives plasma From blood 0 donor<br><br>B: Plasma blood group 0 receives plasma
from blood group A donor<br><br>C: Plasma blood group A receives plasma from AB
blood donor group<br><br>D: Plasma blood group B receives plasma from AB blood
donor group<br><br></td><td width="50" style="border:solid 1px black; border-
collapse:collapse ">A</td><td width="250" style="border:solid 1px black; border-
collapse:collapse ">REF : Self explanatory
EXP : As we all know plasma contains antibody and RBC contains antigen. So in a
type specific plasma trnsfusion i.e antibody trnsfusion a patient is having A B
antibodies already. Even O group plasma has the same agglutination will occur
hence. So it means Patient with o blood group is receiving from O blood group.=
indirectly.</td></tr><tr valign=top height="15" style="background-color:White;"><td
width="30" style="border:solid 1px black; border-collapse:collapse ">116</td><td
width="250" style="border:solid 1px black; border-collapse:collapse ">Flow
cytometry analysis is done for:</td><td width="200" style="border:solid 1px black;
border-collapse:collapse ">A: Blood glucose estimation<br><br>B: LDL fraction
estimation in serum<br><br>C: CD4/CD8 estimation in AIDS<br><br>D: Separation of
proteins<br><br></td><td width="50" style="border:solid 1px black; border-
collapse:collapse ">C</td><td width="250" style="border:solid 1px black; border-
collapse:collapse ">REF :Molecular biomethods handbook By John M. Walker, Ralph
Rapley
EXP : Flow cytometry is literally measuring cells or particles while they are
moving in a liquid . The primary uses of this technology is in haemotolgy ,
haematopathology , immunology & trransplant medicine . It is especially useful for
classifying subsets of cells in leukemias and AIDS . this method can also be used
to count reticulocytes .</td></tr><tr valign=top height="15" style="background-
color:#EFF3FB;"><td width="30" style="border:solid 1px black; border-
collapse:collapse ">117</td><td width="250" style="border:solid 1px black; border-
collapse:collapse ">Liver synthesizes all except</td><td width="200"
style="border:solid 1px black; border-collapse:collapse ">A: CJ complement
component<br><br>B: HapToglobin<br><br>C: Fibrinogen<br><br>D:
Immunoglobulin<br><br></td><td width="50" style="border:solid 1px black; border-
collapse:collapse ">D</td><td width="250" style="border:solid 1px black; border-
collapse:collapse ">REF : Ganong, 22nd ed, pgno 541
EXP : Immmunoglobulins are produced by B lymphocytes( plasma cells), whereas alpha
and beta globulins are produced primarily by hepatocytes.Most of the plasma
proteins are synthesized by liver.</td></tr><tr valign=top height="15"
style="background-color:White;"><td width="30" style="border:solid 1px black;
border-collapse:collapse ">118</td><td width="250" style="border:solid 1px black;
border-collapse:collapse ">Vital capacity is decreased, timed vital capacity (FEV
1.0 %) is normal in:</td><td width="200" style="border:solid 1px black; border-
collapse:collapse ">A: Bronchial asthma<br><br>B: Scoliosis<br><br>C: Chronic
bronchitis<br><br>D: Acute bronchitis<br><br></td><td width="50"
style="border:solid 1px black; border-collapse:collapse ">B</td><td width="250"
style="border:solid 1px black; border-collapse:collapse ">Vital capacity is
decreased, timed vital capacity (FEV 1.0 %) is normal in Scoliosis</td></tr><tr
valign=top height="15" style="background-color:#EFF3FB;"><td width="30"
style="border:solid 1px black; border-collapse:collapse ">119</td><td width="250"
style="border:solid 1px black; border-collapse:collapse ">A segment of an
eucaryotic gene that is not represented in the mature mRNA, is known as</td><td
width="200" style="border:solid 1px black; border-collapse:collapse ">A:
Intron<br><br>B: Exon<br><br>C: Plasmid<br><br>D: TATA box<br><br></td><td
width="50" style="border:solid 1px black; border-collapse:collapse ">A</td><td
width="250" style="border:solid 1px black; border-collapse:collapse ">reference:
Harpers Biochemistry 26th ed Page 361
Explanation: Interspersed within the amino acid-coding portions (exons) of many
genes are long sequences of DNA that do not contribute to the genetic information
ultimately translated into the amino acid sequence of a protein molecule. In fact,
these intervening sequences (introns) actually
interrupt the coding region of structural genes. These</td></tr><tr valign=top
height="15" style="background-color:White;"><td width="30" style="border:solid 1px
black; border-collapse:collapse ">120</td><td width="250" style="border:solid 1px
black; border-collapse:collapse ">In molecular cloning blue white screening is
used:</td><td width="200" style="border:solid 1px black; border-collapse:collapse
">A: To screen for recombinant vectors<br><br>B: To detect gene mutations<br><br>C:
To identify desired chromosomal DNA insert in plasmid vectors<br><br>D: To detect
DNA in situ<br><br></td><td width="50" style="border:solid 1px black; border-
collapse:collapse ">C</td><td width="250" style="border:solid 1px black; border-
collapse:collapse ">REF :http://en.wikipedia.org/wiki/Blue_white_screen
EXP: The blue-white screen is a molecular technique that allows for the detection
of successful ligations in vector-based gene cloning. DNA of interest is ligated
into a vector. The vector is then transformed into competent cell (bacteria). The
competent cells are grown in the presence of X-gal. If the ligation was successful,
the bacterial colony will be white; if not, the colony will be blue. This technique
allows for the quick and easy detection of successful ligation, without the need to
individually test each colony</td></tr><tr valign=top height="15"
style="background-color:#EFF3FB;"><td width="30" style="border:solid 1px black;
border-collapse:collapse ">121</td><td width="250" style="border:solid 1px black;
border-collapse:collapse ">The type of enzyme inhibtion (in which succinate
dehydrogenase reaction is inhibited by malonate is an example of:</td><td
width="200" style="border:solid 1px black; border-collapse:collapse ">A:
Noncompetitive<br><br>B: Uncompetitive<br><br>C: Competitive<br><br>D:
Allosteric<br><br></td><td width="50" style="border:solid 1px black; border-
collapse:collapse ">C</td><td width="250" style="border:solid 1px black; border-
collapse:collapse ">Ref: Harper, 26 ed, pgno 67
Expl: Enzyme inhibition can be 2 types. Competitive and Non-competitive. In
competitive inhibition the inhibitor binds reversible to the same site the
substrate would bind normally , and therefore competes with the substrate for that
site. here the malonate competes with succinate(substrate) for the succinate
dehydrogenase reaction.</td></tr><tr valign=top height="15" style="background-
color:White;"><td width="30" style="border:solid 1px black; border-
collapse:collapse ">122</td><td width="250" style="border:solid 1px black; border-
collapse:collapse ">Phosphodiester bonds are found:</td><td width="200"
style="border:solid 1px black; border-collapse:collapse ">A: Between two chains of
insulin<br><br>B: Between two strands of DNA<br><br>C: In phospholipids<br><br>D:
All of the above<br><br></td><td width="50" style="border:solid 1px black; border-
collapse:collapse ">B</td><td width="250" style="border:solid 1px black; border-
collapse:collapse ">A phosphodiester bond is a group of strong covalent bonds
between the phosphorus atom in a phosphate group and two other molecules over two
ester bonds. Phosphodiester bonds make up the backbone of the strands of DNA. In
DNA and RNA, the phosphodiester bond is the linkage between the 3 carbon atom of
one sugar molecule and the 5 carbon of another, deoxyribose in DNA and ribose in
RNA.</td></tr><tr valign=top height="15" style="background-color:#EFF3FB;"><td
width="30" style="border:solid 1px black; border-collapse:collapse ">123</td><td
width="250" style="border:solid 1px black; border-collapse:collapse ">Glycogen is a
polysaccharide composed of glucose units linked by</td><td width="200"
style="border:solid 1px black; border-collapse:collapse ">A: Alpha-1,4<br><br>B:
Alpha-1,6<br><br>C: Beta-1,3<br><br>D: Beta-1,4<br><br></td><td width="50"
style="border:solid 1px black; border-collapse:collapse ">A</td><td width="250"
style="border:solid 1px black; border-collapse:collapse
">Reference:http://oregonstate.edu/instruction/bb450/summer09/lecture/glycogennotes
.html
Explanation:Glycogen is a polysaccharide composed of glucose units linked by alpha
1-4 glycosidic bonds, with occasional alpha 1-6 glycosidic bonds which provide
branching points.</td></tr><tr valign=top height="15" style="background-
color:White;"><td width="30" style="border:solid 1px black; border-
collapse:collapse ">124</td><td width="250" style="border:solid 1px black; border-
collapse:collapse ">Action of primer is to:</td><td width="200"
style="border:solid 1px black; border-collapse:collapse ">A: Removal of smear
layer<br><br>B: Increases surface free energy of dentin<br><br>C: Forms thin layer
between collagen and resin<br><br>D: Bonds with composite<br><br></td><td
width="50" style="border:solid 1px black; border-collapse:collapse ">B</td><td
width="250" style="border:solid 1px black; border-collapse:collapse ">REF :Page 245
Sturdevant Operative dentistry
EXP: The adhesive system must have a low surface tension and the substrate
must have a high surface free energy for adequate interfacial contact.dentin
consists of two distinct substrates, one of high-surface energy (hydroxyapatite)
and one of low surface energy (collagen). Thus, after etching with acidic
agents, the dense web of exposed collagen is a lowsurface energy substrate An
increase in the critical surface tension of dentin by surface-active components
(such as primers) is highly desirable in this case, since a direct correlation
between surface energy of dentin and shear bond strengths has been
demonstrated</td></tr><tr valign=top height="15" style="background-
color:#EFF3FB;"><td width="30" style="border:solid 1px black; border-
collapse:collapse ">125</td><td width="250" style="border:solid 1px black; border-
collapse:collapse ">The function of fluoride in flux during soldering of stainless
steel is:</td><td width="200" style="border:solid 1px black; border-
collapse:collapse ">A: Forms a surface protective layer<br><br>B: Decreases the
melting point<br><br>C: Dissolves chromic oxide layer<br><br>D: Dissolves copper
oxide layer<br><br></td><td width="50" style="border:solid 1px black; border-
collapse:collapse ">C</td><td width="250" style="border:solid 1px black; border-
collapse:collapse ">ref phillips 11 th ed pg 609 : fluoride flux is type III flux
ie it dissolves any metal oxide with which it comes in contact , acting as a
solvent (stainless steel has a protective layer of chromium oxide on the
surface)</td></tr><tr valign=top height="15" style="background-color:White;"><td
width="30" style="border:solid 1px black; border-collapse:collapse ">126</td><td
width="250" style="border:solid 1px black; border-collapse:collapse ">The mode of
curing in microwave cured dentures is by</td><td width="200" style="border:solid
1px black; border-collapse:collapse ">A: Heat conduction<br><br>B: Energy
conversion<br><br>C: Electrical resistance<br><br>D: Magnetic field
creation<br><br></td><td width="50" style="border:solid 1px black; border-
collapse:collapse ">B</td><td width="250" style="border:solid 1px black; border-
collapse:collapse ">ref phil;ips 11 th ed pg 734 : resins can also be polymerised
by microwave energy - ruling out other options and by knowing the mechanism of
working of a microwave - microwave radiation is converted to heat energy when it
strikes a surface</td></tr><tr valign=top height="15" style="background-
color:#EFF3FB;"><td width="30" style="border:solid 1px black; border-
collapse:collapse ">127</td><td width="250" style="border:solid 1px black; border-
collapse:collapse ">Function of carnauba wax</td><td width="200"
style="border:solid 1px black; border-collapse:collapse ">A: To provide a smooth
and glossy surface<br><br>B: Increase toughness<br><br>C: Decrease
toughness<br><br>D: Decrease flow at room temperature<br><br></td><td width="50"
style="border:solid 1px black; border-collapse:collapse ">D</td><td width="250"
style="border:solid 1px black; border-collapse:collapse ">Ref to Phillip’s,
Anusavice, 11th edition, pg no 286. Carnauba wax is quite hard, and it has a
relatively high melting point. It is combined with paraffin to decrease the flow at
mouth temperature. It also contributes to the glossiness of the wax
surface.</td></tr><tr valign=top height="15" style="background-color:White;"><td
width="30" style="border:solid 1px black; border-collapse:collapse ">128</td><td
width="250" style="border:solid 1px black; border-collapse:collapse ">A veterinary
doctor had pyrexia of unknown origin. His blood culture was positive for gram-
negative short bacilli, which was oxidase positive. Which one of the following is
the likely organism grown in culture?</td><td width="200" style="border:solid 1px
black; border-collapse:collapse ">A: Pasturella spp.<br><br>B: Francisella
spp.<br><br>C: Bartonella spp<br><br>D: Brucella spp.<br><br></td><td width="50"
style="border:solid 1px black; border-collapse:collapse ">D</td><td width="250"
style="border:solid 1px black; border-collapse:collapse ">Reference
Anathanarayanan 7th Edition Pages 331, 345 and 420
Explanation:
Brucellosis is a zoonosis transmitted to humans from infected animals. Its clinical
features are not disease specific. synonyms: (e.g., Mediterranean fever, Malta
fever, Gibraltar fever, Cyprus fever); from the remittent character of its fever
(e.g., undulant fever); or from its resemblance to malaria and typhoid (e.g.,
typhomalarial fever, intermittent typhoid).
Explanation
1. Pasteurella multocida is a common pathogen in infected dog and cat bites.
2. Francisella tularensis is the etiologic agent of tularemia, which, with rare
exceptions, is the only disease produced by this genus. The organism is a small,
gram-negative, pleomorphic, nonmotile, non-spore-forming bacillus measuring 0.2 um
by 0.2 to 0.7 um.
3. Bartonella spp., including B. bacilliformis, B. henselae, B. quintana, and B.
clarridgeiae, are tiny gram-negative bacilli that can adhere to and invade
mammalian cells, including endothelial cells and erythrocytes.
4. Brucella are small aerobic gram-negative bacilli are unencapsulated, nonmotile,
non-spore-forming, facultative intracellular parasites that cause lifelong
infection in animals. Brucella are Catalase positive and oxidase positive. Nitrates
are reduced to nitrites. Citrate is not utilized. Indole is not produced and MR and
VP tests are negative</td></tr><tr valign=top height="15" style="background-
color:#EFF3FB;"><td width="30" style="border:solid 1px black; border-
collapse:collapse ">129</td><td width="250" style="border:solid 1px black; border-
collapse:collapse ">Enzyme found in Retrovirus</td><td width="200"
style="border:solid 1px black; border-collapse:collapse ">A: Reverse Trans
criptase<br><br>B: RNA polymerase<br><br>C: DNA polymerase<br><br>D:
Topoisomerse<br><br></td><td width="50" style="border:solid 1px black; border-
collapse:collapse ">A</td><td width="250" style="border:solid 1px black; border-
collapse:collapse ">ref:ref:Ananthanarayan 8 th edi pg no 565
exp : Retroviruses are enveloped spherical viruses that are released by budding
through the host cell membrane .The characteristic feature of retroviruses is the
presence within the virion of the unusual enzyme RNA dependent DNA polymerase or
reverse transcriptase . This enzyme prepares a DNA copy of the retroviral RNA
genome -initially an RNA :DNA hybrid & then its double stranded DNA form
.</td></tr><tr valign=top height="15" style="background-color:White;"><td
width="30" style="border:solid 1px black; border-collapse:collapse ">130</td><td
width="250" style="border:solid 1px black; border-collapse:collapse ">Which of the
following statements about gram positive cocci is False:</td><td width="200"
style="border:solid 1px black; border-collapse:collapse ">A: Staphylococcus
saprophyticus causes UTI in females<br><br>B: Micrococci are oxidase
positive<br><br>C: Most Enterococci are sensitive to penicillin<br><br>D:
Pneumococci are capsulated<br><br></td><td width="50" style="border:solid 1px
black; border-collapse:collapse ">C</td><td width="250" style="border:solid 1px
black; border-collapse:collapse ">reference: ananthnarayan 7th ed page no : 200
Explanation:
►Enterococcii are frequently resistant to Penicillin
►Enterococci are more resistant to Penicillin than other streptococci.
►Resistance to Penicillin may occur either due to beta lactamase production or due
to altered Penicillin binding proteins
►Enterococcii may also be resistant to Aminoglycosides, Cephalosporins,
Monobactems, and Vancomycin
►Micrococci are Aerobic, Oxidase positive, Catalase positive
►Pneumococci have polysaccharide capsule</td></tr><tr valign=top height="15"
style="background-color:#EFF3FB;"><td width="30" style="border:solid 1px black;
border-collapse:collapse ">131</td><td width="250" style="border:solid 1px black;
border-collapse:collapse ">Organisms which do not grow in the complete absence of
oxygen but grow best in an amount of oxygen less than that contained in the air are
known as:</td><td width="200" style="border:solid 1px black; border-
collapse:collapse ">A: Facultative aerobes<br><br>B: Obligate anaerobes<br><br>C:
Microaerophiles<br><br>D: Facultative anaerobes<br><br></td><td width="50"
style="border:solid 1px black; border-collapse:collapse ">C</td><td width="250"
style="border:solid 1px black; border-collapse:collapse ">obligate aerobe: grow
only in the presence of oxygen, obligate anaerobe: grow only in anaerobic
conditions, facultative anaerobic: ordinarily aeroic but can grow in the presence
of oxygen.</td></tr><tr valign=top height="15" style="background-color:White;"><td
width="30" style="border:solid 1px black; border-collapse:collapse ">132</td><td
width="250" style="border:solid 1px black; border-collapse:collapse ">The three
general classesof vaccines are</td><td width="200" style="border:solid 1px black;
border-collapse:collapse ">A: killed organisms, virulent bacteria and attenuated
viruses.<br><br>B: toxoids, antitoxins and attenuated organisms.<br><br>C: killed
organisms, toxoids and attenuated organisms.<br><br>D: killed organisms, antitoxins
and gamma globulins.<br><br></td><td width="50" style="border:solid 1px black;
border-collapse:collapse ">C</td><td width="250" style="border:solid 1px black;
border-collapse:collapse ">Reference:J.K. Sinha & S. Bhattacharya. A Text Book of
Immunology. Academic Publishers. p. 318.
Explanation:Vaccines are dead or inactivated organisms or purified products derived
from them.
There are several types of vaccines in use. These represent different strategies
used to try to reduce risk of illness, while retaining the ability to induce a
beneficial immune response. They are:
Killed
Attentuated
toxoids
Subunit
Conjugate
Experimenta;
Valence</td></tr><tr valign=top height="15" style="background-color:#EFF3FB;"><td
width="30" style="border:solid 1px black; border-collapse:collapse ">133</td><td
width="250" style="border:solid 1px black; border-collapse:collapse ">When a drug
is evaluated for its usefulness in controlled conditions, it is termed as a trial
signifying</td><td width="200" style="border:solid 1px black; border-
collapse:collapse ">A: Efficacy<br><br>B: Effectiveness<br><br>C:
Efficiency<br><br>D: Effect modification<br><br></td><td width="50"
style="border:solid 1px black; border-collapse:collapse ">A</td><td width="250"
style="border:solid 1px black; border-collapse:collapse ">REF ;
EXP ; In a healthcare context, efficacy indicates the capacity for beneficial
change (or therapeutic effect) of a given intervention (e.g. a medicine, medical
device, surgical procedure, or a public health intervention</td></tr><tr valign=top
height="15" style="background-color:White;"><td width="30" style="border:solid 1px
black; border-collapse:collapse ">134</td><td width="250" style="border:solid 1px
black; border-collapse:collapse ">Most important factor for calculating dose for
NSAIDS in children?</td><td width="200" style="border:solid 1px black; border-
collapse:collapse ">A: Age of child<br><br>B: Body weight of child<br><br>C:
Flavour of salt<br><br>D: Extent of pain<br><br></td><td width="50"
style="border:solid 1px black; border-collapse:collapse ">B</td><td width="250"
style="border:solid 1px black; border-collapse:collapse ">REF :Essentials of
Pharmacology for Dentistry, Tripathi page 50
EXP: although various formula for calculation of the child dose based on the age
are there. But the dose can be calculated by Body Wt ( MORE ACCURATELY) and for
many drugs manufacturer give dosage in mg/kg basis.</td></tr><tr valign=top
height="15" style="background-color:#EFF3FB;"><td width="30" style="border:solid
1px black; border-collapse:collapse ">135</td><td width="250" style="border:solid
1px black; border-collapse:collapse ">Which of the following is not true about
barbiturates?</td><td width="200" style="border:solid 1px black; border-
collapse:collapse ">A: They depress all areas of CNS.<br><br>B: Act at GABA:BZD
receptor Cl- channel complex.<br><br>C: They depress glutamate induced neuronal
depolarization.<br><br>D: They bind to BZD receptor.<br><br></td><td width="50"
style="border:solid 1px black; border-collapse:collapse ">D</td><td width="250"
style="border:solid 1px black; border-collapse:collapse ">Ref K.D Tripathi-5th
Edition pg no 359Barbiturates depress all areas of the CNS, but the recticular
activating system is the most sensitive; its depression is primarily responsible
for the inability to maintain wakefulness. They act primarily at the GABA:BZD
receptor- chlorine ion channel complex and potentiate GABAergic inhibition by
increasing the life time of chlorine channel opening induced by GABA. and inhibit
calcium dependent release of neurotransmitters. In addition they depress glutamate
induced neuronal depolarization through AMPA receptors. At very high
concentrations, barbiturates depress sodium and potassium channels
also.</td></tr><tr valign=top height="15" style="background-color:White;"><td
width="30" style="border:solid 1px black; border-collapse:collapse ">136</td><td
width="250" style="border:solid 1px black; border-collapse:collapse ">Which of the
following precipitate hypokalaemia and cardiac arrythmias " when used along with
digitalis?</td><td width="200" style="border:solid 1px black; border-
collapse:collapse ">A: Diuretics<br><br>B: Quanidine<br><br>C: Verampril<br><br>D:
Adrenergic drugs<br><br></td><td width="50" style="border:solid 1px black; border-
collapse:collapse ">A</td><td width="250" style="border:solid 1px black; border-
collapse:collapse ">Ref to pg no 464 of K.D. Tripathi, 5th edition. Diuretics cause
hypokalemia which can precipitate digitalis arrythmias, potassium supplements may
be given prophylactically in such cases.</td></tr><tr valign=top height="15"
style="background-color:#EFF3FB;"><td width="30" style="border:solid 1px black;
border-collapse:collapse ">137</td><td width="250" style="border:solid 1px black;
border-collapse:collapse ">Abnormal lab finding in hemophilia:</td><td width="200"
style="border:solid 1px black; border-collapse:collapse ">A: Increased PT<br><br>B:
Increased PTT<br><br>C: Increased CT<br><br>D: Increased BT<br><br></td><td
width="50" style="border:solid 1px black; border-collapse:collapse ">B</td><td
width="250" style="border:solid 1px black; border-collapse:collapse ">REF :Robbins
8th edi pg no 474
EXP : Hemophilia A is the most common hereditary disease associated with serious
bleeding. It is an X-linked recessive disorder that is caused by reduction in
factor VIII activity. It primarily affects males.Typically, patients with
hemophilia A have a prolonged PTT that is corrected by mixing the patients plasma
with normal plasma. Treatment involves infusion of factor VIII.
Factor IX Deficiency (Hemophilia B, Christmas Disease)- Severe factor IX
deficiency is an X-linked disorder that is indistinguishable clinically from
hemophilia A but is much less common. The PTT is prolonged, and the bleeding time
is normal. The diagnosis of Christmas disease (named after the first patient with
this condition) is made with specific assays of factor IX. It is treated by
infusion of recombinant factor IX.
</td></tr><tr valign=top height="15" style="background-color:White;"><td width="30"
style="border:solid 1px black; border-collapse:collapse ">138</td><td width="250"
style="border:solid 1px black; border-collapse:collapse ">A peripheral smear with
increased neutrophils, basophils, eosinophils, and platelets is highly suggestive
of:</td><td width="200" style="border:solid 1px black; border-collapse:collapse
">A: Actute myeloid leukemia<br><br>B: Acute lymphoblastic leukemia<br><br>C:
Chronic myelogenous leukemia<br><br>D: Myelodysplastic syndrome<br><br></td><td
width="50" style="border:solid 1px black; border-collapse:collapse ">C</td><td
width="250" style="border:solid 1px black; border-collapse:collapse ">Ref: Robins,
7th ed, pgno 697
Expl: Haematlogical Findings in CML: Anaemia, Leucocytosis, increased platelet
count, Basophillia, Eosinophillia and immature white cells. The peripheral blood
picture other three option will have decrease in plate count and
neutrophil.</td></tr><tr valign=top height="15" style="background-
color:#EFF3FB;"><td width="30" style="border:solid 1px black; border-
collapse:collapse ">139</td><td width="250" style="border:solid 1px black; border-
collapse:collapse ">The processs in which the primitive vascular network is
assembled from angioblasts (endothelial cell precursors) during embryonic
development is called as</td><td width="200" style="border:solid 1px black;
border-collapse:collapse ">A: vasculogenesis<br><br>B: angiogenesis,<br><br>C:
neovascularization<br><br>D: all of the above<br><br></td><td width="50"
style="border:solid 1px black; border-collapse:collapse ">A</td><td width="250"
style="border:solid 1px black; border-collapse:collapse ">Reference: general
Pathology by Robbins, 8th edition, page no: 68-70
explanation:Blood vessels are assembled by two processes:
1) vasculogenesis, in which the primitive vascular network is assembled from
angioblasts (endothelial cell precursors) during embryonic development; and
2) angiogenesis, or neovascularization, in which preexisting vessels send out
capillary sprouts to produce new vessels</td></tr><tr valign=top height="15"
style="background-color:White;"><td width="30" style="border:solid 1px black;
border-collapse:collapse ">140</td><td width="250" style="border:solid 1px black;
border-collapse:collapse ">The elevation of gingival tissue normaly found directly
lingual to the embrasure between maxillary central incisors is the</td><td
width="200" style="border:solid 1px black; border-collapse:collapse ">A: incisive
papilla<br><br>B: lingual frenum<br><br>C: incisive tubercle<br><br>D: median
rugae<br><br></td><td width="50" style="border:solid 1px black; border-
collapse:collapse ">A</td><td width="250" style="border:solid 1px black; border-
collapse:collapse ">The elevation of gingival tissue normaly found directly lingual
to the embrasure between maxillary central incisors is the incisive
papilla</td></tr><tr valign=top height="15" style="background-color:#EFF3FB;"><td
width="30" style="border:solid 1px black; border-collapse:collapse ">141</td><td
width="250" style="border:solid 1px black; border-collapse:collapse ">If a childs
teeth do not form, this would primarily affect the growth of the:</td><td
width="200" style="border:solid 1px black; border-collapse:collapse ">A:
Maxilla.<br><br>B: Mandible.<br><br>C: Whole face.<br><br>D: Alveolar
bone.<br><br></td><td width="50" style="border:solid 1px black; border-
collapse:collapse ">D</td><td width="250" style="border:solid 1px black; border-
collapse:collapse ">If a childs teeth do not form, this would primarily affect the
growth of the alveolar bone</td></tr><tr valign=top height="15" style="background-
color:White;"><td width="30" style="border:solid 1px black; border-
collapse:collapse ">142</td><td width="250" style="border:solid 1px black; border-
collapse:collapse ">A one-year old child is expected to have erupted which of the
following primary Maxillary and mandibular teeth?</td><td width="200"
style="border:solid 1px black; border-collapse:collapse ">A: Incisors and
canines<br><br>B: Incisors and first molars<br><br>C: Central incisors and
canines<br><br>D: Mandibular central and lateral incisors and maxillary central
incisors<br><br></td><td width="50" style="border:solid 1px black; border-
collapse:collapse ">B</td><td width="250" style="border:solid 1px black; border-
collapse:collapse ">Ref: Orbans, 10th ed, pg no 372
Expl: CI erupts at 6 to 71/2 months and LI erupts at 7 to 9 months, 1st molars at
12 to 14months, canines at 16 to 18 months, 2nd molars at 20 to 24
months</td></tr><tr valign=top height="15" style="background-color:#EFF3FB;"><td
width="30" style="border:solid 1px black; border-collapse:collapse ">143</td><td
width="250" style="border:solid 1px black; border-collapse:collapse ">the order of
development of tooth germ is</td><td width="200" style="border:solid 1px black;
border-collapse:collapse ">A: bud-cap-bell stage<br><br>B: cap-bell-bud
stage<br><br>C: bell-cap-bud stage<br><br>D: non<br><br></td><td width="50"
style="border:solid 1px black; border-collapse:collapse ">A</td><td width="250"
style="border:solid 1px black; border-collapse:collapse ">orbans oral histology and
embryology -11th edition,pg no.32
bud,cap and bell stage is explained upon the shape of tooth germ.</td></tr><tr
valign=top height="15" style="background-color:White;"><td width="30"
style="border:solid 1px black; border-collapse:collapse ">144</td><td width="250"
style="border:solid 1px black; border-collapse:collapse ">The electrolytes in
tissue fluid help to retain water. Whichof the following is of greatest influence
in this phenomenon?</td><td width="200" style="border:solid 1px black; border-
collapse:collapse ">A: K+ .<br><br>B: Na+<br><br>C: CI-<br><br>D:
HPO4=<br><br></td><td width="50" style="border:solid 1px black; border-
collapse:collapse ">B</td><td width="250" style="border:solid 1px black; border-
collapse:collapse ">Reference: Guyton, Physiology, 11th ed, pg no: 815
Explanation:When a person becomes dehydrated, large amounts of aldosterone almost
always are secreted by the cortices of the adrenal glands.Within 1 to 3 hours this
aldosterone causes increased activation of the enzyme and transport mechanisms for
all aspects of sodium absorption by the intestinal epithelium.And the increased
sodium
absorption in turn causes secondary increases in absorption of chloride ions,
water, and some other
substances. This effect of aldosterone is especially important in the colon because
it allows virtually no loss of sodium
chloride in the feces and also little water loss.Thus, the function of aldosterone
in the intestinal tract is the
same as that achieved by aldosterone in the renal tubules, which also serves to
conserve sodium chloride and water in the body when a person become becomes
dehydrated.</td></tr><tr valign=top height="15" style="background-
color:#EFF3FB;"><td width="30" style="border:solid 1px black; border-
collapse:collapse ">145</td><td width="250" style="border:solid 1px black; border-
collapse:collapse ">Enamel crystals</td><td width="200" style="border:solid 1px
black; border-collapse:collapse ">A: Are the same size as dentin crystals<br><br>B:
Are about 5 microns thick<br><br>C: Contain organic inclusions<br><br>D: Are
approximately hexagonal in cross section<br><br></td><td width="50"
style="border:solid 1px black; border-collapse:collapse ">D</td><td width="250"
style="border:solid 1px black; border-collapse:collapse ">Ref: Ten cate, 7th ed,
pgno 143
Expl: Enamel is built from closely packed and long, ribbon like caronatoapatite
crystals. In cross sectios the crystal appears Hexogonal in outline in maturing
enamel. However, fully mature enamel crystals are no longer perfectly hexagonal
rather appear as irregular in outline.</td></tr><tr valign=top height="15"
style="background-color:White;"><td width="30" style="border:solid 1px black;
border-collapse:collapse ">146</td><td width="250" style="border:solid 1px black;
border-collapse:collapse ">83. Most of the lymph is returned to the blood at which
of the following sites?</td><td width="200" style="border:solid 1px black; border-
collapse:collapse ">A: Right brachiocephalic vein<br><br>B: Right external jugular
vein<br><br>C: Junction of left internal jugular and subclavian veins<br><br>D:
Junction of right internal jugular and subclavian veins<br><br></td><td width="50"
style="border:solid 1px black; border-collapse:collapse ">C</td><td width="250"
style="border:solid 1px black; border-collapse:collapse ">Reference:Human
Physiology: From Cells to Systems, by Lauralee Sherwood
Explanation:The blood does not directly come in contact with the parenchymal cells
and tissues in the body, but constituents of the blood first exit the microvascular
exchange blood vessels to become interstitial fluid, which comes into contact with
the parenchymal cells of the body. Lymph is the fluid that is formed when
interstitial fluid enters the initial lymphatic vessels of the lymphatic system.
The lymph is then moved along the lymphatic vessel network by either intrinsic
contractions of the lymphatic passages or by extrinsic compression of the lymphatic
vessels via external tissue forces (e.g. the contractions of skeletal muscles).
Eventually, the lymph vessels empty into the lymphatic ducts, which drain into one
of the two subclavian veins (near the junctions of the subclavian veins with the
internal jugular veins).</td></tr><tr valign=top height="15" style="background-
color:#EFF3FB;"><td width="30" style="border:solid 1px black; border-
collapse:collapse ">147</td><td width="250" style="border:solid 1px black; border-
collapse:collapse ">Which of the following is not a B-cell neoplasm?</td><td
width="200" style="border:solid 1px black; border-collapse:collapse ">A: Hairy cell
leukemia<br><br>B: Angiocentric lymphoma<br><br>C: Mantle cell lymphoma<br><br>D:
Burkitts lymphoma<br><br></td><td width="50" style="border:solid 1px black;
border-collapse:collapse ">B</td><td width="250" style="border:solid 1px black;
border-collapse:collapse ">Ref: Robbins, 7th ed, pg no 671
Expl: Angiocentric lymphoma is a T-cell neoplasm. The other three are B-cell
lymphomas.</td></tr><tr valign=top height="15" style="background-color:White;"><td
width="30" style="border:solid 1px black; border-collapse:collapse ">148</td><td
width="250" style="border:solid 1px black; border-collapse:collapse ">Water hammer
pulse seen in</td><td width="200" style="border:solid 1px black; border-
collapse:collapse ">A: Aortic stenoSiS<br><br>B: Aortic regurgitation<br><br>C:
Aortic stenosis and Aortic regurgitation<br><br>D: Mitral
regurgitation<br><br></td><td width="50" style="border:solid 1px black; border-
collapse:collapse ">B</td><td width="250" style="border:solid 1px black; border-
collapse:collapse ">REF : Harrison, 16th ed, pgno 1400
EXP : Water hammer pulse is seen in Aortic regurgitation. It is a large bounding
pulse, best felt on radial artery with the patients arm elevated, caused due to
increased stroke volume od left ventricle and decrease in</td></tr><tr valign=top
height="15" style="background-color:#EFF3FB;"><td width="30" style="border:solid
1px black; border-collapse:collapse ">149</td><td width="250" style="border:solid
1px black; border-collapse:collapse ">_______ was one of the first to recommend
taper as such, prescribing 5% to 20% per inch (3 to 12 degrees, respectively). A
taper of 2.5 to 6.5 degrees has been suggested as optimum.</td><td width="200"
style="border:solid 1px black; border-collapse:collapse ">A: Johnston<br><br>B:
Ward<br><br>C: Andrew<br><br>D: Tylman<br><br></td><td width="50"
style="border:solid 1px black; border-collapse:collapse ">B</td><td width="250"
style="border:solid 1px black; border-collapse:collapse ">Reference: Nallaswamy,
Textbook of prosthodontics, pg no: 567
Explanation: Theoretically, the more nearly parallel the opposing walls of a
preparation, the greater should be the retention. However, parallel walls are
impossible to create in the mouth without producing preparation undercuts.
Ward was one of the first to recommend taper as such, prescribing 5% to 20% per
inch (3 to 12 degrees, respectively). A taper of 2.5 to 6.5 degrees has been
suggested as optimum, but there is only a slight increase in stress as taper is
increased from 0 to 15 degrees. However, at 20 degrees, stress concentration was
found to increase sharply.
</td></tr><tr valign=top height="15" style="background-color:White;"><td width="30"
style="border:solid 1px black; border-collapse:collapse ">150</td><td width="250"
style="border:solid 1px black; border-collapse:collapse ">Blood collected without
anticoagulant and centrifuged the supernatant is</td><td width="200"
style="border:solid 1px black; border-collapse:collapse ">A: Plasma<br><br>B:
Serum<br><br>C: Serum minus fibrin<br><br>D: Plasma plus fibrin<br><br></td><td
width="50" style="border:solid 1px black; border-collapse:collapse ">B</td><td
width="250" style="border:solid 1px black; border-collapse:collapse ">reference
page 24: Choudhury if blood is collected without anticoagulant and is allowed to
clot and some further time is allowed so that the clot is allowed to shrink, a
fluid separates out. This fluid can be centrifuged further and the clear
supernatant part can be collected, this is called serum. So serum is plasma minus
fibrinogen.</td></tr><tr valign=top height="15" style="background-
color:#EFF3FB;"><td width="30" style="border:solid 1px black; border-
collapse:collapse ">151</td><td width="250" style="border:solid 1px black; border-
collapse:collapse ">which of the following is an antiemetic</td><td width="200"
style="border:solid 1px black; border-collapse:collapse ">A: promethazine<br><br>B:
metoclopromaide<br><br>C: Ondansetron<br><br>D: all of the above<br><br></td><td
width="50" style="border:solid 1px black; border-collapse:collapse ">D</td><td
width="250" style="border:solid 1px black; border-collapse:collapse ">REF:
Tripathi, Pharmacology, 6th ed, pg no:641
EXP:
The drugs used for vomiting are:
ANTICHOLINERGICS: Hyoscine
HI ANTAGONISTS: Promethazine
PROKINETIC DRUGS: Metoclopromaide
NEUROLEPTIC DRUGS: Chlopromoazine
5 HT3 ANTAGONISTS: Ondansetron</td></tr><tr valign=top height="15"
style="background-color:White;"><td width="30" style="border:solid 1px black;
border-collapse:collapse ">152</td><td width="250" style="border:solid 1px black;
border-collapse:collapse ">Complications of total thyroidectomy include all,
except:</td><td width="200" style="border:solid 1px black; border-
collapse:collapse ">A: Hoarseness<br><br>B: Airway obstruction<br><br>C:
Hemorrhage<br><br>D: Hypercalcemia<br><br></td><td width="50" style="border:solid
1px black; border-collapse:collapse ">D</td><td width="250" style="border:solid
1px black; border-collapse:collapse ">REF ; Das surgery pg no 657
EXP : Post operative complications after total thyroidectomy are diveided into
immediate complications and late complications . Immediate complications are ;
haemmorhage , infection , recurrent laryngeal nerve paralysis , thyroid crisis or
storm , respiratory obstruction , parathyroid insufficiency or tetany , Late
complications include thyroid insufficeiency , recurrent thyrotoxicosis ,
progressive exohtholmos , hypertrophic scar or keloid .</td></tr><tr valign=top
height="15" style="background-color:#EFF3FB;"><td width="30" style="border:solid
1px black; border-collapse:collapse ">153</td><td width="250" style="border:solid
1px black; border-collapse:collapse ">Secondary haemorrhage may be seen in</td><td
width="200" style="border:solid 1px black; border-collapse:collapse ">A: 5
days<br><br>B: 10 days<br><br>C: 15 days<br><br>D: 20 days<br><br></td><td
width="50" style="border:solid 1px black; border-collapse:collapse ">B</td><td
width="250" style="border:solid 1px black; border-collapse:collapse ">reference:
Page 48 General surgery,Love and Bailey
Explanation
Secondary haemorrhage
Secondary haemorrhage occurs after 7—14 days, and is due to infection and sloughing
of part of the wall of an artery.
Reactionary haemorrhage may follow primary haemorrhage within 24 hours (usually 4—6
hours) and is mainly due to rolling (‘slipping’) of a ligature, dislodgement of a
clot or cessation of reflex vasospasm.</td></tr><tr valign=top height="15"
style="background-color:White;"><td width="30" style="border:solid 1px black;
border-collapse:collapse ">154</td><td width="250" style="border:solid 1px black;
border-collapse:collapse ">Treatment of cystic hygroma is</td><td width="200"
style="border:solid 1px black; border-collapse:collapse ">A: Surgical
excision<br><br>B: Injection of sclerosants<br><br>C: Irradiation<br><br>D: no
treatment required<br><br></td><td width="50" style="border:solid 1px black;
border-collapse:collapse ">A</td><td width="250" style="border:solid 1px black;
border-collapse:collapse ">reference: Page 207 General surgery,Love and Bailey
Explanation
Cystic hygroma is an abnormal lymph-filled, often multilocular, space which usually
presents in childhood as a soft, brilliantly transluminable swelling in the base of
the neck. It is also found in the head and inguinat regions as they
develop from primitive lymph cisterns. It behaves like a benign tumour and grows
gradually in size, leading to cosmetic problems and compression of surrounding
structures.
Treatment:
Recurrence is common after simple aspiration and injection of sclerosant.
Excision is the treatment of choice but is technically challenging due to the large
number of vital structures in the vicinity.</td></tr><tr valign=top height="15"
style="background-color:#EFF3FB;"><td width="30" style="border:solid 1px black;
border-collapse:collapse ">155</td><td width="250" style="border:solid 1px black;
border-collapse:collapse ">Which of the following in true in case of AIDS patient?
</td><td width="200" style="border:solid 1px black; border-collapse:collapse ">A:
Acute pain<br><br>B: Diffuse red lesion of the attached gingiva<br><br>C: Gingiva
covered with pseudomembrane gingiva<br><br>D: Gingival itching<br><br></td><td
width="50" style="border:solid 1px black; border-collapse:collapse ">B</td><td
width="250" style="border:solid 1px black; border-collapse:collapse ">Reference:
Shafers Oral pathology 6th ed page
explanation: HIV associated periodontal lesions of by EC clearinghouse incluse:
1) Linear gingival erythema: non plaque induce gingivitis with distinct
erythematous band of the marginal gingiva with either diffuse or punctate erythema
of the attached gingiva.
2) Necrotising periodontal disease.
3) chronic periodontitis.</td></tr><tr valign=top height="15" style="background-
color:White;"><td width="30" style="border:solid 1px black; border-
collapse:collapse ">156</td><td width="250" style="border:solid 1px black; border-
collapse:collapse ">Acinic cell carcinomas of the salivary gland arise most often
in the</td><td width="200" style="border:solid 1px black; border-collapse:collapse
">A: Parotid salivary gland<br><br>B: Minor salivary glands<br><br>C: Submandibular
salivary gland.<br><br>D: Sublingual salivary gland<br><br></td><td width="50"
style="border:solid 1px black; border-collapse:collapse ">A</td><td width="250"
style="border:solid 1px black; border-collapse:collapse ">Reference: Shafers Oral
pathology 6th ed page 231
Explanation:
1) 80% in parotid gland, not seen in any other major gland
2) most common intraoral location is lips and buccal mucosa.</td></tr><tr
valign=top height="15" style="background-color:#EFF3FB;"><td width="30"
style="border:solid 1px black; border-collapse:collapse ">157</td><td width="250"
style="border:solid 1px black; border-collapse:collapse ">Premature synostosis of
coronal suture along with basal sutures is seen in:</td><td width="200"
style="border:solid 1px black; border-collapse:collapse ">A: Oxycephaly<br><br>B:
Brachycephaly<br><br>C: Trigonocephaly<br><br>D: Scandamocephaly<br><br></td><td
width="50" style="border:solid 1px black; border-collapse:collapse ">A</td><td
width="250" style="border:solid 1px black; border-collapse:collapse ">REF :Ghai
pediatrics pg no 602
EXP: .Oxycephaly- A congenital malformation of the skull in which premature
closure of the coronal and sagittal sutures results in accelerated upward growth of
the head, giving it a long, narrow appearance with the top pointed or conic. Also
called acrocephaly, hypsicephaly, oxycephalia, steeple head, tower head, tower
skull, turricephaly .The skull is foreshortened and the vault is high more pointed
and dome shaped. A short antero-posterior diameter with a high sloping forehead is
rather typical of Oxycephaly.</td></tr><tr valign=top height="15"
style="background-color:White;"><td width="30" style="border:solid 1px black;
border-collapse:collapse ">158</td><td width="250" style="border:solid 1px black;
border-collapse:collapse ">Ehler Danlos syndrome is:</td><td width="200"
style="border:solid 1px black; border-collapse:collapse ">A: Autosomal
dominant<br><br>B: Autosomal recessive<br><br>C: Sex linked dominant<br><br>D: Sex
linked recessive<br><br></td><td width="50" style="border:solid 1px black; border-
collapse:collapse ">A</td><td width="250" style="border:solid 1px black; border-
collapse:collapse ">REF :Atlas of genetic diagnosis and counseling By Harold Chen
page 342
EXP: Most common inheritance pattern of Ehlers danlos syndrome is autosomal dominat
although some typoes with autosomas recessive and X liked recessive are
seen.</td></tr><tr valign=top height="15" style="background-color:#EFF3FB;"><td
width="30" style="border:solid 1px black; border-collapse:collapse ">159</td><td
width="250" style="border:solid 1px black; border-collapse:collapse ">The following
is feature of Gardeners syndrome:</td><td width="200" style="border:solid 1px
black; border-collapse:collapse ">A: Multiple osteomas<br><br>B: Ulcerative
colitis<br><br>C: Lacrimal Gysts<br><br>D: Leiyomyoma<br><br></td><td width="50"
style="border:solid 1px black; border-collapse:collapse ">A</td><td width="250"
style="border:solid 1px black; border-collapse:collapse ">REF : Oral pathology
Shafers 6th edition page no 47
EXP : Gardner syndrome, also known as familial colorectal polyposis, is an
autosomal dominant form of polyposis characterized by the presence of multiple
polyps in the colon together with tumors outside the colon.The extracolonic tumors
may include osteomas of the skull, thyroid cancer, epidermoid cysts, fibromas and
sebaceous cysts,[3] as well as the occurrence of desmoid tumors</td></tr><tr
valign=top height="15" style="background-color:White;"><td width="30"
style="border:solid 1px black; border-collapse:collapse ">160</td><td width="250"
style="border:solid 1px black; border-collapse:collapse ">Hydrolysis of sucrose by
the enzyme sucrose yields</td><td width="200" style="border:solid 1px black;
border-collapse:collapse ">A: glucose<br><br>B: maltose.<br><br>C:
fructose.<br><br>D: both a and c<br><br></td><td width="50" style="border:solid
1px black; border-collapse:collapse ">D</td><td width="250" style="border:solid
1px black; border-collapse:collapse ">Reference:Ouyang X, Cirillo P, Sautin Y et
al. (June 2008). "Fructose Consumption as a Risk Factor for Non-alcoholic Fatty
Liver Disease". J. Hepatol. 48 (6): 993–9
Explanation:Fructose exists in foods either as a monosaccharide (free fructose) or
as a unit of a disaccharide (sucrose). Free fructose is absorbed directly by the
intestine. When fructose is consumed in the form of sucrose, it is digested (broken
down) and then absorbed as free fructose. As sucrose comes into contact with the
membrane of the small intestine, the enzyme sucrase catalyzes the cleavage of
sucrose to yield one glucose unit and one fructose unit, which are then each
absorbed.</td></tr><tr valign=top height="15" style="background-color:#EFF3FB;"><td
width="30" style="border:solid 1px black; border-collapse:collapse ">161</td><td
width="250" style="border:solid 1px black; border-collapse:collapse ">Generally,
the permanent maxillary central incisor has</td><td width="200"
style="border:solid 1px black; border-collapse:collapse ">A: two mamelons and two
developmental lobes.<br><br>B: two mamelons and three developmental
lobes.<br><br>C: three mamelons and two developmental lobes.<br><br>D: three
mamelons and four developmental lobes.<br><br></td><td width="50"
style="border:solid 1px black; border-collapse:collapse ">D</td><td width="250"
style="border:solid 1px black; border-collapse:collapse ">Reference: Wheelers,
Dental anatomy, pg no: 92
Explanation:In the anterior teeth, the four lobes are called the mesial, labial,
distal, and lingual lobes. Mesial, labial and distal lobes form mammelons in
central incisor and the lingual lobe forms the cingulum.</td></tr><tr valign=top
height="15" style="background-color:White;"><td width="30" style="border:solid 1px
black; border-collapse:collapse ">162</td><td width="250" style="border:solid 1px
black; border-collapse:collapse ">The bisecting angle technique is used to
take</td><td width="200" style="border:solid 1px black; border-collapse:collapse
">A: Bitewing radiographs<br><br>B: PeriapicaL radiographs<br><br>C: True occlusal
radiographs of the mandible<br><br>D: None of the above<br><br></td><td width="50"
style="border:solid 1px black; border-collapse:collapse ">B</td><td width="250"
style="border:solid 1px black; border-collapse:collapse ">Ref to oral radiology,
White and Pharoah, 5th edition, pg no 89-90 An early method for aligning the x-ray
beam and film with the teeth and jaws was the bisecting-angle technique. In this
method the film is placed as close to the teeth as possible without deforming it.
However, when the film is in this position, it is not parallel to the long axes of
the teeth. This arrangement inherently causes distortion. Nevertheless, by
directing the central ray perpendicular to an imaginary plane that bisects the
angle between the teeth and the film, the practitioner can make the length of the
tooths image on the film correspond to the actual length of the tooth. It is used
to take periapicaL radiographs.</td></tr><tr valign=top height="15"
style="background-color:#EFF3FB;"><td width="30" style="border:solid 1px black;
border-collapse:collapse ">163</td><td width="250" style="border:solid 1px black;
border-collapse:collapse ">The darkening of radiograph x-ray depends on all 4·
except:</td><td width="200" style="border:solid 1px black; border-
collapse:collapse ">A: Thickness of object<br><br>B: Quality and quantity of x-
rays<br><br>C: Angulation cone<br><br>D: Velocity of electron emitted from
cathode<br><br></td><td width="50" style="border:solid 1px black; border-
collapse:collapse ">C</td><td width="250" style="border:solid 1px black; border-
collapse:collapse ">Ref to oral radiology, White and Pharoah, 5th edition, pg no
78As exposure of the film increases, its optical density increases. Radiographic
density is influenced by exposure and the thickness and density of the
subject.</td></tr><tr valign=top height="15" style="background-color:White;"><td
width="30" style="border:solid 1px black; border-collapse:collapse ">164</td><td
width="250" style="border:solid 1px black; border-collapse:collapse ">An x-ray
picture shows bone expansion with honeycomb or soap bubble appearance and is
eccentrically ballooned. Cortical bone is destroyed and a periosteal reaction
evident. Most likely the lesion could be:</td><td width="200" style="border:solid
1px black; border-collapse:collapse ">A: Malignant melanoma<br><br>B: Central
ossifying fibroma<br><br>C: Aneurysmal bone cyst<br><br>D: Giant cell
tumor<br><br></td><td width="50" style="border:solid 1px black; border-
collapse:collapse ">C</td><td width="250" style="border:solid 1px black; border-
collapse:collapse ">Ref to oral radiology, White and Pharoah, 5th edition, pg no
503-504An ABC (Aneurysmal bone cyst) in the jaw usually manifests as a fairly rapid
bony swelling (Usually buccal or labial). Pain is an occasional complaint, and the
involved area may be tender on palpation. The mandible is involved more often than
the maxilla (ratio of 3:2), and the molar and ramus regions are more involved than
the anterior region. The periphery usually is well defined, and the shape is
circular or "hydraulic." Small initial lesions may show no evidence of an internal
structure. Often the internal aspect has a multilocular appearance (honey comb or
soap bubble appearance).</td></tr><tr valign=top height="15" style="background-
color:#EFF3FB;"><td width="30" style="border:solid 1px black; border-
collapse:collapse ">165</td><td width="250" style="border:solid 1px black; border-
collapse:collapse ">Etiology of mucocele is related to</td><td width="200"
style="border:solid 1px black; border-collapse:collapse ">A: Mechanical trauma to
the minor salivary gland excretory duct<br><br>B: Salivary calculi in the excretory
duct of major salivary gland<br><br>C: Acute infections<br><br>D: Smoking
tobacco<br><br></td><td width="50" style="border:solid 1px black; border-
collapse:collapse ">A</td><td width="250" style="border:solid 1px black; border-
collapse:collapse ">reference: textbook of oral surgery by Bhalaji page no
466</td></tr><tr valign=top height="15" style="background-color:White;"><td
width="30" style="border:solid 1px black; border-collapse:collapse ">166</td><td
width="250" style="border:solid 1px black; border-collapse:collapse ">The finish
line given with cast metal crowns is:</td><td width="200" style="border:solid 1px
black; border-collapse:collapse ">A: chamfer<br><br>B: shoulder<br><br>C: knife
edge<br><br>D: sloping shoulder<br><br></td><td width="50" style="border:solid 1px
black; border-collapse:collapse ">A</td><td width="250" style="border:solid 1px
black; border-collapse:collapse ">Reference: Nallaswamy, Textbook of
prosthodontics, pg no: 571
Explanation: INDICATIONS FOR FINISH LINES:
Chamfer: Lingual margins of metal ceramic, Margins of cast metal
Shoulder/ radial shoulder/sloping shoulder : Margins of all ceramic, Facial margins
of metal ceramic
Shoulder with bevel: Facial margins of metal ceramic, ledge or step Inlays and
onlays
Knife edge/ featheredge: Tipped teeth, lingual margin of mand.post

</td></tr><tr valign=top height="15" style="background-color:#EFF3FB;"><td


width="30" style="border:solid 1px black; border-collapse:collapse ">167</td><td
width="250" style="border:solid 1px black; border-collapse:collapse ">In systemic
LAtoxicity there is :</td><td width="200" style="border:solid 1px black; border-
collapse:collapse ">A: Post depression convulsion<br><br>B: Post convulsion
depression<br><br>C: Convulsions<br><br>D: Depression<br><br></td><td width="50"
style="border:solid 1px black; border-collapse:collapse ">B</td><td width="250"
style="border:solid 1px black; border-collapse:collapse ">Ref to pg no 214
Monheim’s , 7th edition. Although local anesthetics used in dentistry have the
ability to produce overt signs and symptoms of CNS stimulation, the effect is
actually produced by depressed of certain inhibitory centers. Depression of
inhib</td></tr><tr valign=top height="15" style="background-color:White;"><td
width="30" style="border:solid 1px black; border-collapse:collapse ">168</td><td
width="250" style="border:solid 1px black; border-collapse:collapse ">Which of the
following is not a feature of Le Fort II fracture:</td><td width="200"
style="border:solid 1px black; border-collapse:collapse ">A: EnophthaLmos<br><br>B:
Malocclusion<br><br>C: Paraesthesia<br><br>D: CSF rhinorrhea<br><br></td><td
width="50" style="border:solid 1px black; border-collapse:collapse ">A</td><td
width="250" style="border:solid 1px black; border-collapse:collapse ">Ref to pg no
328-329 of Neelima Malik. Enophthalmos is seen in zygomatic complex fracture,
orbital fractures and Le Fort III fracture. It is either caused by escape of
orbital contents like fat or by an increase in the volume of the bony
orbit.</td></tr><tr valign=top height="15" style="background-color:#EFF3FB;"><td
width="30" style="border:solid 1px black; border-collapse:collapse ">169</td><td
width="250" style="border:solid 1px black; border-collapse:collapse ">Oxygen
cylinder Is coded as</td><td width="200" style="border:solid 1px black; border-
collapse:collapse ">A: Black cylinder with white shoulder<br><br>B: Black cylinder
with grey shoulders<br><br>C: White cylinders with black shoulders<br><br>D: Grey
cylinder with white shoulders<br><br></td><td width="50" style="border:solid 1px
black; border-collapse:collapse ">A</td><td width="250" style="border:solid 1px
black; border-collapse:collapse ">Cylinders of pure (nearly) oxygen are black
(there are various colour codes for gas cylinders, but it varies from country to
country), but medical oxygen is black with a white stripe near the top.

* Hydrogen cylinders have a red shoulder.


* Nitrous oxide cylinders have a blue shoulder.
* Nitrogen cylinders have a black shoulder</td></tr><tr valign=top height="15"
style="background-color:White;"><td width="30" style="border:solid 1px black;
border-collapse:collapse ">170</td><td width="250" style="border:solid 1px black;
border-collapse:collapse ">Alpha adrenergic agonists are used in combination with
local anesthetics to</td><td width="200" style="border:solid 1px black; border-
collapse:collapse ">A: increase the rate of liver metabolism of local
anesthetic<br><br>B: increase the concentration of L.A Qt receptor site<br><br>C:
stimulate myocardial contraction<br><br>D: increases vascular absorption of
L.A<br><br></td><td width="50" style="border:solid 1px black; border-
collapse:collapse ">B</td><td width="250" style="border:solid 1px black; border-
collapse:collapse ">Ref to pg no 32 of Malamed, 2nd edition. Activation of alpha
receptors by a sympathomimetic drug produces a response that includes the
contraction of smooth muscle in blood vessels, vasoconstriction. Hence when used
with LA, the absorption of LA agent int</td></tr><tr valign=top height="15"
style="background-color:#EFF3FB;"><td width="30" style="border:solid 1px black;
border-collapse:collapse ">171</td><td width="250" style="border:solid 1px black;
border-collapse:collapse ">ln a healthy gingival, the distance between gingival
margin and free gingival groova is</td><td width="200" style="border:solid 1px
black; border-collapse:collapse ">A: 0.1-0.5 mm<br><br>B: 0.5-1 mm<br><br>C: 0.5-
1.5 mm<br><br>D: 1 mm-2 mm<br><br></td><td width="50" style="border:solid 1px
black; border-collapse:collapse ">C</td><td width="250" style="border:solid 1px
black; border-collapse:collapse ">Reference: Carranza 10th edition page 46
Explanation: In 50% of the individual the unattached (marginal) gingiva is
separated from the attached gingiva by free gingival groove. The marginal gingiva
is usually 1 mm wide.</td></tr><tr valign=top height="15" style="background-
color:White;"><td width="30" style="border:solid 1px black; border-
collapse:collapse ">172</td><td width="250" style="border:solid 1px black; border-
collapse:collapse ">The reverse bevel incision is made</td><td width="200"
style="border:solid 1px black; border-collapse:collapse ">A: Allow atraumatic
reflection of the gingival margins<br><br>B: Remove the infected tissue in the
sulcus<br><br>C: Provide access to the alveolar crest<br><br>D: All of the
above<br><br></td><td width="50" style="border:solid 1px black; border-
collapse:collapse ">B</td><td width="250" style="border:solid 1px black; border-
collapse:collapse ">ref:Carranza 10th edi pg no 928.929
exp : Internal bevel incsion - is the incision from which the flap is reflected to
expose the underlying bone & root . This incision is also termed the 1st incision
because it is the initial incision in the reflection of a flap , also called as
reverse bevel incision because its bevel is in reverse direction from that of the
gingivectomy incision . This incision removes the pocket lining .</td></tr><tr
valign=top height="15" style="background-color:#EFF3FB;"><td width="30"
style="border:solid 1px black; border-collapse:collapse ">173</td><td width="250"
style="border:solid 1px black; border-collapse:collapse ">Eluanin fibres are seen
in :</td><td width="200" style="border:solid 1px black; border-collapse:collapse
">A: Gingiva<br><br>B: Cementum<br><br>C: Alveolar bone<br><br>D: Periodontal
ligaments<br><br></td><td width="50" style="border:solid 1px black; border-
collapse:collapse ">D</td><td width="250" style="border:solid 1px black; border-
collapse:collapse ">Reference: Carranza 10th edition page 70 Explanation: Although
periodontal ligament does not contain mature elastin, two immature forms are
found, oxytalan and eluanin. Oxytalan: runs parallel to the root surface in the
vertical direction and bend to attach to the cementum in the cervical third of the
root. They are thought to regulate the blood flow. An elastic meshwork has been
described in PDL as being composed of many elastin lamellae with peripheral
oxytalan and eluanin fibres.</td></tr><tr valign=top height="15" style="background-
color:White;"><td width="30" style="border:solid 1px black; border-
collapse:collapse ">174</td><td width="250" style="border:solid 1px black; border-
collapse:collapse ">Which microorganism has 90% presence in localized aggressive
periodontitis?</td><td width="200" style="border:solid 1px black; border-
collapse:collapse ">A: Spirochetes<br><br>B: P. Gingivalis<br><br>C: P.
Intermedia<br><br>D: A. Actinomyecetemcomitans<br><br></td><td width="50"
style="border:solid 1px black; border-collapse:collapse ">D</td><td width="250"
style="border:solid 1px black; border-collapse:collapse ">Reference: p/ 507 of
Carranza, 10th edition. Explanation: A. actinomycetemcomitans has been considered
as the prime pathogen for Aggressive periodontitis because A. Association: A.a is
associated with 90 % cases of aggressive periodontitis B. Disease
progression:Sites with evidence of disease progression show elevated proportions of
A.a. C. Antibody titers: Patients with LAP have elevated serum antibody titers to
A.a D. Association with treatment outcomes: Correlation has been found between the
reduction of the number of A.a and successful treatment outcome. E. Demonstration
of virulence factors: Virulence factors that may contribute to the disease have
been demonstrated in A.a.</td></tr><tr valign=top height="15" style="background-
color:#EFF3FB;"><td width="30" style="border:solid 1px black; border-
collapse:collapse ">175</td><td width="250" style="border:solid 1px black; border-
collapse:collapse ">CADIA System is used to check?</td><td width="200"
style="border:solid 1px black; border-collapse:collapse ">A: Progression of
Periodontal Disease<br><br>B: Furcation Involvement<br><br>C: Mobility of
Teeth<br><br>D: Bleeding Sites<br><br></td><td width="50" style="border:solid 1px
black; border-collapse:collapse ">A</td><td width="250" style="border:solid 1px
black; border-collapse:collapse ">Reference: Carranza 10th edition pg no 587
Explanation: Computer assisted densitometric image analysis system (CADIA): a
video camera measures the light transmitted through a radiograph, and the signals
from the camera are converted to the gray scale images. Bone density can be
followed quantitatively over time (i.e progression of the periodontal
diseases).</td></tr><tr valign=top height="15" style="background-color:White;"><td
width="30" style="border:solid 1px black; border-collapse:collapse ">176</td><td
width="250" style="border:solid 1px black; border-collapse:collapse ">Gingiva is
supplied by</td><td width="200" style="border:solid 1px black; border-
collapse:collapse ">A: Supra periosteal vessels<br><br>B: Subperiosteal
vessels<br><br>C: Apical vessels<br><br>D: All of the above<br><br></td><td
width="50" style="border:solid 1px black; border-collapse:collapse ">A</td><td
width="250" style="border:solid 1px black; border-collapse:collapse ">Reference:
p/60, carranza 10th ed Explanation: There are 3 sources of blood supply to the
gingiva they are, supraperiosteal arterioles along the facial and lingual surfaces
of the alveolar bone. vessels of the periodontal ligament and arterioles which
emerge from the crest of the interdental septa</td></tr><tr valign=top height="15"
style="background-color:#EFF3FB;"><td width="30" style="border:solid 1px black;
border-collapse:collapse ">177</td><td width="250" style="border:solid 1px black;
border-collapse:collapse ">periodontal diseases are seen more in adults compared to
children because:</td><td width="200" style="border:solid 1px black; border-
collapse:collapse ">A: children have increased vascular supply<br><br>B: children
do have a COL present in the gingiva<br><br>C: in children the oral cavity do not
have gram negative prganisms which cause PDL disease<br><br>D: all of the
above<br><br></td><td width="50" style="border:solid 1px black; border-
collapse:collapse ">D</td><td width="250" style="border:solid 1px black; border-
collapse:collapse ">Reference: Shobha Tandon, 2nd ed, pg no:
Explanation: Less prevalence of periodontal disease is seen in children because:
a. Children have increased vascular supply
b. Col is absent in children which manifest plaque
c. The grame negative organisms are not present in children</td></tr><tr valign=top
height="15" style="background-color:White;"><td width="30" style="border:solid 1px
black; border-collapse:collapse ">178</td><td width="250" style="border:solid 1px
black; border-collapse:collapse ">The resistance form is that shape of cavity
which:</td><td width="200" style="border:solid 1px black; border-collapse:collapse
">A: Prevents displacement of restoration<br><br>B: Permits the restoration to
withstand occlusal forces<br><br>C: Allows aJequate instrumentation<br><br>D:
Allows the restoration to withstand occlusal forces and prevent
displacement<br><br></td><td width="50" style="border:solid 1px black; border-
collapse:collapse ">B</td><td width="250" style="border:solid 1px black; border-
collapse:collapse ">REF :Sturdevant Operative dentistry Page no: 290
EXP : Primary resistance form may be defined as that shape and placement of the
preparation walls that best enable both the restoration and the tooth to withstand,
without fracture, masticatory forces delivered principally in the long axis of the
tooth.</td></tr><tr valign=top height="15" style="background-color:#EFF3FB;"><td
width="30" style="border:solid 1px black; border-collapse:collapse ">179</td><td
width="250" style="border:solid 1px black; border-collapse:collapse ">Beveling the
pulpoaxial line angle of a class II cavity is done to</td><td width="200"
style="border:solid 1px black; border-collapse:collapse ">A: Increase the strength
of the restoration<br><br>B: Improved marginal adaptation<br><br>C: To prevent the
fracture of enamel<br><br>D: To prevent the fracture of amalgam<br><br></td><td
width="50" style="border:solid 1px black; border-collapse:collapse ">D</td><td
width="250" style="border:solid 1px black; border-collapse:collapse ">REF
:Sturdevant Operative dentistry page 290
EXP ; Rounding external angles (those with apices directed externally [e.g.,
axiopulpal line angles]) reduces the stress on some restorative materials (amalgam
and porcelain), thus increasing resistance to fracture of the restorative
material</td></tr><tr valign=top height="15" style="background-color:White;"><td
width="30" style="border:solid 1px black; border-collapse:collapse ">180</td><td
width="250" style="border:solid 1px black; border-collapse:collapse ">The shape of
the dentinal caries in spread of caries is:</td><td width="200"
style="border:solid 1px black; border-collapse:collapse ">A: Always V-shaped with
base towards DE]<br><br>B: Always V-shaped with base towards pulp<br><br>C: Depends
on type of lesion<br><br>D: None of above<br><br></td><td width="50"
style="border:solid 1px black; border-collapse:collapse ">A</td><td width="250"
style="border:solid 1px black; border-collapse:collapse ">REF :Sturdevant Operative
dentistry Page no: 96
EXP : The dentinoenamel junction has the least resistance to caries attack and
allows rapid lateral spreading once caries has penetrated the enamel . Because of
these characteristics, dentinal caries is V-shaped in cross-section with a wide
base at the DEJ
and the apex directed pulpally</td></tr><tr valign=top height="15"
style="background-color:#EFF3FB;"><td width="30" style="border:solid 1px black;
border-collapse:collapse ">181</td><td width="250" style="border:solid 1px black;
border-collapse:collapse ">The main cause of failure of replanted teeth is</td><td
width="200" style="border:solid 1px black; border-collapse:collapse ">A:
Infection<br><br>B: Pulp necrosis<br><br>C: Calcific metamorphosis<br><br>D:
External root resorption<br><br></td><td width="50" style="border:solid 1px black;
border-collapse:collapse ">D</td><td width="250" style="border:solid 1px black;
border-collapse:collapse ">Reference: Endodontics Ingle Page no: 157
Explanation: Replacement resorption occurs when there has been death of the
periodontal ligament cells. Clastic cells, derived from the adjacent bone, cause a
progressive replacement of dentin by bone. Inflammatory
(infective) resorption may be superimposed on replacement resorption. Ultimately,
the tooth is replaced by bone as it is progressively resorbed.</td></tr><tr
valign=top height="15" style="background-color:White;"><td width="30"
style="border:solid 1px black; border-collapse:collapse ">182</td><td width="250"
style="border:solid 1px black; border-collapse:collapse ">what is hypoxic
hypoxia</td><td width="200" style="border:solid 1px black; border-
collapse:collapse ">A: arterial hypoxia<br><br>B: venous hypoxia<br><br>C: anemic
hypoxia<br><br>D: stagnant hypoxia<br><br></td><td width="50" style="border:solid
1px black; border-collapse:collapse ">A</td><td width="250" style="border:solid
1px black; border-collapse:collapse ">Ref: Sembulingam, 2nd ed, pg no: 559
Exp: Hypoxic hypoxia means the decreased oxygen supply to the blood. Also called as
arterial hypoxia</td></tr><tr valign=top height="15" style="background-
color:#EFF3FB;"><td width="30" style="border:solid 1px black; border-
collapse:collapse ">183</td><td width="250" style="border:solid 1px black; border-
collapse:collapse ">Which of the following inhalational agents is the induction
agent of choice in children?</td><td width="200" style="border:solid 1px black;
border-collapse:collapse ">A: Methoxyflurane<br><br>B: Sevoflurane<br><br>C:
Desflurane<br><br>D: Isoflurane<br><br></td><td width="50" style="border:solid 1px
black; border-collapse:collapse ">B</td><td width="250" style="border:solid 1px
black; border-collapse:collapse ">REF ;goodman and gilman text book of pharmacology
EXP ; Sevoflurane is widely used, particularly for outpatient anesthesia, because
of its rapid recovery profile. It is well-suited for inhalation induction of
anesthesia (particularly in children) because it is not irritating to the airway.
Induction of anesthesia is rapidly achieved using inhaled concentrations of
2</td></tr><tr valign=top height="15" style="background-color:White;"><td
width="30" style="border:solid 1px black; border-collapse:collapse ">184</td><td
width="250" style="border:solid 1px black; border-collapse:collapse ">The puLp horn
that is most LikeLy to be exposed during cavity preparation on deciduous moLars
is</td><td width="200" style="border:solid 1px black; border-collapse:collapse
">A: Mesiobuccal of first molar<br><br>B: Distobuccal of first molar<br><br>C:
Mesiobuccal of second molar<br><br>D: Distobuccal of second molar<br><br></td><td
width="50" style="border:solid 1px black; border-collapse:collapse ">A</td><td
width="250" style="border:solid 1px black; border-collapse:collapse ">ref mcdonald
8th edition pg no ; exp;The lingual surface has three cusps: a mesiolingual cusp
that is large and well developed, a distolingual cusp, and a third and smaller
supplemental cusp (cusp of Carabelli). A well-defined groove separates the
mesiolingual cusp from the distolingual cusp. On the occlusal surface a prominent
oblique ridge connects the mesiolingual cusp with the distobuccal cusp .The pulpal
outline of the primary teeth follows the dentoenamel junction more closely than
that of the permanent teeth. The pulpal horns are longer and more pointed than the
cusps would indicate. The dentin also has less bulk or thickness, and so the pulp
is</td></tr><tr valign=top height="15" style="background-color:#EFF3FB;"><td
width="30" style="border:solid 1px black; border-collapse:collapse ">185</td><td
width="250" style="border:solid 1px black; border-collapse:collapse ">Which
orthodontic wire shows shape memory:</td><td width="200" style="border:solid 1px
black; border-collapse:collapse ">A: TMA<br><br>B: NiTi<br><br>C: Co-Cr<br><br>D:
a-Ti<br><br></td><td width="50" style="border:solid 1px black; border-
collapse:collapse ">D</td><td width="250" style="border:solid 1px black; border-
collapse:collapse ">REF ;PROFFIT 4TH EDI PG NO 296
EXP ; The transformation is a
mechanical analogue to the thermally-induced shape memory effect. In other words,
the austenitic alloy undergoes a transition in internal structure in response to
stress, without requiring a significant temperature change (which is possible
because for these materials, the transition temperature is very close to room
temperature). Some currentlymarketed
wires are almost dead soft at room temperature, and become elastic at mouth
temperatures, which can
make them easier to place initially but the exceptional range that goes with
superelasticity is obtainable only if a stress-induced transformation also
occurs</td></tr><tr valign=top height="15" style="background-color:White;"><td
width="30" style="border:solid 1px black; border-collapse:collapse ">186</td><td
width="250" style="border:solid 1px black; border-collapse:collapse ">The Moyers
analyses requires the measurement of the:</td><td width="200" style="border:solid
1px black; border-collapse:collapse ">A: Mesiodistal of the erupted permanent
maxillary centrals and laterals.<br><br>B: Space available in the maxillary and
mandibular posterior quadrants.<br><br>C: Mesiodistal diameter of the unerupted
premolars.<br><br>D: Mesiodistal diameter of the unerupted maxillary and mandibular
permanent cuspids and premolars. [the first option must have been as Mesiodistal of
the erupted permanent MANDIBULAR centrals and laterals]<br><br></td><td width="50"
style="border:solid 1px black; border-collapse:collapse ">D</td><td width="250"
style="border:solid 1px black; border-collapse:collapse ">REF:PROFFIT 4TH EDITION
PG NO 160 EXP;To utilize the Moyers
prediction tables, the mesiodistal width of the lower incisors is measured and this
number is used to predict the size of both the lower and upper unerupted canines
and premolars. The size of the lower incisors correlates better with the size of
the upper canines and premolars than does the size of the upper incisors, because
upper lateral incisors are extremely variable
teeth</td></tr><tr valign=top height="15" style="background-color:#EFF3FB;"><td
width="30" style="border:solid 1px black; border-collapse:collapse ">187</td><td
width="250" style="border:solid 1px black; border-collapse:collapse ">Single tooth
cross bite can be treated by:</td><td width="200" style="border:solid 1px black;
border-collapse:collapse ">A: RemovabLe or fixed appLiances<br><br>B: Expansion
appLiance<br><br>C: ExtraoraL eLastic<br><br>D: Head gear<br><br></td><td
width="50" style="border:solid 1px black; border-collapse:collapse ">A</td><td
width="250" style="border:solid 1px black; border-collapse:collapse ">REF: PROFFIT
4TH EDT PG NO EXP;An anterior crossbite caused by lingual position of the maxillary
incisors can be corrected using (B) a 36 mil lingual arch with soldered 22 mil
finger springs. A guide wire can be placed between the incisors as shown here to
keep the springs from moving incisallY and in older pt fixed appliance therapy is
indicated</td></tr><tr valign=top height="15" style="background-color:White;"><td
width="30" style="border:solid 1px black; border-collapse:collapse ">188</td><td
width="250" style="border:solid 1px black; border-collapse:collapse ">A child who
had a congenital defect of cleft lip and cleft palate is most likely to suffer from
which kind of malocclusion:</td><td width="200" style="border:solid 1px black;
border-collapse:collapse ">A: Bilateral posterior cross bite<br><br>B: A collapsed
anterior mandibular arch<br><br>C: Protrusion and spadng of maxillary anterior
teeth<br><br>D: .Class II division 1 malocclusion<br><br></td><td width="50"
style="border:solid 1px black; border-collapse:collapse ">A</td><td width="250"
style="border:solid 1px black; border-collapse:collapse ">REF; PROFFIT 4TH EDT PG
NO 262 EXP:As the canine and premolar teeth erupt, posterior crossbite is likely to
develop, particularly on the cleft side in a unilateral cleft patient, and the
teeth are likely to be malaligned.</td></tr><tr valign=top height="15"
style="background-color:#EFF3FB;"><td width="30" style="border:solid 1px black;
border-collapse:collapse ">189</td><td width="250" style="border:solid 1px black;
border-collapse:collapse ">The movement of bone in response to its own growth is
termed as :</td><td width="200" style="border:solid 1px black; border-
collapse:collapse ">A: Rotation.<br><br>B: Secondary dispLacement.<br><br>C:
Primary dispLacement.<br><br>D: Differentiation.<br><br></td><td width="50"
style="border:solid 1px black; border-collapse:collapse ">C</td><td width="250"
style="border:solid 1px black; border-collapse:collapse ">ref :prOFFIT 4TH EDT PG
NO; 87 EXP;Growth of the nasomaxillary area is produced by two basic mechanisms:
(1) passive displacement, created by growth in the cranial base that pushes the
maxilla forward, and (2) active growth of the maxillary structures and
nose</td></tr><tr valign=top height="15" style="background-color:White;"><td
width="30" style="border:solid 1px black; border-collapse:collapse ">190</td><td
width="250" style="border:solid 1px black; border-collapse:collapse ">What changes
the condition between troponintropomyosin complex and actin in the relaxed muscle
state?</td><td width="200" style="border:solid 1px black; border-collapse:collapse
">A: Ca and K bind to T tubules.<br><br>B: calcium bind to troponin.<br><br>C:
sodium interact with actin.<br><br>D: interaction with myosin.<br><br></td><td
width="50" style="border:solid 1px black; border-collapse:collapse ">B</td><td
width="250" style="border:solid 1px black; border-collapse:collapse
">Reference:Harper, Biochemistry, 26th ed, pg no:
Explanation:The correct answer is Choice B. Calcium ions bind strongly to troponin
(Choice B) changing the condition between troponintropomyosin complex and actin in
the relaxed muscle state to produce a new relationship that leads to muscle
contraction.
T-tubule (Choice A) facilitates transmission of action potentials to all skeletal
muscle fibers. It does not bind with calcium ions.
Calcium ions also do not bind with actin (choice C) or myosin (Choice D). Actin
must attach to the heads of the myosin filaments to cause contraction.
Sarcoplasmic reticulum (Choice E) releases calcium ion in the vicinity of all
myofibrils to bring about contraction</td></tr><tr valign=top height="15"
style="background-color:#EFF3FB;"><td width="30" style="border:solid 1px black;
border-collapse:collapse ">191</td><td width="250" style="border:solid 1px black;
border-collapse:collapse ">Disadvantage of flexible major connector:</td><td
width="200" style="border:solid 1px black; border-collapse:collapse ">A: Causes
injury to underlying soft and hard tissues<br><br>B: Forces concentrated on
individual tooth<br><br>C: Damages periodontium of supporting teeth<br><br>D: All
of above<br><br></td><td width="50" style="border:solid 1px black; border-
collapse:collapse ">D</td><td width="250" style="border:solid 1px black; border-
collapse:collapse ">REF ; Mc Cracken 11th edi pg no 35
EXP : A major connector is the component of the partial denture that connects the
parts of the prosthesis located on one side of the arch with those on the opposite
side. It is that unit of the partial denture to which all other parts are directly
or indirectly attached.It is through the major connector that other components of
the partial denture become unified and effective. If the major connector is
flexible, the ineffectiveness of connected components jeopardizes the supporting
oral structures and can be a detriment to the comfort of the patient. Failure of
the major connector to provide rigidity may be manifested by traumatic damage to
periodontal support of abutment teeth, injury to residual ridges, or impingement of
underlying tissue.</td></tr><tr valign=top height="15" style="background-
color:White;"><td width="30" style="border:solid 1px black; border-
collapse:collapse ">192</td><td width="250" style="border:solid 1px black; border-
collapse:collapse ">Ethylene oxide is an agent that</td><td width="200"
style="border:solid 1px black; border-collapse:collapse ">A: reversibly inhibits
growth.<br><br>B: is antiseptic.<br><br>C: disinfects.<br><br>D:
sterilizes.<br><br></td><td width="50" style="border:solid 1px black; border-
collapse:collapse ">C</td><td width="250" style="border:solid 1px black; border-
collapse:collapse ">Reference:Conviser S.. "The Future of Ethylene Oxide
Sterilization". ICT Magazine. Retrieved 2009-10-23.
Explanation:Ethylene oxide inhibits growth of microorganisms (disinfectant
properties) and when present in high concentrations, can completely destroy them.
Strong alkylating properties make ethylene oxide a universal poison for protoplasm:
it causes clotting of proteins, deactivation of enzymes and other biologically
important components of a living organism.
Ethylene oxide acts more strongly against bacteria, especially gram-positive
bacteria, than against yeast and fungi.The disinfectant effect of ethylene oxide is
similar to that of sterilization by heat, but because of limited penetration, it
affects only the surface. The Sterility Assurance Level, after a certain specified
exposure to ethylene oxide is 10−6, meaning that the chance of finding a single
bacterium is below 1 per million.</td></tr><tr valign=top height="15"
style="background-color:#EFF3FB;"><td width="30" style="border:solid 1px black;
border-collapse:collapse ">193</td><td width="250" style="border:solid 1px black;
border-collapse:collapse ">A complete clasp assembly consists of a</td><td
width="200" style="border:solid 1px black; border-collapse:collapse ">A: Retenive
arm and a reciprocal arm<br><br>B: Circumferential arm and a bar-type arm<br><br>C:
Proximal plate and a retentive arm<br><br>D: Retentive arm, an occlusal rest and
reciprocating elements<br><br></td><td width="50" style="border:solid 1px black;
border-collapse:collapse ">D</td><td width="250" style="border:solid 1px black;
border-collapse:collapse ">ref nallaswamy 1st ed pg 352: components of clasp,
retentive arm, reciprocal arm, shoulder, rest, body,</td></tr><tr valign=top
height="15" style="background-color:White;"><td width="30" style="border:solid 1px
black; border-collapse:collapse ">194</td><td width="250" style="border:solid 1px
black; border-collapse:collapse ">For a complete denture, the most important area
of the impression In making a final impression is</td><td width="200"
style="border:solid 1px black; border-collapse:collapse ">A: Ridge area of maxilla
and buccal shelf of mandible<br><br>B: Lingual border area of mandible<br><br>C:
Junction of hard and soft palate of maxilla and disto Lingual area of
mandible<br><br>D: Mid palatal area of maxilla and ridge of the
mandible<br><br></td><td width="50" style="border:solid 1px black; border-
collapse:collapse ">C</td><td width="250" style="border:solid 1px black; border-
collapse:collapse ">Ref to pg no 158, 197 and 214 of Boucher’s, 9th edition. The
posterior palatal seal area is an important area in upper final impression whereas
the distolingual area of the mandible is important in the final impression of the
mandible.</td></tr><tr valign=top height="15" style="background-color:#EFF3FB;"><td
width="30" style="border:solid 1px black; border-collapse:collapse ">195</td><td
width="250" style="border:solid 1px black; border-collapse:collapse ">Thyroid
hormones, glucocorticoid gonadal steroids are similar in that each</td><td
width="200" style="border:solid 1px black; border-collapse:collapse ">A: is derived
from cholesterol.<br><br>B: is derived from amino acids.<br><br>C: acts on only one
target organ or tissue.<br><br>D: is released in response to signals from the
hypothalamic-anterior pituitary complex.<br><br></td><td width="50"
style="border:solid 1px black; border-collapse:collapse ">D</td><td width="250"
style="border:solid 1px black; border-collapse:collapse ">Reference: Guyton,
Physiology, 11th ed, pg no:920
Explanation:The hormones of the anterior pituitary play major roles in the control
of metabolic functions throughout the body.
• Growth hormone promotes growth of the entire body by affecting protein formation,
cell multiplication, and cell differentiation.
• Adrenocorticotropin (corticotropin) controls the secretion of some of the
adrenocortical hormones, which affect the metabolism of glucose, proteins, and
fats.
• Thyroid-stimulating hormone (thyrotropin) controls the rate of secretion of
thyroxine and triiodothyronine by the thyroid gland, and these hormones control the
rates of most intracellular chemical reactions in the body.
• Prolactin promotes mammary gland development and milk production.
• Two separate gonadotropic hormones, follicle-stimulating hormone and luteinizing
hormone, control growth of the ovaries and testes, as well as their hormonal and
reproductive activities.
The two hormones secreted by the posterior pituitary play other roles.
• Antidiuretic hormone (also called vasopressin) controls the rate of water
excretion into the urine, thus helping to control the concentration of water in the
body fluids.
• Oxytocin helps express milk from the glands of the breast to the nipples during
suckling and possibly helps in the delivery of the baby at the end of
gestation.</td></tr><tr valign=top height="15" style="background-color:White;"><td
width="30" style="border:solid 1px black; border-collapse:collapse ">196</td><td
width="250" style="border:solid 1px black; border-collapse:collapse ">Bromocresol
green dye used in carie activity test acts as:</td><td width="200"
style="border:solid 1px black; border-collapse:collapse ">A: pH indicator in.
salivary activity test<br><br>B: Changes colour from green to yello<br><br>C:
Indicates the degree of caries activi<br><br>D: None of the above<br><br></td><td
width="50" style="border:solid 1px black; border-collapse:collapse ">C</td><td
width="250" style="border:solid 1px black; border-collapse:collapse ">ref soben
peter 4th ed pg 433: it is a colorimetric caries indicator test</td></tr><tr
valign=top height="15" style="background-color:#EFF3FB;"><td width="30"
style="border:solid 1px black; border-collapse:collapse ">197</td><td width="250"
style="border:solid 1px black; border-collapse:collapse ">Distribution of dental
caries in a community is best described by:</td><td width="200"
style="border:solid 1px black; border-collapse:collapse ">A: Descriptive and
experimental study<br><br>B: Descriptive and analytical study<br><br>C:
Descriptive, analytical and experimental study<br><br>D: Descriptive study
only<br><br></td><td width="50" style="border:solid 1px black; border-
collapse:collapse ">D</td><td width="250" style="border:solid 1px black; border-
collapse:collapse ">REF: soben peter 4th ed pg 51,58 : by definitions of
descriptive and analytical studies, we can conclude that analytical is aimed at an
individual level but here in the question the community as a whole is considered
EXP :</td></tr><tr valign=top height="15" style="background-color:White;"><td
width="30" style="border:solid 1px black; border-collapse:collapse ">198</td><td
width="250" style="border:solid 1px black; border-collapse:collapse ">Which of the
following vaccine is not included in EPI schedule?</td><td width="200"
style="border:solid 1px black; border-collapse:collapse ">A: DPT<br><br>B:
MMR<br><br>C: BCG<br><br>D: OPV<br><br></td><td width="50" style="border:solid 1px
black; border-collapse:collapse ">B</td><td width="250" style="border:solid 1px
black; border-collapse:collapse ">ref:Park 20th edi pg no 112-113 , table 41
exp : In May 1974, the WHO officially launched a global immunization programme,
known as Expanded Programme on Immunization (EPI) to protect all children of the
world against six vaccine-preventable diseases, namely diphtheria, whooping cough,
tetanus, polio, tuberculosis and measles by the year 2000. EPI was launched in
India in January 1978.</td></tr><tr valign=top height="15" style="background-
color:#EFF3FB;"><td width="30" style="border:solid 1px black; border-
collapse:collapse ">199</td><td width="250" style="border:solid 1px black; border-
collapse:collapse ">Which characteristic of a diagnostic test listed below measures
the proportion of people with a disease who are correctly identified by a positive
test?</td><td width="200" style="border:solid 1px black; border-collapse:collapse
">A: Specificity<br><br>B: Sensitivity<br><br>C: Correctness<br><br>D:
Perfectness<br><br></td><td width="50" style="border:solid 1px black; border-
collapse:collapse ">B</td><td width="250" style="border:solid 1px black; border-
collapse:collapse ">Specificity measures the proportion of those without disease
who are correctly identified by the negative test Note: Sensitivity and specificity
are inversely proportional as the specificity of a test increases. The sensitivity
decreases. Specificity is defined by the number of true negative (TN) results
divided by the total number of false positive (FP) plus true negative (TN) results
in the sample. Summary: Specificity = TN I FP + TN. Sensitivity is defined as the
number of true positives (TP) divided by the total number of potential positive
findings (true positives and false negatives) in the sample. Summary: Sensitivity =
TP I TP + FN.</td></tr><tr valign=top height="15" style="background-
color:White;"><td width="30" style="border:solid 1px black; border-
collapse:collapse ">200</td><td width="250" style="border:solid 1px black; border-
collapse:collapse ">The epidemiological studies include the following types of
study</td><td width="200" style="border:solid 1px black; border-collapse:collapse
">A: preventive<br><br>B: restorative<br><br>C: statistical<br><br>D:
analytical<br><br></td><td width="50" style="border:solid 1px black; border-
collapse:collapse ">D</td><td width="250" style="border:solid 1px black; border-
collapse:collapse ">reference Soben Peter 2nd edition pg no 92, epidemiology has
three distinct divisions 1) descriptive, 2) analytical ad 3)
eperimental.</td></tr></table><br><br><center><input type="button" value="Print"
onclick="window.print();"></center>

Das könnte Ihnen auch gefallen